PDA

Archiv verlassen und diese Seite im Standarddesign anzeigen : Das relativistische Zwillingsparadoxon


Semmelweis
02.08.15, 19:48
Das relativistische Zwillingsparadoxon ist bizarr.
Angenommen, ein Raumschiff enfernt sich mit relativistischer Geschwindigkeit von der Erde. Es ist vereinbart, dass es ein Lichtjahr entfernt stoppt, so dass die Relativgeschwindigkeit zur Erde dann vernachlässigbar klein ist.
Nun sind beide der Meinung, dass der jeweils andere in Zeitlupe zu sehen war und glauben, dass die Uhr des jeweils anderen (die beim Start natürlich synchronisiert wurde) ein kleineres Datum anzeigt als die eigene.
Jetzt könnte man sich doch über normale lichtschnelle Signale verständigen und der Raumfahrer einfach fragen: "Bei mir ist der 2. August 2015, 20:45. Welches Datum ist bei euch auf der Erde? Bitte um sofortige Antwort". Dann könnte er zwei Jahre später doch mit der Antwort herausfinden, was wirklich geschehen ist?
Sie befinden sich ja immer noch in der gleichen Realität und können miteinander kommunizieren.

Ich
02.08.15, 20:26
Nun sind beide der Meinung, dass der jeweils andere in Zeitlupe zu sehen war und glauben, dass die Uhr des jeweils anderen (die beim Start natürlich synchronisiert wurde) ein kleineres Datum anzeigt als die eigene.
Natürlich.
Wobei das kein Wunder ist, die Signale der Erduhr kommen ja erst ein Jahr später an. Auf jeden Fall sind die hintendran.
Jetzt könnte man sich doch über normale lichtschnelle Signale verständigen und der Raumfahrer einfach fragen: "Bei mir ist der 2. August 2015, 20:45. Welches Datum ist bei euch auf der Erde? Bitte um sofortige Antwort". Dann könnte er zwei Jahre später doch mit der Antwort herausfinden, was wirklich geschehen ist?
Dann kommt raus, dass die Uhr im Raumschiff weniger Zeit anzeigt.

Zeitvergleiche an unterschiedlichen Orten sind aber vom verwendeten Bezugssystem abhängig. Hier habe ich das verwendet, in dem beide am Schluss in Ruhe sind. Hätte ich eines verwendet, in dem das Raumschiff während des Flugs in Ruhe war, wäre der Vergleich andersherum ausgefallen.
Eindeutigkeit gibt es nur, wenn beide Beobachter am selben Ort sind.
Das sollte man berücksichtigen, wenn man von dem spricht, was "wirklich" geschehen ist.

TomS
02.08.15, 20:31
Eindeutigkeit gibt es nur, wenn beide am selben Ort sind.
Genau. Und deswegen schlage ich vor, das sogenannte Zwillingsparadoxon für den Fall der Rückkehr zum Ausgangspunkt zu analysieren. Wie Ich sagt, liegt dabei Eindeutigkeit vor;und es wird klar, das es kein Paradoxon gibt.

Harti
03.08.15, 08:34
Hallo TomS,

Genau. Und deswegen schlage ich vor, das sogenannte Zwillingsparadoxon für den Fall der Rückkehr zum Ausgangspunkt zu analysieren. Wie Ich sagt, liegt dabei Eindeutigkeit vor;und es wird klar, das es kein Paradoxon gibt.

Wie ist es möglich, dass der Reisezwilling während der Reise nichts davon merkt, dass er langsamer altert als sein Bruder auf der Erde, und beim Wiedersehen plötztlich (wann genau ?) merkt, dass er jünger ist ?

MfG
Harti

Ich
03.08.15, 08:46
Wie soll er das merken? Auf den Kalender schauen?
Der Witz ist ja, dass für ihn alles normal ist. Das folgt direkt aus dem Relativitätsprinzip. Es gibt kein einziges (lokales) Experiment, mit dem sein Bewegungszustand feststellbar wäre.

Semmelweis
03.08.15, 10:44
Wie soll er das merken? Auf den Kalender schauen?
Der Witz ist ja, dass für ihn alles normal ist. Das folgt direkt aus dem Relativitätsprinzip. Es gibt kein einziges (lokales) Experiment, mit dem sein Bewegungszustand feststellbar wäre.

Er kann sich doch aber über die wirkliche Realität auf der Erde informieren, wenn die Antwort lauten würde: "2.August 2016", die dann am 2. August 2017 bei ihm eintrifft weil er ja genau 1 LJ entfernt ist, dann wüßte er, dass seine Wahrnehmung einer Erde in Zeitlupe nur eine Fata Raumgana war.
Denn wenn er ein gutes Teleskop an Bord hätte und die Erduhr dauernd beobachten würde müßte er laut dem Zwillingsparadoxon auch in Ruhe noch immer eine dauerhafte Zeitverschiebung beobachten, die ja nicht einfach so verschwinden kann.

Es würde sich dann die paradoxe Situation ergeben, dass er mit seinem Teleskop ablesen kann "27.Juli 2016" obwohl ihm die Erde "2.August 2016" als menschliche Antwort gibt und beide Signale am 2.August 2017 wahrgenommen werden können.

Marco Polo
03.08.15, 11:56
Es würde sich dann die paradoxe Situation ergeben, dass er mit seinem Teleskop ablesen kann "27.Juli 2016" obwohl ihm die Erde "2.August 2016" als menschliche Antwort gibt und beide Signale am 2.August 2017 wahrgenommen werden können.

Was man mit dem Teleskop abliest, stimmt aber nicht mit den Messvorhersagen gemäß SRT überein, da beim Ablesen noch der Dopplereffekt dazu kommt.

Semmelweis
03.08.15, 12:38
Was man mit dem Teleskop abliest, stimmt aber nicht mit den Messvorhersagen gemäß SRT überein, da beim Ablesen noch der Dopplereffekt dazu kommt.

Die Erde und das Raumschiff befinden sich ja in Ruhe zueinander, und beide könnten mit ihren Teleskopen beim jeweils anderen eine kleinere Uhrzeit ablesen. Um herauszufinden, welche Uhr nun tatsächlich nachgeht müßten sie nur kommunizieren und genau 1 Jahr abziehen, da das Signal 1 Jahr zum Empfänger braucht. Es geht ja nicht um irgendwelche Ungenauigkeiten sondern um die einfache Feststellung, welche Uhr nun gegenüber der anderen wirklich nachgeht.

Um das ganze auf die Spitze zu treiben: Sie könnten sich durch Kommunikation doch wohl einigen, welche Uhr von beiden 1000 Jahre nachgeht wenn sie sich in Ruhe zueinander befinden und einfach nur die Signallaufzeit berücksichtigen müssen, die ja bekannt ist.

Ich
03.08.15, 12:45
Er kann sich doch aber über die wirkliche Realität auf der Erde informieren, wenn die Antwort lauten würde: "2.August 2016", die dann am 2. August 2017 bei ihm eintrifft weil er ja genau 1 LJ entfernt ist, dann wüßte er, dass seine Wahrnehmung einer Erde in Zeitlupe nur eine Fata Raumgana war.
Das wäre aber nicht die Antwort.


Denn wenn er ein gutes Teleskop an Bord hätte und die Erduhr dauernd beobachten würde müßte er laut dem Zwillingsparadoxon auch in Ruhe noch immer eine dauerhafte Zeitverschiebung beobachten, die ja nicht einfach so verschwinden kann.Ja. ***EDIT: Kommt drauf an, was du mit "Zeitverschiebung" meinst. Ich nehme das "Ja" also zurück und lasse die Frage unbeantwortet, bis sie mal eindeutig definiert gestellt ist.

Es würde sich dann die paradoxe Situation ergeben, dass er mit seinem Teleskop ablesen kann "27.Juli 2016" obwohl ihm die Erde "2.August 2016" als menschliche Antwort gibt und beide Signale am 2.August 2017 wahrgenommen werden können....wenn die Antwort "2. August 2016" wäre. Sie ist aber "8. August 2016", und genau das liest er auch an seinem Teleskop ab.

Die ganze Signalschickerei kannst du dir auch sparen. Weil beide im selben Bezugssystem ruhen, reicht es, wenn er direkt am Ziel den 8. August 2014 an der Erde abliest und ein Jahr Signallaufzeit dazurechnet.

Semmelweis
03.08.15, 13:06
Das wäre aber nicht die Antwort.


Ja.

...wenn die Antwort "2. August 2016" wäre. Sie ist aber "8. August 2016", und genau das liest er auch an seinem Teleskop ab.

Die ganze Signalschickerei kannst du dir auch sparen. Weil beide im selben Bezugssystem ruhen, reicht es, wenn er direkt am Ziel den 8. August 2014 an der Erde abliest und ein Jahr Signallaufzeit dazurechnet.

Gut, extrem Formuliert kann er dann der Erde senden: "Ich sehe, dass eure Uhr 1000 Jahre nachgeht", und die Erde antwortet ihm das Gleiche, weil sie das aus ihrer Perspektive auch so sieht.

Wenn nun aber beide Parteien die Signallaufzeit kennen können sie sich doch über ihre wirkliche Uhrzeit unterhalten.

Ich
03.08.15, 13:24
Gut, extrem Formuliert kann er dann der Erde senden: "Ich sehe, dass eure Uhr 1000 Jahre nachgeht", und die Erde antwortet ihm das Gleiche, weil sie das aus ihrer Perspektive auch so sieht.
Wieso sollte sie das auch so sehen?
Wenn nun aber beide Parteien die Signallaufzeit kennen können sie sich doch über ihre wirkliche Uhrzeit unterhalten. Die Signallaufzeit ist abhängig vom Bezugssystem.

Semmelweis
03.08.15, 13:39
Wieso sollte sie das auch so sehen?
Die Signallaufzeit ist abhängig vom Bezugssystem.

Na, Es ist doch so dass beide Parteien sich gegenseitig in Zeitlupe sehen, wenn das Raumschiff sich von der Erde entfernt. Theoretisch kann sich das zu tausenden Jahren Zeitdifferenz aufsummieren und beide Parteien sind dann der Meinung, dass die Uhr des anderen tausende Jahre nachgeht.

Das Bezugssystem ist doch ziemlich egal, wenn sich beide zueinander in Ruhe befinden und sich über ihre eklatante Unstimmigkeit unterhalten.
Es geht ja nicht um Nuancen, sondern um die einfache Tatsache, dass nicht beide Uhren zugleich tausende Jahre nachgehen können.

Ich
03.08.15, 13:44
Na, Es ist doch so dass beide Parteien sich gegenseitig in Zeitlupe sehen, wenn das Raumschiff sich von der Erde entfernt. Theoretisch kann sich das zu tausenden Jahren Zeitdifferenz aufsummieren und beide Parteien sind dann der Meinung, dass die Uhr des anderen tausende Jahre nachgeht.Aber doch nicht, wenn sie die Uhren so vergleichen wie du vorschlägst. Die Situation ist doch überhaupt nicht symmetrisch. Der Vergleich ergibt, dass die Raumschiffuhr nachgeht, Punkt.
Klar sehen sich beide in Zeitlupe. Aber nicht gleich lang.
Das Bezugssystem ist doch ziemlich egal, wenn sich beide zueinander in Ruhe befinden und sich über ihre eklatante Unstimmigkeit unterhalten.
Es gibt keine Unstimmigkeit. Beide sind der Meinung, dass die Raumschiffuhr nachgeht.

Semmelweis
03.08.15, 13:56
Aber doch nicht, wenn sie die Uhren so vergleichen wie du vorschlägst. Die Situation ist doch überhaupt nicht symmetrisch. Der Vergleich ergibt, dass die Raumschiffuhr nachgeht, Punkt.
Klar sehen sich beide in Zeitlupe. Aber nicht gleich lang.
Es gibt keine Unstimmigkeit. Beide sind der Meinung, dass die Raumschiffuhr nachgeht.

Sobald sich die beiden Gegenseitig in Zeitlupe sehen kann sich eine Zeitdifferenz von tausenden Jahren ergeben wenn der Raumfahrer sich nur weit genug mit relativistischer Geschwindigkeit entfernt, ob nun Symmetrie vorliegt oder nicht und egal ob das nun gleich lang passiert oder nicht.
Ist das korrekt?

Ich
03.08.15, 14:09
Ja. Aber natürlich nicht bei einer Endentfernung von einem Lichtjahr.

Bauhof
03.08.15, 14:13
Wenn nun aber beide Parteien die Signallaufzeit kennen können sie sich doch über ihre wirkliche Uhrzeit unterhalten.

Hallo Semmelweis,

für den daheimgebliebenen Zwilling und ebenso für den reisenden Zwilling ist die "wirkliche Uhrzeit" zu jedem Zeitpunkt derjenige Zeigerstand, den beide auf ihren mitgeführten Uhren ablesen.

Entscheidend für die spätere Differenz zwischen den beiden Uhren ist die Größe der Relativgeschwindigkeit zwischen den beiden Zwillingen und die Dauer der Reisezeit.

Weiter entscheidend ist, dass der reisende Zwilling umkehrt und dabei das Inertialsystem wechselt. Deswegen ist das Relativitätsprinzip ab dem Umkehrzeitpunkt nicht mehr anwendbar. Ebenso ist das Relativitätsprinzip während der Beschleunigungsphasen nicht anwendbar. Warum? Weil nur der reisende Zwilling beschleunigt, nicht aber der ruhende Zwilling.

Die Signallaufzeiten haben keine Einfluss auf die spätere Differenz zwischen den beiden Uhren, wenn sich die Zwillinge wieder auf der Erde treffen.

Ein grobes Missverständnis wäre es, die Lichtlaufzeiten für den relativistischen Effekt der Einsteinschen Zeitdilatation verantwortlich zu machen. Verantwortlich ist der nichteuklidische Charakter der Minkowski-Raumzeit.

M.f.G. Eugen Bauhof

P.S.
Falls du das nicht verstanden hast, dann frage nach und sage uns, was du nicht verstanden hast. Aber unterlasse es, wieder neue Behauptungen aufzustellen, die jeder Grundlage entbehren.

TomS
03.08.15, 14:27
Ich gebe die Hoffnung nicht auf und verweise auf meine Darstellung hier:

http://www.physikerboard.de/topic,37752,-faq---zeitdilatation-und-zwillingsparadoxon.html

Bauhof
03.08.15, 14:52
Ich gebe die Hoffnung nicht auf und verweise auf meine Darstellung hier:

http://www.physikerboard.de/topic,37752,-faq---zeitdilatation-und-zwillingsparadoxon.html

Hallo TomS,

deine Darstellung ist nicht nur korrekt, sondern auch eine Verallgemeinerung auf die ART, danke dafür. Aber ich bezweifle, dass das Semmelweis nachvollziehen kann. Seine Reaktionen werden es zeigen.

M.f.G. Eugen Bauhof

Harti
03.08.15, 16:16
Hallo Eugen Bauhof,

Weiter entscheidend ist, dass der reisende Zwilling umkehrt und dabei das Inertialsystem wechselt. Deswegen ist das Relativitätsprinzip ab dem Umkehrzeitpunkt nicht mehr anwendbar.

Bedeutet dies, dass das Zwillingsparadoxon mit der SRT nicht lösbar ist, weil das Relativitätsprinzip ein Postulat der SRT ist ?

MfG
Harti

Marco Polo
03.08.15, 16:43
Bedeutet dies, dass das Zwillingsparadoxon mit der SRT nicht lösbar ist, weil das Relativitätsprinzip ein Postulat der SRT ist ?

Nein. Das bedeutet, dass ab dem Inertialsystemwechsel des Reisezwillings keine Symmetrie zwischen den beiden Bezugssystemen mehr vorhanden ist, weil Beschleunigungen in der SRT nicht relativ sind.

Bauhof
03.08.15, 17:01
Hallo Eugen Bauhof,

Bedeutet dies, dass das Zwillingsparadoxon mit der SRT nicht lösbar ist, weil das Relativitätsprinzip ein Postulat der SRT ist ?

MfG
Harti

Hallo Harti,

nein, das bedeutet das nicht. Da hast du wieder mal etwas völlig falsch verstanden. Das Relativitätsprinzip gilt für alle Beobachter, die mit konstanter Geschwindigkeit relativ zueinander bewegt sind. Der reisende Zwilling reist nicht mit konstanter Geschwindigkeit, z.B. weil er wenden muss, um zurückkehren zu können.

Bitte lies dies nach in unserer Arbeitsplattform SRT (http://www.manus-zeitforum.de/1/41595/Arbeitsplattform_SRT) im Zeitforum. Hieraus ein Zitat:

Relativitätsprinzip:
Zentraler Begriff der Relativitätstheorie, nach der die physikalischen Gesetze für alle Beobachter, die mit konstanter Geschwindigkeit relativ zueinander bewegt sind, die gleiche Form haben; daher ist jeder dieser Beobachter gleichermaßen zu der Behauptung berechtigt, er befinde sich in Ruhe. Dieses Prinzip wird in der allgemeinen Relativitätstheorie zum Äquivalenzprinzip erweitert. Bereits in der Newtonschen Mechanik waren alle Inertialsysteme gleichberechtigt. Die Newtonschen Axiome der Mechanik zeichnen kein Inertialsystem gegenüber dem anderen aus. Der Übergang von einem Inertialsystem in ein anderes Inertialsystem wird in der Newtonschen Mechanik durch eine Galilei-Transformation vollzogen.

Es ist genau umgekehrt: Es würde keine Zeitdilatation in Erscheinung treten, falls das Relativitätsprinzip trotz Zwillingsumkehr weiterhin gültig wäre.

Das Zwillingsparadoxon ist allein mit der Speziellen Relativitätstheorie ohne Widerspruch auflösbar. Die ART kann das natürlich auch (und zwar ganz allgemein), ist aber dazu nicht notwendig.

M.f.G. Eugen Bauhof

TomS
03.08.15, 17:02
HBedeutet dies, dass das Zwillingsparadoxon mit der SRT nicht lösbar ist, weil das Relativitätsprinzip ein Postulat der SRT ist ?
Nein, das bedeutet, dass man zur Lösung auch beschleunigte Bewegungen betrachten muss, was im Rahmen der SRT möglich ist.

Es bedeutet auch, dass die Erklärungen, die auf Lorentz-Transformationen und Inertialsystemen basieren, den Kern der Argumentation verfehlen, da man besser mit (invarianten) Eigenzeiten argumentiert.

Siehe auch dazu der von mir verlinkte Beitrag.

Semmelweis
03.08.15, 20:45
Ja. Aber natürlich nicht bei einer Endentfernung von einem Lichtjahr.

Wie soll das denn gehen, dass sich die beiden Parteien nach der Kommunikation einig darüber sind, dass nur die Uhr des Raumfahrers nachgeht, wenn dieser doch eindeutig Beobachten kann, dass auch die Uhr auf der Erde nachgeht?
Solche Paradoxien sind für mich ein Hinweis, dass etwas mit der Betrachtung der Vorgänge nicht stimmt.

okotombrok
03.08.15, 21:03
Solche Paradoxien sind für mich ein Hinweis, dass etwas mit der Betrachtung der Vorgänge nicht stimmt.

Hallo Semmelweis,
da sehe ich für dich nur zwei Möglichkeiten:

Entweder suchst du den Fehler bei Einstein und den unzähligen Wissenschaftlern nach ihm in Verbindung mit Ignoranz aller empirischen Bestätigungen,
oder du suchst den Fehler bei dir.:(

Mein Tipp: gehe etwas unvoreingenommener heran; ein Verständnis stellt sich erst nach und nach ein und nutze vor allem die Kompetenz einiger Mitglieder hier, die gibt es nicht in allen Foren.


mfg okotombrok

TomS
03.08.15, 23:07
Wie soll das denn gehen, dass sich die beiden Parteien nach der Kommunikation einig darüber sind, dass nur die Uhr des Raumfahrers nachgeht, wenn dieser doch eindeutig Beobachten kann, dass auch die Uhr auf der Erde nachgeht?
Solche Paradoxien sind für mich ein Hinweis, dass etwas mit der Betrachtung der Vorgänge nicht stimmt.
Wenn zwei Beobachter zwei unterschiedliche Routen durch die Raumzeit zurücklegen, dann besteht überhaupt kein logischer Grund, dass ihre Zeiten übereinstimmen. Wenn beide dann wieder zusammentreffen, sind unterschiedliche Eigenzeiten immer noch kein Paradoxon, solange ihre Routen nicht völlig symmetrisch waren.

TheoC
04.08.15, 00:09
Ich gebe die Hoffnung nicht auf und verweise auf meine Darstellung hier:

http://www.physikerboard.de/topic,37752,-faq---zeitdilatation-und-zwillingsparadoxon.html

Hi TomS

könntest du mir bitte "in natürlicher Sprache" erklären was passiert, wenn eine Raumschiff von der Erde startet, ständig mit 1g beschleuigt, und nach einem grossen Kreis wieder nach einem Jahr an der Erde vorbeifliegt?

Wie "sieht" dieser Vorgang aus der Sicht eines Beobachters auf der Erde aus, und wie aus der Sicht des Raumschiffpiloten? Welche Uhrzeit zeigt welche Uhr an?

Danke + lg

Theo

Slash
04.08.15, 05:34
Hi TomS

könntest du mir bitte "in natürlicher Sprache" erklären was passiert, wenn eine Raumschiff von der Erde startet, ständig mit 1g beschleuigt, und nach einem grossen Kreis wieder nach einem Jahr an der Erde vorbeifliegt?


Wenn zwei Beobachter zwei unterschiedliche Routen durch die Raumzeit zurücklegen, dann besteht überhaupt kein logischer Grund, dass ihre Zeiten übereinstimmen. Wenn beide dann wieder zusammentreffen, sind unterschiedliche Eigenzeiten immer noch kein Paradoxon, solange ihre Routen nicht völlig symmetrisch waren.



Hallo TheoC,

ich versuche mich einmal, nachdem, was ich verstanden habe (nicht unbedingt aus TomS Link, sondern insgesamt). Das kann auch falsch sein, was ich schreibe (deshalb bitte gerne korrigieren).

Also: Erde und Raumschiff bewegen sich beide mit einem Betrag c ihres jeweiligen Vierervektors (Vierergeschwindigkeit) im Minkowskiraum.
https://de.wikipedia.org/wiki/Vierervektor#Vierergeschwindigkeit

Frage: Ist dies richtig?

c ist die imaginäre Achse (eine Länge), dx1/dt, dx2/dt, dx3/dt die Geschwindikgeiten bzgl. den reellen Raumachsen.

Jetzt kurvt das Raumschiff irgendwie durch das Weltall und die Erde auch und wie es so ist, werden die Geschwindigkeiten hochintegriert und ergeben die Position.

Am Ende sind Raumschiff und Erde wieder zusammen (im Raum). Das bedeutet hochintegriert haben sich die Raumpositionen wieder getroffen.
Da das Raumschiff vielmehr herumgekurvt ist, als die Erde, bedeutet es - hochintegriert, dass die "Positionen" in der Zeit anders sind (denn der Betrag der Vierergeschwindigkeit war immer konstant c).

VG
Slash

TomS
04.08.15, 05:46
@TheoC: Meinst du, dass das Raumschiff mit v = const. entlang des Kreises fliegt, wobei v so gewählt ist, dass die Raumschiffbesatzung eine Beschleunigung von a = 1g spürt? So wie in dem Link beschrieben? (also nicht konstante Beschleunigung in Tangentialrichtung mit wachsendem v).

Und was meinst du mit "wie sieht dieser Vorgang aus der Sicht eines Beobachters auf der Erde aus?" Meinst du die Eigenzeit T2 des Reisenden, die der Zwilling auf der Erde jeweils seiner Eigenzeit T1 zuordnet, also T2 = f(T1), d.h. als Funktion von T1? Oder meinst du, welche Zeit T2' der Zwilling auf der Erde zu seiner Zeit T1 auf der Uhr des Reisenden abliest? (wenn er diese mittels eines Fernrohrs beobachtet, oder wenn der Reisende ihm Lichtsignale mit seiner kodierten Zeitinformation zuschickt?) Letzteres bezieht außerdem noch die Lichtlaufzeit vom Reisenden zur Erde mit ein, d.h. die Berechnung ist deutlich komplizierter, ohne dass wir dadurch mehr physikalische Einsicht gewinnen würden.

Betrachten wir zunächst v = const. und T2 = f(T1) ohne Berücksichtigung der Lichtlaufzeit. T2 ist nach der Rückkehr um das in der vorletzten Formel berechnete Delta kleiner. Wegen konstantem Geschwindigkeitsbetrag entlang der Reise ist außerdem T2/T1 = q entlang der Reiseroute konstant. Der Reisende altert also unterwegs konstant um diesen Faktor langsamer. Der Zwilling auf der Erde kann zu jeder eigenen Eigenzeit T1 die aus seiner Sicht "jetzt" gültige Eigenzeit T2 = f(T1) = q * T1 des Reisenden berechnen. Umgekehrt kann der Reisende zu jeder Eigenzeit T2 die aus seiner Sicht "jetzt" gültige Eigenzeit T1 = T2 / q des Zwillings auf der Erde berechnen.

Achtung! Da der Reisende kein Inertialsysteme definiert, ist diese Berechnung rein formal und hat nichts mit einer Lorentztransformation zu tun. Die Eigenzeit des Reisenden definiert keine global gültige Koordinatenzeit.

Soviel zur Berechnung T2 = f(T1). Bekommst du das mit der Lichtlaufzeit selbst hin? Das ist nur elementare Geometrie. Man berechnet zunächst T2 = f(T1); als nächstes berechnet man für dieses T1 die Position auf dem Kreis sowie die daraus resultierende Lichtlaufzeit Tc. D.h. das Lichtsignal mit der Zeitinformation T2 = q * T1 kommt auf der Erde bei T1' = T1 + Tc an.

Ich
04.08.15, 07:50
Wie soll das denn gehen, dass sich die beiden Parteien nach der Kommunikation einig darüber sind, dass nur die Uhr des Raumfahrers nachgeht, wenn dieser doch eindeutig Beobachten kann, dass auch die Uhr auf der Erde nachgeht?
Solche Paradoxien sind für mich ein Hinweis, dass etwas mit der Betrachtung der Vorgänge nicht stimmt.
Der nächste Beitrag dieser Art außerhalb des dafür vorgesehenen Unterforums zieht eine Schreibsperre nach sich.

-Ich-

Ich
04.08.15, 08:04
Wie soll das denn gehen, dass sich die beiden Parteien nach der Kommunikation einig darüber sind, dass nur die Uhr des Raumfahrers nachgeht, wenn dieser doch eindeutig Beobachten kann, dass auch die Uhr auf der Erde nachgeht?
Solche Paradoxien sind für mich ein Hinweis, dass etwas mit der Betrachtung der Vorgänge nicht stimmt.
Nehmen wir an, er kann in einer Stunde bremsen. Dann liest er sowohl vor wie nach dem Bremsvorgang am Teleskop den 8. August 2014 ab.
Gibt er ein Jahr dazu, stellt er fest, dass seine Uhr langsamer ging, er hat ja erst den 2. August 2015.
Lichtlaufzeiten sind abhängig vom Bezugsyystem, das habe ich dir schon erklärt. Im bewegten System vor dem Bremsen hätte er sie zwölf Tage (*) kürzer angesetzt. Er hätte also zum 8. August 2014 kein ganzes Jahr dazu addiert und wäre beim 27. Juli 2015 rausgekommen. Daraus hätte er geschlossen, dass die Erduhr nachgeht.
Da ist kein Paradox.

(*) Die genauen Zahlen hängen von Reisedauer und -geschwindigkeit ab, die du nicht spezifiziert hast. Falls du jemals lernen willst, wie das alles funktioniert. müsste man mal ein vernünftiges, konkretes Beispies aufsetzen.

Ich
04.08.15, 08:20
Also: Erde und Raumschiff bewegen sich beide mit einem Betrag c ihres jeweiligen Vierervektors (Vierergeschwindigkeit) im Minkowskiraum.
https://de.wikipedia.org/wiki/Vierervektor#Vierergeschwindigkeit

Frage: Ist dies richtig?Es ist richtig, dass der Betrag der Vierergeschwindigkeit immer gleich c (oder 1, je nach Einheiten) ist.
c ist die imaginäre Achse (eine Länge), dx1/dt, dx2/dt, dx3/dt die Geschwindikgeiten bzgl. den reellen Raumachsen.c ist die Lichtgeschwindigkeit. Die kannst du auch gleich 1 setzen.
Der Abschnitt "Interpretation" ist bis auf die Formel kompletter Käse. Das hat nichts mit der Vierergeschwindigkeit zu tun.
Da das Raumschiff vielmehr herumgekurvt ist, als die Erde, bedeutet es - hochintegriert, dass die "Positionen" in der Zeit anders sind (denn der Betrag der Vierergeschwindigkeit war immer konstant c).Man kann das so rechnen, aber das hat wie gesagt nichts mit der Vierergeschwindigkeit zu tun. Was du da machst: du ordnest im ruhenden Bezugssystem jedem dt ein dtau (Eigenzeit) zu, das derweil vergeht. Und das integrierst du dann auf:
tau = \int_{tau_0}^{tau_1} dtau ->
tau = \int_{t_0}^{t_1} dtau/dt * dt,
wobei dtau/dt das Inverse des Gammafaktors ist. Dieses Integral wurde schon in der Originalveröffentlichung von Einstein angegeben, bevor man überhaupt an Vierervektoren dachte.

Semmelweis
04.08.15, 12:27
Nehmen wir an, er kann in einer Stunde bremsen. Dann liest er sowohl vor wie nach dem Bremsvorgang am Teleskop den 8. August 2014 ab.
Gibt er ein Jahr dazu, stellt er fest, dass seine Uhr langsamer ging, er hat ja erst den 2. August 2015.
Lichtlaufzeiten sind abhängig vom Bezugsyystem, das habe ich dir schon erklärt. Im bewegten System vor dem Bremsen hätte er sie zwölf Tage (*) kürzer angesetzt. Er hätte also zum 8. August 2014 kein ganzes Jahr dazu addiert und wäre beim 27. Juli 2015 rausgekommen. Daraus hätte er geschlossen, dass die Erduhr nachgeht.
Da ist kein Paradox.

(*) Die genauen Zahlen hängen von Reisedauer und -geschwindigkeit ab, die du nicht spezifiziert hast. Falls du jemals lernen willst, wie das alles funktioniert. müsste man mal ein vernünftiges, konkretes Beispies aufsetzen.

Eine Schreibsperre? Weil ich ein Paradoxon beschreibe? Kritikimmunisierung ist ja weit verbreitet, in der wissenschaftlichen Betrachtung aber eher fehl am Platz. Obwohl mir mathematisches Wissen fehlt bin ich doch sehr Realitätssicher und Realität hat Vorrang vor Mathematik, da nicht sicher ist ob Quantenobjekte diese Mathematik in der Realität auch anwenden.

Ich spreche bei meinen Betrachtungen immer von einem ruhenden Raumschiff gegenüber der Erde, bei dem sich beide Parteien darüber klar werden möchten welche Uhr gegenüber der anderen nachgeht.
Solange sie sich durch Kommunikation in einem ruhenden System darüber einig werden können kann die Beobachtung durch ein Fernrohr im gleichen ruhenden System nun mal nicht ergeben, dass beide Uhren nachgehen.

Semmelweis
04.08.15, 12:45
Hallo Semmelweis,
da sehe ich für dich nur zwei Möglichkeiten:

Entweder suchst du den Fehler bei Einstein und den unzähligen Wissenschaftlern nach ihm in Verbindung mit Ignoranz aller empirischen Bestätigungen,
oder du suchst den Fehler bei dir.:(

Mein Tipp: gehe etwas unvoreingenommener heran; ein Verständnis stellt sich erst nach und nach ein und nutze vor allem die Kompetenz einiger Mitglieder hier, die gibt es nicht in allen Foren.


mfg okotombrok

Mein Gedankenexperiment ist ja ein Versuchsaufbau der Experimentalphysik, den man wirklich so gestalten könnte.

Ohne jetzt überheblich wirken zu wollen habe ich auch schon die Quantenphänomene einfach und schlüssig, ja fast schon selbstverständlich interpretiert, weil Philosophen nun mal eine andere Herangehensweise haben als Physiker.

Ich
04.08.15, 13:25
Eine Schreibsperre? Weil ich ein Paradoxon beschreibe?
Weil du nach wiederholter Ermahnung im allgemeinen Forenbereich eine bestehende Theorie kritisierst. Die Forenregeln sind in diesem Punkt eindeutig, und ich werde sicher nicht noch einmal ein Auge zudrücken.

Diese Regel hat einen guten und einfachen Grund: Dieses Forum soll interessierten Personen die Möglichkeit bieten, sich über Physik zu unterhalten und ihre Kenntnisse zu verbessern. Es soll halbwegs verlässliche Informationen bereitstellen und nicht von Cranks zugespamt werden, die kritikresistent ihren immer gleichen Blödsinn loswerden wollen.

Ich erkläre es dir zum letzten Mal:

Du kannst im Unterforum "Theorien jenseits der Standardphysik" deine "Paradoxa" diskutieren. Auch dort bleibst du auf der Sachebene und gehst auf die Argumente anderer angemessen ein.
Oder du kannst hier lernen, wie die SRT die von dir geschilderte Situation behandelt. Wenn du etwas nicht verstehst, hast du die Holschuld, mithilfe gezielter Fragen zu einem Verständnis zu kommen. Niemand hier hat die Bringschuld, dir etwas zu erklären oder gar die Theorie zu verteidigen.

EDIT:
Am konkreten Beispiel bedeutet das: Ich muss dir nicht beweisen, dass da kein Paradoxon besteht. Man hat dir Antworten gegeben. Wenn du sie nicht verstanden hast, versuche die Dinge durch genaue Nachfrage zu klären. Vielleicht verstehst du es dann, vielleicht nicht. Wenn nicht, schade für dich, aber nicht unser Problem.

Semmelweis
04.08.15, 14:35
Weil du nach wiederholter Ermahnung im allgemeinen Forenbereich eine bestehende Theorie kritisierst. Die Forenregeln sind in diesem Punkt eindeutig, und ich werde sicher nicht noch einmal ein Auge zudrücken.

Diese Regel hat einen guten und einfachen Grund: Dieses Forum soll interessierten Personen die Möglichkeit bieten, sich über Physik zu unterhalten und ihre Kenntnisse zu verbessern. Es soll halbwegs verlässliche Informationen bereitstellen und nicht von Cranks zugespamt werden, die kritikresistent ihren immer gleichen Blödsinn loswerden wollen.

Ich erkläre es dir zum letzten Mal:

Du kannst im Unterforum "Theorien jenseits der Standardphysik" deine "Paradoxa" diskutieren. Auch dort bleibst du auf der Sachebene und gehst auf die Argumente anderer angemessen ein.
Oder du kannst hier lernen, wie die SRT die von dir geschilderte Situation behandelt. Wenn du etwas nicht verstehst, hast du die Holschuld, mithilfe gezielter Fragen zu einem Verständnis zu kommen. Niemand hier hat die Bringschuld, dir etwas zu erklären oder gar die Theorie zu verteidigen.

EDIT:
Am konkreten Beispiel bedeutet das: Ich muss dir nicht beweisen, dass da kein Paradoxon besteht. Man hat dir Antworten gegeben. Wenn du sie nicht verstanden hast, versuche die Dinge durch genaue Nachfrage zu klären. Vielleicht verstehst du es dann, vielleicht nicht. Wenn nicht, schade für dich, aber nicht unser Problem.

Es gibt Formelcranks, die die Realität nicht besonders ernst nehmen und Realitätscranks, die mathematische Formeln nicht besonders ernst nehmen.

Da ich ein Realitätscrank bin, ist das für mich wohl das falsche Forum.

Harti
04.08.15, 14:52
Hallo TomS,


Es bedeutet auch, dass die Erklärungen, die auf Lorentz-Transformationen und Inertialsystemen basieren, den Kern der Argumentation verfehlen, da man besser mit (invarianten) Eigenzeiten argumentiert.


Ich versuche mal in einem Raumzeitmodell zu argumentieren:

Die Abreise des Reisezwillings stellt ein Ereignis1 in der Raumzeit dar, und die Rückkehr zum Erdzwilling stellt ein Ereignis2 dar. Die Distanz zwischen Ereignis 1 und Ereignis 2 stellt ein invariantes Raumzeitintervall dar. In der Raumzeit kann man Raum und Zeit nicht getrennt betrachten, sie sind zu einer Einheit verschmolzen. Das Raumzeitintervall repräsentiert ein tatsächliches Geschehen und entspricht damit dem biologischen Alterungsprozess beider Zwillinge.
Ganz allgemein sind in der Raumzeit tatsächliches Geschehen (Raumzeitintervalle) invariant, wie es auch die elektromagnetische Wechselwirkung ist (Konstanz der Lichtgeschwindigkeit).
Raum und Zeit und die mit ihnen gebildeten Koordinatensysteme sind dem- gegenüber relativ. Dies führt bei getrennter Betrachtung von Raum und Zeit zu den bekannten relativistischen Effekten, die allerdings nur die Maßstäbe und nicht tatsächliches (biologisches) Geschehen betreffen.

Man kann auch noch anders argumentieren:
Der Reisezwilling führt durch Rückkehr zum selben Ort in der Raumzeit den ersten Takt einer periodischen Bewegung aus. Er verändert sich räumlich im Ergebnis nicht und für ihn vergeht genauso viel Zeit wie für den Erdzwilling.

Auch bei periodischen Bewegungen, die Grundlage von Uhren sind, (Pendel, Unruh, Quarz, Atomschwingung) gehen wir davon aus, dass sie rein zeitliche Veränderungen darstellen. Welchen Weg Milliarden Schwingungen (periodische Bewegungen) eines Atoms in einer Sekunde darstellen, spielt keine Rolle. Eine solche Atomuhr wird als reines Zeitmessinstrument aufgefasst.

MfG
Harti

Semmelweis
04.08.15, 15:13
Hallo TomS,


Es bedeutet auch, dass die Erklärungen, die auf Lorentz-Transformationen und Inertialsystemen basieren, den Kern der Argumentation verfehlen, da man besser mit (invarianten) Eigenzeiten argumentiert.


Ich versuche mal in einem Raumzeitmodell zu argumentieren:

Die Abreise des Reisezwillings stellt ein Ereignis1 in der Raumzeit dar, und die Rückkehr zum Erdzwilling stellt ein Ereignis2 dar. Die Distanz zwischen Ereignis 1 und Ereignis 2 stellt ein invariantes Raumzeitintervall dar. In der Raumzeit kann man Raum und Zeit nicht getrennt betrachten, sie sind zu einer Einheit verschmolzen. Das Raumzeitintervall repräsentiert ein tatsächliches Geschehen und entspricht damit dem biologischen Alterungsprozess beider Zwillinge.
Ganz allgemein sind in der Raumzeit tatsächliches Geschehen (Raumzeitintervalle) invariant, wie es auch die elektromagnetische Wechselwirkung ist (Konstanz der Lichtgeschwindigkeit).
Raum und Zeit und die mit ihnen gebildeten Koordinatensysteme sind dem- gegenüber relativ. Dies führt bei getrennter Betrachtung von Raum und Zeit zu den bekannten relativistischen Effekten, die allerdings nur die Maßstäbe und nicht tatsächliches (biologisches) Geschehen betreffen.

Man kann auch noch anders argumentieren:
Der Reisezwilling führt durch Rückkehr zum selben Ort in der Raumzeit den ersten Takt einer periodischen Bewegung aus. Er verändert sich räumlich im Ergebnis nicht und für ihn vergeht genauso viel Zeit wie für den Erdzwilling.

Auch bei periodischen Bewegungen, die Grundlage von Uhren sind, (Pendel, Unruh, Quarz, Atomschwingung) gehen wir davon aus, dass sie rein zeitliche Veränderungen darstellen. Welchen Weg Milliarden Schwingungen (periodische Bewegungen) eines Atoms in einer Sekunde darstellen, spielt keine Rolle. Eine solche Atomuhr wird als reines Zeitmessinstrument aufgefasst.

MfG
Harti

Ich vermute mal, dass sich die Realitätsprobleme dadurch ergeben, dass man Zeit als wirkliche Dimension betrachtet und nicht nur als biologische Taktveränderung, die man rein als Zahl darstellen kann.

Ich
04.08.15, 15:29
Ich vermute mal, dass sich die Realitätsprobleme dadurch ergeben, dass man Zeit als wirkliche Dimension betrachtet und nicht nur als biologische Taktveränderung, die man rein als Zahl darstellen kann.Und wieder...

Wie du richtig erkannt hast, ist das für dich das falsche Forum. Da sich dir die Konsequenz daraus offensichtlich nicht erschließt, gebe ich Hilfestellung.

-Ich-

TomS
04.08.15, 15:38
@Harti: ich glaube nicht, dass du das richtig verstanden hast

Ich
04.08.15, 16:11
Die Abreise des Reisezwillings stellt ein Ereignis1 in der Raumzeit dar, und die Rückkehr zum Erdzwilling stellt ein Ereignis2 dar. Die Distanz zwischen Ereignis 1 und Ereignis 2 stellt ein invariantes Raumzeitintervall dar.
Mit diesen Distanzen ist es fast genauso wie mit Abständen in der herkömmlichen Geometrie. Du kannst von A nach B auf unterschiedlichsten Wegen gelangen.
Der eine geht direkt dort hin und hat die Strecke AB zu gehen.
Der andere macht einen Umweg über C und hat die Strecke AC + CB zu gehen.
Wie weit sie gehen müssen ist unabhängig vom Bezugssystem. Einer hat weiter zu gehen als der andere, egal wie man es betrachet oder ausrechnet.

In der Raumzeit, bezogen auf das Zwillingsparadox, bedeutet die Strecke AB die Eigenzeit, die vergeht, wenn du auf direktem Wege von Ereignis A (Abreise) zu Ereignis B (Ankunft) gehst.
AC+CB ist der Reisezwilling, der zwischendurch das Ereignis C (Umkehrpunkt) besucht.
In der Geometrie der Raumzeit gilt aber: AB > AC + CB! Das heißt, für den Reisezwilling ist weniger Reisezeit vergangen.

Bauhof
04.08.15, 17:20
Mit diesen Distanzen ist es fast genauso wie mit Abständen in der herkömmlichen Geometrie. Du kannst von A nach B auf unterschiedlichsten Wegen gelangen.
Der eine geht direkt dort hin und hat die Strecke AB zu gehen.
Der andere macht einen Umweg über C und hat die Strecke AC + CB zu gehen.
Wie weit sie gehen müssen ist unabhängig vom Bezugssystem. Einer hat weiter zu gehen als der andere, egal wie man es betrachet oder ausrechnet.

In der Raumzeit, bezogen auf das Zwillingsparadox, bedeutet die Strecke AB die Eigenzeit, die vergeht, wenn du auf direktem Wege von Ereignis A (Abreise) zu Ereignis B (Ankunft) gehst.
AC+CB ist der Reisezwilling, der zwischendurch das Ereignis C (Umkehrpunkt) besucht.
In der Geometrie der Raumzeit gilt aber: AB > AC + CB! Das heißt, für den Reisezwilling ist weniger Reisezeit vergangen.

Hallo ICH,

besser kann man es nicht erklären.
Nur für Harti (den ich vom Zeitforum kenne) will ich noch ein paar redundante Bemerkungen machen, denn ich befürchte, er hat weitere Einwände:

Die geometrischen Strecken AB, AC und CB entsprechen den durchlebten Eigenzeiten der Zwillinge. Deshalb ist für den reisenden Zwilling weniger Zeit vergangen als für den ruhenden Zwilling. Warum? Weil die Reisezeit (AC + CB) kleiner ist als "Ruhezeit" des daheimgebliebenen Zwillings.

In der herkömmlichen Geometrie gilt (AC + CB) > AB. In der Raumzeit-Geometrie gilt wegen dem nichteuklidischen Charakter dieser Raumzeit: (AC + CB) < AB.

M.f.G. Eugen Bauhof

Marco Polo
04.08.15, 19:07
Hallo ICH,

besser kann man es nicht erklären.
Nur für Harti (den ich vom Zeitforum kenne) will ich noch ein paar redundante Bemerkungen machen, denn ich befürchte, er hat weitere Einwände:

Die geometrischen Strecken AB, AC und CB entsprechen den durchlebten Eigenzeiten der Zwillinge. Deshalb ist für den reisenden Zwilling weniger Zeit vergangen als für den ruhenden Zwilling. Warum? Weil die Reisezeit (AC + CB) kleiner ist als "Ruhezeit" des daheimgebliebenen Zwillings.

In der herkömmlichen Geometrie gilt (AC + CB) > AB. In der Raumzeit-Geometrie gilt wegen dem nichteuklidischen Charakter dieser Raumzeit: (AC + CB) < AB.

M.f.G. Eugen Bauhof

Seh ich auch so, Eugen.

TomS
04.08.15, 20:15
Ich halte die Notation "AC + CB < AC" für nicht sehr hilfreich.

Marco Polo
04.08.15, 20:27
Mag sein. Aber falsch ist es nicht.

Harti
04.08.15, 22:54
Hallo Ich,

In der Raumzeit, bezogen auf das Zwillingsparadox, bedeutet die Strecke AB die Eigenzeit, die vergeht, wenn du auf direktem Wege von Ereignis A (Abreise) zu Ereignis B (Ankunft) gehst.
AC+CB ist der Reisezwilling, der zwischendurch das Ereignis C (Umkehrpunkt) besucht.
In der Geometrie der Raumzeit gilt aber: AB > AC + CB! Das heißt, für den Reisezwilling ist weniger Reisezeit vergangen.

Nach meiner Ansicht kann man die Veränderungen der Zwillinge in der Raumzeit mit Hilfe von Vektoren darstellen, weil Vektoren Raum und Zeit repräsentieren.

Der Erdzwilling verändert sich nur auf der Zeitachse von A nach B, VektorAB. Der Reisezwilling verändert sich im Verhältnis zur Raum- und zur Zeitachse von A nach C, Vektor AC, wendet dort und verändert sich von C nach B, Vektor CB.
AB = AC + CB ergibt die Vektorrechnung.
Dies bedeutet raumzeitlich besteht zwischen den Veränderungen von Erdzwilling und Reisezwilling kein Unterschied.
Ich benutze in der Raumzeitbetrachtung bewußt das Wort "Veränderung", weil der Begriff "Bewegung" allgemeinsprachlich als Gegensatz zu Ruhe rein räumlich definiert ist.

MfG
Harti

Ich
05.08.15, 07:34
AB = AC + CB ergibt die Vektorrechnung.Richtig.
Das Problem ist, dass ich hier keine Striche drübermachen kann. Mein AB > AC + CB bezog sich (das habe ich auch so geschrieben) auf die jeweiligen Strecken, also die Längen der Vektoren.
Die Vektorgleichung AB = AC + CB sagt nichts weiter, als dass beide sich wieder am Ereignis B treffen. Das ist nicht sonderlich aufregend. Das Entscheidende ist, dass sie unterschiedliche Strecken (sprich: Eigenzeiten) zurücklegen mussten, um dorthin zu kommen.

Harti
05.08.15, 08:54
Hallo ich,
Richtig.
Das Problem ist, dass ich hier keine Striche drübermachen kann. Mein AB > AC + CB bezog sich (das habe ich auch so geschrieben) auf die jeweiligen Strecken, also die Längen der Vektoren

Auch ich konnte keine kleinen Pfeile über die Buchstaben machen.
Der Betrag der Vektoren repräsentiert nach meiner Meinung sowohl den Raum wie die Zeit, eben durch den jeweiligen Bezug auf die Achsen. Auf der Raumachse ist die räumliche Veränderung des Reisezwillings 0, weil er sich vor und zurück bewegt. Er ist am Ende der Reise wieder am selben Raumpunkt. Übrig bleibt für ihn, wie für den Erdzwilling, allein die zeitliche Veränderung.
In einem Raumzeitmodell sind nach meiner Meinung relativistische Effekte nicht darstellbar. Es gibt auch keine Geschwindigkeiten im herkömmlichen Sinn, weil Raum und Zeit nicht trennbar sind.
Es bleibt daher nur die Frage, ob die Raumzeitintervalle des Erdzwillings und des Reisezwillings gleich oder verschieden sind. Ich bin der Meinung, dass sie gleich sind und belege das mit der Vektorrechnung

MfG
Harti

Ich
05.08.15, 09:10
In einem Raumzeitmodell sind nach meiner Meinung relativistische Effekte nicht darstellbar. Es gibt auch keine Geschwindigkeiten im herkömmlichen Sinn, weil Raum und Zeit nicht trennbar sind.
Es bleibt daher nur die Frage, ob die Raumzeitintervalle des Erdzwillings und des Reisezwillings gleich oder verschieden sind. Ich bin der Meinung, dass sie gleich sind und belege das mit der Vektorrechnung
Das geht langsam auch aus dem Rahmen des allgemeinen Forums heraus.
Ich führe mal als Notation ein: s(AB) sei das Intervall zwischen den Ereignissen A und B. Das ist die "Länge" (genau: Norm) des Vektors AB.

(1) Dann gilt für zeitartige, zukunftsgerichtete Vektoren die Dreiecksungleichung s(AB) >= s(AC) +s(CB).

(2) Das Intervall s(AB) entspricht der verstrichenen Eigenzeit eines Beobachters, der sich unbeschleunigt von Ereignis A nach Ereignis B bewegt. Ebenso die anderen Intervalle.

Irgendein Problem mit (1) oder (2)?

Harti
06.08.15, 12:29
Hallo Ich,
ich muss mich erst mal aus der Diskussion verabschieden, weil ich 2 Wochen ortsabwesend bin.
MfG
Harti

Ich
06.08.15, 13:18
Dann viel Spass, falls es in den Urlaub geht.

Harti
09.09.15, 09:18
Hallo Ich,

ich will meine Ansicht noch einmal auf der Grundlage eines Raumzeitmodells begründen:

In einem Raumzeitmodell gibt es nur eine Zeitachse. Damit ist in diesem Modell nur eine Uhr vorstellbar. Dies wird in manchen Darstellungen bildlich als engmaschiges Gitternetz im Raum mit gleichgehenden Uhren an den Knotenpunkten dargestellt (siehe "Physik der Raumzeit" , Taylor/Wheeler, 1994 Spektrum Akademischer Verlag, Seite 63 ff). Es gibt in einem solchen Raumzeitmodell keine relativistischen Effekte.

Zwillinge mit zwei Uhren sind in diesem Modell nicht darstellbar und damit auch keine Koordinatensysteme, die im Verhältnis zueinender gedreht sind.
Der Reisezwilling bewegt sich mit der sogenannten, gleichbleibenden Vierergeschwindigkeit. Bei Annahme einer Gleichwertigkeit von Raum und Zeit hat diese Geschwindigkeit den einheitenfreien Wert 1, auf der Grundlage der herkömmlichen Einheiten (Meter, Sekunde) entspricht sie der Lichtgeschwindigkeit.

Der Alterungsprozess beider Zwillinge entspricht der Dauer der Reise des Reisezwillings. Je weiter der Reisezwilling reist, um so älter sind beide Zwillinge bei seiner Rückkehr.

Man kann diese Darstellung auch anders raumzeitlich beschreiben:
Die raumzeitliche Distanz zwischen Abreise (Ereignis1) und Rückkehr (Ereignis2) stellt ein Raumzeitintervall dar. Die Berechnung dieses Raumzeitintervalls lautet:
Raumzeitintervall = sqrt (Zeitentfernung x Zeitentfernung - Raumentfernung x Raumentfernung)
Da die Zwillinge sich bei beiden Ereignissen am selben Raumpunkt befinden, ist die Raumentfernung 0. Als Ergebnis bleibt deshalb die einfache Zeitentfernung als Raumzeitintervall übrig.

Dieses Ergebnis entspricht auch in dem Sinne dem Raumzeitmodell, dass Raumzeitintervalle invariantes, tatsächliches Geschehen repräsentieren. Dies ist im Zwillingsparadoxon der tatsächliche, identische Alterungsprozess beider Zwillinge.

Meine Lösung des Zwillingsparadoxons auf der Grundlage eines Raumzeitmodells:

Beide Zwillinge sind beim Wiedersehen gleichalt.

MfG
Harti

JoAx
09.09.15, 16:04
In einem Raumzeitmodell gibt es nur eine Zeitachse.
...


Das ist nicht korrekt.


Zwillinge mit zwei Uhren sind in diesem Modell nicht darstellbar und damit auch keine Koordinatensysteme, die im Verhältnis zueinender gedreht sind.


Das ist nicht korrekt.


Der Reisezwilling bewegt sich mit der sogenannten, gleichbleibenden Vierergeschwindigkeit.


Das ist nicht korrekt. Alles (nicht nur der "Reisezwilling") "bewegt" sich durch die Raumzeit mit der gleichbleibenden Vierergeschwindigkeit.


Da die Zwillinge sich bei beiden Ereignissen am selben Raumpunkt befinden, ist die Raumentfernung 0. Als Ergebnis bleibt deshalb die einfache Zeitentfernung als Raumzeitintervall übrig.


Und warum sind die Zwillinge dann unterschiedlich alt? Wo kommt das her?


Beide Zwillinge sind beim Wiedersehen gleichalt.


Nö.

TomS
09.09.15, 16:28
Meine Lösung des Zwillingsparadoxons auf der Grundlage eines Raumzeitmodells: Beide Zwillinge sind beim Wiedersehen gleichalt.
Bevor wir jetzt theoretisch diskutieren, an welchen Stellen deine Argumentation falsch ist (es sind mehrere Stellen), können wir doch mal rein praktisch feststellen, dass sie falsch ist, weil es Experimente gibt, die deine Schlussfolgerung explizit widerlegen. D.h. deine sogenannte "Lösung" ist keine Lösung.

Ansonsten gebe ich die Hoffnung nicht auf und verweise gerne auf meine Darstellung hier:

http://www.physikerboard.de/topic,37752,-faq---zeitdilatation-und-zwillingsparadoxon.html

Harti
10.09.15, 07:49
Hallo JoAx und TomS,
ich mache eigentlich nichts anderes, als die vierdimensionale (raumzeitliche) Betrachtung konsequent auf das Zwillingsparadoxon anzuwenden.

Ich versuche mal, den Unterschied in unserer Betrachtung an einem reduzierten, rein räumlichen Beispiel zu erklären.

Ich stelle mir zwei parallel liegende Stäbe A und B vor. In einem von den Stäben gebildeten eindimensionalen Bezugssystem sind sie gleichlang. Ich drehe nun Stab A. In dem ursprünglichen eindimensionalen Bezugssystem erscheint A dann (perspektivisch) verkürzt, in einem höher dimensionierten, zweidimensionalen (flächigen) Bezugssystem bleiben beide Stäbe gleichlang.

Beim Übergang von einer herkömmlichen, getrennten Betrachtung von Raum (dreidimensional) und Zeit (eindimensional) zu einer vierdimsionalen (raumzeitlichen) Betrachtung sind die Verhältnisse zwar komplizierter, weil Raum und Zeit begrifflich nicht ohne weiteres kompatibel sind, aber im Prinzip genauso. Die relativistischen Effekte sind in einem Raumzeitmodell nicht darstellbar.


Das ist nicht korrekt. Alles (nicht nur der "Reisezwilling") "bewegt" sich durch die Raumzeit mit der gleichbleibenden Vierergeschwindigkeit.


Genau. Und wie sollen dann relativistische Effekte entstehen ?

Zu experimentellen Nachweisen von relativistischen Effekten:
Wenn ein Beobachter seine Uhr als Bezugssystem verwendet, verbleibt er in dem oben von mir dargestellten reduzierten Beispiel in seinem eindimensionalen Bezugssystem und stellt auf einer im Verhältnis zu ihm bewegten Uhr eine Zeitdilatation fest.

Mich würde interessieren, ob es experimentelle Nachweise dafür gibt, dass tatsächliches (raumzeitliches) Geschehen, z.B. Wachstum oder Alterungsprozesse relativistischen Effekten unterliegen.
Bei dem bewegten Objekt dürfte es sich nicht um eine Uhr oder eine periodische Bewegung handeln, die ihrem Wesen nach nichts anderes als eine Uhr ist.

MfG
Harti

TomS
10.09.15, 08:20
ich mache eigentlich nichts anderes, als die vierdimensionale (raumzeitliche) Betrachtung konsequent auf das Zwillingsparadoxon anzuwenden.
Ich denke, du verstehst die Mathematik nicht; deswegen kannst du das gar nicht konsequent anwenden.

Die relativistischen Effekte sind in einem Raumzeitmodell nicht darstellbar.
Das ist doch Quatsch. In einem vernünftigen Modell sind sie offensichtlich darstellbar (SRT, ART).

Und wie sollen dann relativistische Effekte entstehen ?
Lies, was ich unter dem o.g. Link gesachrieben habe, und frag präzise nach bzw. identifiziere Fehler in der Mathematik.

Zu experimentellen Nachweisen von relativistischen Effekten: Wenn ein Beobachter seine Uhr als Bezugssystem verwendet, ...
Das ist mathematisch i.A unmöglich!!

Es liegen zwei Beobachter B und B' vor, d.h. beide müssten jeweils ihre eigene Uhr U bzw. U' verwenden, um in ihrem Ruhesystem eine Zeitkoordinate t bzw. t' identisch mit ihrer jeweiligen Eigenzeit T bzw T' (die sie jeweils auf ihrer Uhr ablesen) zu definieren. Außerdem muss zur Berechnung und zum Vergleich eine Umrechung bzw. eine Beziehung zwischen beiden Bezugssystemen und Zeiten möglich sein.

Da mindestens ein Beobachter beschleunigt ist, definiert er kein Inertialsystem. Damit kannst du die Mathematik der SRT an dieser Stelle nicht anwenden (da die SRT Berzugssysteme immer mit Inertialsystemen gleichsetzt). Und damit hast du schlichtweg keine Argumentationsgrundlage, um überhaupt etwas über die Beziehung zwischen beiden Zeiten aussagen zu können.

Mich würde interessieren, ob es experimentelle Nachweise dafür gibt, dass tatsächliches (raumzeitliches) Geschehen, z.B. Wachstum oder Alterungsprozesse relativistischen Effekten unterliegen.
Bei dem bewegten Objekt dürfte es sich nicht um eine Uhr oder eine periodische Bewegung handeln, die ihrem Wesen nach nichts anderes als eine Uhr ist.
Es existiert kein Unterschied zwischen "Alterungsprozessen" und "periodische Bewegungen". Wichtig, ist, dass man ein Zeitnormal festsetzen kann. Ob das jetzt eine Atomuhr ist oder das Wachstum von Riesensequoias ist zunächst irrelevant. Rein praktisch muss man natürlich ein Zeitnormal benutzen, das die erforderliche Präzision liefert, d.h. die Riesensequoias scheiden aus; man nutzt besser die Atomuhr (und diese liefert ein "tatsächliches raumzeitliches Geschehen").

Bzgl. experimenteller Nachweise fallen mir sofort Messungen mittels Atomuhren in Flugzeugen und Satelliten ein, sowie Zerfallszeiten von instabilen Teilchen (an Beschleunigern sowie aus der kosmischen Strahlung)

...

Generell noch eine Anmerkung zu deiner Argumentation: du versuchst dich sozusagen an einem "no-go theorem", d.h. du versuchst zu argumentieren, dass die Zeitdilatation = der Alterungsunterschied der beiden Zwillinge prinzipiell nicht existieren kann. Jede Argumentation basiert auf Annahmen, Axiomen o.ä.; diese must du präzise formulieren (das hast du bisher nicht getan). Sobald du das getan hast, kannst du das "no-go-theorem" evtl. unter der Voraussetzung dieser Annahmen beweisen. Damit ist allerdings nichts gesagt über eine andere Argumentation auf Basis anderer Annahmen oder Axiomen. D.h. selbst wenn deine Axiome und deine Argumente logisch korrekt und präzise wären, würde sie ggf. nichts über die Natur aussagenM; d.h. nicht deine Argumente, sondern deine Annahmen wären unzutreffend. Ich muss also gar nicht deine Argumente im einzelnen nachvollziehen und widerlegen, es genügt, dass sie zu experimentell widerlegten Kosnequenzen führen. Damit ist dein gesamtes Gedankengebäude falsch, ohne dass ich dir genau nachweisen müsste, wo genau es falsch ist; es reicht aus, dass es zu einer Schlussfolgerung führt, die experimentell widerlegt ist.


Immer noch gebe ich die Hoffnung nicht auf und verweise auf meine Darstellung hier:

http://www.physikerboard.de/topic,37752,-faq---zeitdilatation-und-zwillingsparadoxon.html

Versuche sie zu verstehen, frage nach, weise auf Fehler hin, ...

JoAx
10.09.15, 14:39
Genau. Und wie sollen dann relativistische Effekte entstehen ?


Wenn du dem Rat von TomS folgst, dann begreifst du zu aller erst, was die relativistischen Effekte sind (das scheinst du nicht zu wissen) und, wie/dass sie genau so zustande kommen.

Um da schneller voran zu kommen, empfiehlt sich, den Kopf von den Vorurteilen frei zu machen. (Vor allem von dem Vorurteil, dass du bei diesem Thema korrekte Einschätzungen machst.)

Harti
10.09.15, 17:19
Hallo TomS,

Ansonsten gebe ich die Hoffnung nicht auf und verweise gerne auf meine Darstellung hier:

http://www.physikerboard.de/topic,37752,-faq---zeitdilatation-und-zwillingsparadoxon.html

leider bin ich als pensionierter Richter dem Kommunikationsmittel Mathematik nicht hinreichend mächtig, um Deine Darstellung zu verstehen.

Zwei Sätze sind mir allerdings aufgefallen:

"Zuerst betrachtet man den Spezialfall, dass der erste Beobachter (Zwilling) in einem Inertialsystem ruht. Dann kann man dessen Ruhesystem als Bezugssystem verwenden, d.h. die Koordinatenzeit t sowie die Eigenzeit tau identifizieren."

Auch ich komme mit meinen Überlegungen zu dem Ergebnis, dass die Uhr des Reisezwillings langsamer gegangen ist, wenn ich den Erdzwilling auszeichne, indem ich ihn mit seiner Uhr zum Bezugssystem mache und damit das Relativitätsprinzip außer Kraft setze.

Für mich ist die Beschäftigung mit den Relativitätstheorien ein Bemühen um Verstehen, bin dabei aber wie gesagt in meinen mathematischen Fähigkeiten beschränkt.

Vielleicht können wir es ja dabei belassen.

mfG
Harti

Plankton
10.09.15, 19:22
Finde die Diskussion hier verwirrend :confused:. Vielleicht kann ich helfen!

Für mich ist die Beschäftigung mit den Relativitätstheorien ein Bemühen um Verstehen, bin dabei aber wie gesagt in meinen mathematischen Fähigkeiten beschränkt.

Schau dir doch ein paar "praktische Beispiele" an:
https://de.wikipedia.org/wiki/Doppler-Effekt#Doppler-Effekt_ohne_Medium

TomS
10.09.15, 20:46
Zwei Sätze sind mir allerdings aufgefallen:

"Zuerst betrachtet man den Spezialfall, dass der erste Beobachter (Zwilling) in einem Inertialsystem ruht. Dann kann man dessen Ruhesystem als Bezugssystem verwenden, d.h. die Koordinatenzeit t sowie die Eigenzeit tau identifizieren."

.. wenn ich den Erdzwilling auszeichne, indem ich ihn mit seiner Uhr zum Bezugssystem mache und damit das Relativitätsprinzip außer Kraft setze.
Ich zeichne den Erdzwilling nicht aus, u d ich setze insbs. das Relativitätsprinzip nicht außer Kraft. Das Ergebnis der Rechnung hängt auch nicht davon ab, welches Bezugssystem ich benutze. Die Differenz zweier Eigenzeiten ist eine Invariante, d.h. sie gilt unabhängig von der speziellen Wahl eines Koordinatensystems.

Alles was ich mache ist, eine geschickte Wahl zu treffen um einfacher rechnen zu können.

Bsp.: die Strecke von München nach Hamburg ist immer gleich lang, unabhängig davon welche Landkarte ich benutze; wähle ich dir Karte geschickt, kann ich die Strecke sehr einfach nachmessen; wähle ich die Karte ungeschickt, muss ich kompliziert rechnen; aber die reale Strecke ist immer gleich lang.

Marco Polo
11.09.15, 01:08
Es liegen zwei Beobachter B und B' vor, d.h. beide müssten jeweils ihre eigene Uhr U bzw. U' verwenden, um in ihrem Ruhesystem eine Zeitkoordinate t bzw. t' identisch mit ihrer jeweiligen Eigenzeit T bzw T' (die sie jeweils auf ihrer Uhr ablesen) zu definieren. Außerdem muss zur Berechnung und zum Vergleich eine Umrechung bzw. eine Beziehung zwischen beiden Bezugssystemen und Zeiten möglich sein.

Da mindestens ein Beobachter beschleunigt ist, definiert er kein Inertialsystem. Damit kannst du die Mathematik der SRT an dieser Stelle nicht anwenden (da die SRT Berzugssysteme immer mit Inertialsystemen gleichsetzt). Und damit hast du schlichtweg keine Argumentationsgrundlage, um überhaupt etwas über die Beziehung zwischen beiden Zeiten aussagen zu können.

Wahrscheinlich verstehe ich dich falsch. Aber meines Wissens kann ich auch dann mit der SRT rechnen, wenn der Reisezwiling beschleunigt.

Nachstehende Formel drückt die Eigenzeit tau als Funktion der beim Ruhezwilling verstrichenen Zeit t aus.

tau=(c/alpha)*ln((alpha*t/c)+sqrt(1+(alpha*t/c)²))oder andersrum

t=(c/alpha)*sinh(alpha*tau/c)tau=Eigenzeit des Reisezwillings
t=die verstrichene Zeit des Erdzwillings
alpha=Eigenbeschleunigung des Reisezwillings

TomS
11.09.15, 06:16
Wahrscheinlich verstehe ich dich falsch. Aber meines Wissens kann ich auch dann mit der SRT rechnen, wenn der Reisezwiling beschleunigt.
Das habe ich auch nie bestritten; ich denke, ich habe hier und in dem von mir verlinkten Beitrag sogar explizit darauf hingewiesen, dass die SRT durchaus mit Beschleunigungen umgehen kann.

Es ging um den Satz "Wenn ein Beobachter seine Uhr als Bezugssystem verwendet ...". Dies ist im Kontext des Zwillingsparadoxons i.A. nicht möglich, da die Eigenzeit des beschleunigten Zwillings (und mindestens einer ist immer beschleunigt) keine Koordinatenzeit in einem Inertialsystem definiert. Die Formulierung der SRT basiert jedoch auf Inertialsytemen, d.h. unbeschleunigten Bezugssystemen.

Man muss also
1) entweder einen Zwilling als ruhend voraussetzen, was eine unnötige Einschränkung bedeutet,
2) oder ein Inertialsystem einführen, innerhalb dessen beide Zwillinge gleichberechtigt beschrieben werden können

(1) ist der Standardweg, hat jedoch den großen Nachteil, dass es so aussieht, als ob es gerade die Tatsache wäre, dass ein Zwilling in einem Inertialsystem ruht während der andere das Inertialsystem wechselt, die zur Zeitdilatation führt. Das ist irreführend. Diese Inertialsysteme sind für die Zeitdilatation irrelevant! Ausschließlich relevant ist eine bezugssystemunabhängige, invariante Größe, die Differenz der Eigenzeiten.

Deswegen wähle ich (2) und führe ein gedachtes Inertialsystem ein, in dem beide Zwillinge beliebig bewegt und auch beschleunigt sein können. D.h. ich benutze keine der beiden Eigenzeiten als Koordinatenzeiten. Darüberhinaus sieht man sofort, dass auch für völlig beliebige Bewegungen die Beschleunigung (bzgl. eines Inertialsystems) nie relevant für die Berechnung der Zeitdilatation ist.

Zusammenfassend: die SRT kann mit Beschleunigungen umgehen; Eigenzeiten von beschleunigten Beobachtern entsprechen keinen Koordinatenzeiten von Inertialsystemen

Hawkwind
11.09.15, 09:55
Deswegen wähle ich (2) und führe ein gedachtes Inertialsystem ein, in dem beide Zwillinge beliebig bewegt und auch beschleunigt sein können.

Wenn ich noch ergänzen darf: man kann auch nicht-inertiale Koordinatensysteme in der SRT einführen; ein bekanntes Bsp sind die Rindler-Koordinaten (gleichmäßig beschleunigtes Bezugssystem).

Das ist aber i.d.R. alles andere als "easy". :)

TomS
11.09.15, 10:18
Ich habe mir bisher verkniffen, das selbst anzusprechen ...

... ja, kann man tun, aber man hat damit sofort ein anderes Problem, nämlich dass dieses Koordinatensystem nicht mehr global gültig ist; anders gesagt, es deckt nur einen Teilbereich des gesamten Minkowskiraumes ab.

https://en.wikipedia.org/wiki/Rindler_coordinates

In unserem Beispiel des Zwillingsparadoxons "verschwindet" der auf der Erde zurückbleibende Zwilling B' hinter dem Rindler-Horizont des beschleunigten Zwillings B; letzterer hat also keine Möglichkeit, seine Eigenzeit T = seine Koordinatenzeit t zu benutzen, um sie mit der Eigenzeit T' des auf der Erde ruhenden Zwillings B' zu vergleichen; insbs. existieren für die Weltlinie von B' keine Koordinaten (t,x).

Um das Zwillingsparadoxon vernünftig zu diskutieren, benötigt man ein Koordinatensystem, das die Weltlinien beider Beobachter vollständig enthält. Das Rindler-Koordinatensystem leistet dies i.A. nicht.

Harti
11.09.15, 13:29
Hallo zusammen,
mir ist noch ein ganz anderer Gedanke gekommen.

Möglicherweise stehen das durch den Alterungsprozess bestimmte Alter der Zwillinge und die Bewegung der Zwillinge in Raum und Zeit garnicht in einem raumzeitlichen Zusammenhang.
Damit meine ich Folgendes: Der Alterungsprozess hat eine gewisse Dauer (Zeit), die dazu gehörige räumliche (geringfügige) Veränderung ist die körperliche Veränderung der Zwillinge (z.B. Körperwachstum, wenn sie bei der Abreise noch jung sind).
Dies liefe darauf hinaus, dass für das Alter der Zwillinge allein ihre jeweilige Eigenzeit maßgebend wäre und relativistische Effekte aus der Reise des Reiszwillings keine Rolle spielen würde.
Was meint ihr dazu ?
MfG
Harti

JoAx
11.09.15, 13:48
...
Was meint ihr dazu ?


Mit anderen Worten - der menschliche Alterungsprozess unterliegt keinen physikalischen Gesetzen.

Korrekt?

TomS
11.09.15, 14:03
Möglicherweise stehen das durch den Alterungsprozess bestimmte Alter der Zwillinge und die Bewegung der Zwillinge in Raum und Zeit gar nicht in einem raumzeitlichen Zusammenhang.
Zunächst ist das so: jeder altert für sich alleine vor sich hin, unabhängig vom anderen; maßgeblich für das Altern ist die die jeweilige Eigenzeit eines Zwillings, abgelesen auf seiner eigenen, mitgeführten Uhr; die Uhr bzw. Eigenzeit des anderen ist irrelevant.

Der Alterungsprozess hat eine gewisse Dauer (Zeit), die dazu gehörige räumliche (geringfügige) Veränderung ist die körperliche Veränderung der Zwillinge (z.B. Körperwachstum, wenn sie bei der Abreise noch jung sind).
Ja.

Besser: Der Alterungsprozess dauert eine gewisse Eigenzeit, abgelesen auf einer mitgeführten Uhr; die dazu gehörige Veränderung ist die körperliche Veränderung der Zwillinge (z.B. Körperwachstum, wenn sie bei der Abreise noch jung sind).

Dies liefe darauf hinaus, dass für das Alter der Zwillinge allein ihre jeweilige Eigenzeit maßgebend wäre ...
Ja, wenn dieses Alter in ihrer jeweiligen Eigenzeit gemessen wird.

Ein Zwilling, der 10 Jahre unterwegs ist - gemessen auf seiner mitbewegten Uhr - altert körperlich um zehn Jahre (der andere, der 20 Jahre unterwegs ist, altert um 20 Jahre)

Was meint ihr dazu ?
Soweit richtig.

Aber dennoch können beide Zwillinge nach der Reise unterschiedlich alt sein, d.h. wenn sie ihr Alter, d.h. ihre Eigenzeiten vergleichen, dann stellen sie Abweichungen fest.

******

Ganz einfaches Beispiel:

Ein Zwilling reist direkt von München nach Hamburg. Der andere reist von München zunächst nach Berlin und dann nach Hamburg. In Hamburg vergleichen beide den Kilometerstand ihrer Autos und stellen fest, dass sie unterschiedlich weit gefahren sind.

Soweit einverstanden?

Plankton
11.09.15, 15:43
...
Soweit einverstanden?
Ich bin zwar nicht gemeint, aber das möchte ich kurz sagen: wunderschöne Ausführungen! :cool:

Harti
11.09.15, 17:34
Hallo JoAx,
Mit anderen Worten - der menschliche Alterungsprozess unterliegt keinen physikalischen Gesetzen.

Korrekt?

Nein so war meine Äußerung nicht gemeint.
Man könnte eher sagen, die Reise bzw. das Daheimbleiben der Zwillinge und der Alterungsprozess sind verschiedene raumzeitliche Vorgänge, die in keinem kausalen Zusammenhang stehen.
Dies würde allerdings bedeuten, dass beide biologisch gleichmäßig altern, egal ob sie zuhause bleiben oder verreisen.:confused:

MfG
Harti

Harti
12.09.15, 08:52
Hallo TomS

Aber dennoch können beide Zwillinge nach der Reise unterschiedlich alt sein, d.h. wenn sie ihr Alter, d.h. ihre Eigenzeiten vergleichen, dann stellen sie Abweichungen fest.
Soweit einverstanden?

Grundsätzlich einverstanden; aber ist das Paradoxon, der Widerspruch damit wirklich aufgeklärt ?

Ich gehe davon aus, dass man ein paradoxes Ergebnis immer dann erhält, wenn in den den Annahmen ein Widerspruch versteckt ist.
Könnte es sein, dass man beim Zwillingsparadoxon zwei raumzeitliche Vorgänge, die in keinem Zusammenhng stehen, miteinander vermengt.
1) Biologischer Alterungsprozeß für beide Zwillinge gleich:
zeitlich: Dauer der Reise (Eigenzeit)
räumlich: 20 cm Wachstum ( Zwillinge befanden sich gerade in einem
pubertären Wachtumsschub)

2) Reise:
zeitlich: Reisedauer für beide Zwillinge aus ihrer Sicht jeweils verschieden;
aus Sicht des Erdzwillings und objektiv für Reisezwilling kürzer,
weil er sich auch räumlich verändert.

Die (unzulässige) Gleichsetzung von Alter nach Ziff. 1) und Reisedauer nach Ziff. 2) erzeugt dann den Widerspruch ???

MfG
Harti

TomS
12.09.15, 09:32
Grundsätzlich einverstanden; aber ist das Paradoxon, der Widerspruch damit wirklich aufgeklärt ?
Es gibt und gab nie ein Paradoxon. Es gibt zwei Zwillinge mit unterschiedlichen Weltlinien durch die Raumzeit, entlang derer jeweils unterschiedliche Eigenzeit vergeht.

Es gibt auch kein Paradoxon, dass unterschiedliche Reiserouten von München nach Hamburg unterschiedlich lang sind. Und die geometrischen Argumente sind tatsächlich fast identisch!

Das dumme ist, dass einem zunächst ein sogenanntes Paradoxon und anschließend eine teilweise unzureichende Auflösung präsentiert wird.

Marco Polo
12.09.15, 18:38
Das dumme ist, dass einem zunächst ein sogenanntes Paradoxon und anschließend eine teilweise unzureichende Auflösung präsentiert wird.

Genau. Beim Zwillingsparadoxon handelt es sich nämlich lediglich um eine Paradoxie (etwas dem Geglaubten, Gemeinten oder Erwarteten Zuwiderlaufendes).

Der Begriff "Paradoxon" (eine scheinbar zugleich wahre und falsche Aussage) ist in diesem konkreten Fall alles andere als angemessen und ist zum einen in Wissenslücken aber auch in didaktisch zweifelhafter Lehrbuchlektüre begründet.

Was beim einen Zwilling passiert, ist völlig unabhängig davon was beim anderen Zwilling passiert. Da gibt es eben keinen Zusammenhang oder Verbindung oder was auch immer.

Jeder durchlebt seine Eigenzeit. Nur mit den Lorentztrafos zu rechnen trifft den Kern der Sache nicht.

TomS
12.09.15, 23:14
Danke!

Dem gibt es nichts mehr hinzuzufügen.

Harti
13.09.15, 06:41
Hallo TomS,

Das dumme ist, dass einem zunächst ein sogenanntes Paradoxon und anschließend eine teilweise unzureichende Auflösung präsentiert wird.

Ist das nicht gerade typisch für Paradoxien. Zunächst erscheint etwas widersprüchlich und wenn man den Widerspruch erklärt hat, ist es keiner mehr und war eigentlich auch von Anfang an keiner.
Hat Einstein, von ihm stammt meines Wissens dieses "Paradoxon", sich damit einen Scherz oder eine Denksportaufgabe erlaubt :rolleyes:

MfG
Harti

Slash
13.09.15, 09:16
aber ist das Paradoxon, der Widerspruch damit wirklich aufgeklärt ?


Hallo Harti,

also ich schalte mich einmal ein - ein wenig weniger physikalisch korrekt vielleicht, eben meine "Sicht" (können andere gerne korrigieren).

Zunächst gingst du auf die Vierergeschwindigkeit ein, d.h. in der Raumzeit bewegen wir uns mit Lichtgeschwindigkeit (x,y,z, -c*t) ----> dieser Vektor hat den Betrag c. Genau hierauf beziehe ich mich.

Wir sind das Inertialsystem und fliegen im Weltraum gerade aus - keine Kräfte wirken auf uns.


Es gibt 3 Uhren: Eine die wir ablesen, eine des 1. Zwillings, eine des 2. Zwillings.

Das Experiment startet. Alle Uhren zeigen beim Start 0:00 an.

Der eine Zwilling entfernt sich langsam von uns (wie eine Schnecke), der andere rast davon.

Nach 5 Jahren (auf meiner Uhr) sind beide Zwillinge bei mir wieder angelangt.
Die Uhr des 1. Zwillings zeigt nach den 5 Jahren auf meiner Uhr an : 4.9994 Jahre
Die Uhr des 2. Zwillings zeigt nach den 5 Jahren auf meiner Uhr an: 0.5 Jahre

Die 4er Geschwindigkeit muss aber für uns 3 alle immer gleich gewesen sein.
Aufintegriert (Integral der Geschw. ---> Strecke) kann die Lösung nur sein, dass - weil wir unterschiedliche Strecken im Raum zurückgelegt haben (bezogen auf mich als Inertialsystem) - aufintegriert derjenige, der die größte Strecke im Raum zurückgelegt hat, die kleinste "Strecke" in der Zeit bzw. c*t zurückgelegt hat.

Der 1. Zwilling (die Schnecke) hat kaum beschleunigt und abgebremst, relativ zu mir gesehen, war seine Geschwindigkeit immer nahe 0.
Der 2. Zwilling hat unglaublich beschleunigt und abgebremst, deswegen auch hohe Geschwindigkeiten erreicht.

Ob es verständlich war, weiß ich nicht.

(Und ob richtig auch nicht ;-)

VG
Slash /


https://de.wikipedia.org/wiki/Vierervektor#Interpretation

(Anm.: Ggf. ist diese Interpretation auch nicht vollständig korrekt, aber ich finde anschaulich).

Harti
14.09.15, 11:25
Hallo Slash

Zunächst gingst du auf die Vierergeschwindigkeit ein, d.h. in der Raumzeit bewegen wir uns mit Lichtgeschwindigkeit (x,y,z, -c*t) ----> dieser Vektor hat den Betrag c. Genau hierauf beziehe ich mich.

Ich denke mit den Koordinaten gibst Du einen Punkt, genannt Ereignis, in der vierdimensionalen Raumzeit an. -ct ist nichts anderes als die Umrechnung/Umbenennung der Zeitkoordinate in eine räumliche Koordinate.
Ein Raumzeitintervall, eine Veränderung in der Raumzeit von Ereignis1 zu Ereignis2, kann man als Vektor darstellen, weil Vektoren Raum und Zeit gleichzeitig repräsentieren. Die Vierergeschwindigkeit ist nach meiner Ansicht eine der Lichtgeschwindigkeit entsprechende Beziehung zwischen Raum und Zeit ( =konstante Geschwindigkeit). Andere Geschwindigkeiten im herkömmlichen Sinn (Modell der Trennung von Raum und Zeit) sind in der Raumzeit nicht darstellbar, weil Raum und Zeit in der Raumzeit untrennbar miteinander verbunden sind (Minkowski).

Nach 5 Jahren (auf meiner Uhr) sind beide Zwillinge bei mir wieder angelangt.
Die Uhr des 1. Zwillings zeigt nach den 5 Jahren auf meiner Uhr an : 4.9994 Jahre
Die Uhr des 2. Zwillings zeigt nach den 5 Jahren auf meiner Uhr an: 0.5 Jahre


Das Beispiel scheint mir nicht so gelungen zu sein. Wie willst Du drei verschiedene Zeiten zu einem Zeitpunkt auf einer Uhr ablesen ?

MfG
Harti

TomS
14.09.15, 14:21
Nach 5 Jahren (auf meiner Uhr) sind beide Zwillinge bei mir wieder angelangt.
Die Uhr des 1. Zwillings zeigt nach den 5 Jahren auf meiner Uhr an : 4.9994 Jahre
Die Uhr des 2. Zwillings zeigt nach den 5 Jahren auf meiner Uhr an: 0.5 Jahre

Das Beispiel scheint mir nicht so gelungen zu sein. Wie willst Du drei verschiedene Zeiten zu einem Zeitpunkt auf einer Uhr ablesen ?

Es gibt DREI Beobachter und somit auch DREI Uhren:

Es gibt 3 Uhren: Eine die wir ablesen, eine des 1. Zwillings, eine des 2. Zwillings ...

Slash
16.09.15, 18:40
Hallo Slash
Ich denke mit den Koordinaten gibst Du einen Punkt, genannt Ereignis, in der vierdimensionalen Raumzeit an.


Richtig, es gibt bspw. das Ereignis "Start", bei dem sich die beiden von mir entfernen und das Ereignis "Ende", bei dem wir alle wieder beisammen sind und bei dem auf meiner Uhr "5 Jahre" angezeigt wird.

Der schnelle Zwilling hat zwischen "Start" und "Ende" viel Strecke im Raum zurückgelegt. Der andere wenig Strecke im Raum. Ich als Inertialsystem sogar gar keine Strecke im Raum.

Der schnelle Zwilling hat aber zwischen "Start" und "Ende" dafür weniger "Strecke" in der "Zeit" bzw. c*t zurückgelegt, da die Voraussetzung ja ist, dass die Vierergeschwindigkeit bei uns allen 3 konstant ist.

VG
Slash

PS: Ich bin kein Physiker und vielleicht ist die Erklärung nicht richtig.

Ich
16.09.15, 21:06
PS: Ich bin kein Physiker und vielleicht ist die Erklärung nicht richtig.Du hast die Erklärung schon richtig aus Wikipedia übernommen, und sie ist auch nicht falsch. Ich halte sie aber für didaktisch katastrophal. Es ist nur so, dass Harti auch bei der m.E. didaktisch richtigen Erklärung (Vektoren und deren Längen) wirklich kurz vorm Ziel ist und trotzdem dermaßen auf dem Schlauch steht, dass nix weitergeht.
Die richtige Erklärung wäre, dass die "Strecke in der Raumzeit", die sie zurücklegen, für jeden Zwilling anders ist. Der eine macht gar keinen Umweg, der andere schon einen, und der dritte einen richtig großen. Allerdingla ist die Geometrie so, dass die Strecke umso kürzer ist, je größer der Umweg (intuitiv schwierig, mathematisch aber sehr einfach). Und dass die Strecke direkt der verstrichenen Eigenzeit entspricht, also der Zeit, die für den jeweiligen Drilling oder Zwilling tatsächlich vergangen ist.

Das weiß Harti aber schon, denke ich, er will es aber irgendwie nicht akzeptieren. Oder er versteht einen der Punkte ums Verrecken nicht, ich kann aber nicht herausfinden, welchen.

Slash
16.09.15, 22:19
Die richtige Erklärung wäre, dass die "Strecke in der Raumzeit", die sie zurücklegen, für jeden Zwilling anders ist. Der eine macht gar keinen Umweg, der andere schon einen, und der dritte einen richtig großen. Allerdingla ist die Geometrie so, dass die Strecke umso kürzer ist, je größer der Umweg (intuitiv schwierig, mathematisch aber sehr einfach). Und dass die Strecke direkt der verstrichenen Eigenzeit entspricht, also der Zeit, die für den jeweiligen Drilling oder Zwilling tatsächlich vergangen ist.

Verstehe ich es richtig, dass die Strecke in der Raumzeit aber nur deshalb kürzer ist, wenn man die Eigenzeiten der Zwillinge betrachtet (die ja kürzer ist).

Bezogen auf das Inertialsystem ist die Strecke in der Raumzeit doch für alle 3 gleich, da 5 Jahre vergangen und sich alle vom Betrag her alle mit konstanter Vierergeschwindigkeit "fortbewegt" haben.

- Richtig?




Das weiß Harti aber schon, denke ich, er will es aber irgendwie nicht akzeptieren. Oder er versteht einen der Punkte ums Verrecken nicht, ich kann aber nicht herausfinden, welchen.

Gut, das kann ich nicht beurteilen. Er selbst ging nach meinem Verständnis von konstanter Vierergeschwindigkeit aus. Von daher finde ich auch die auf Wikipedia beschriebene Lösung / Interpretation nahezu intuitiv.

VG
Slash

Ich
16.09.15, 22:39
Verstehe ich es richtig, dass die Strecke in der Raumzeit aber nur deshalb kürzer ist, wenn man die Eigenzeiten der Zwillinge betrachtet (die ja kürzer ist).Nun...
Und dass die Strecke direkt der verstrichenen Eigenzeit entspricht, also der Zeit, die für den jeweiligen Drilling oder Zwilling tatsächlich vergangen ist.Wenn die Stecke kürzer ist, dann auch die Eigenzeit. Das ist einfach dasselbe.
Das ist alles ganz normal eigentlich. Wenn du im normalen Raum von A nach B über C fährst, machst du einen Umweg, die zurückgelegte Strecke ist länger als die direkte.
Der einzige Unterschied: In der Raumzeit rechnet man die Strecke über den umgekehrten Pythagoras aus, \sqrt{t^2-x^2}, und deswegen sind Umwege kürzer. Ist halt so.
Bezogen auf das Inertialsystem ist die Strecke in der Raumzeit doch für alle 3 gleich, da 5 Jahre vergangen und sich alle vom Betrag her alle mit konstanter Vierergeschwindigkeit "fortbewegt" haben.

- Richtig?Das genau ist die pädagogosche Katastrophe. Du hast sie nur noch nicht ganz ausgelebt: Eigentlich müssten deine drei Weltraumfahrer, wenn sie denn dieselbe Strecke (5 Jahre mit konstanter Vierergeschwindigkeit) mal gerade, mal leicht und mal stark geknickt zurücklegen, an unterschiedlichen Stellen rauskommen, oder? Was du da malst ist ein Eigenzeit-Ort-Diagramm. Die reisen weiter Richtung Ort. also dürfen sie nicht so weit in Richtung Zeit kommen.
Das funktioniert zwar mathematisch, aber es widerspricht vollkommen dem Wesen der RT. Die treffen sich wieder, aber nicht im gleichen Ereignis??? So eine Formulierung ist nicht zukunftsfähig.
Gut, das kann ich nicht beurteilen. Er selbst ging nach meinem Verständnis von konstanter Vierergeschwindigkeit aus. Von daher finde ich auch die auf Wikipedia beschriebene Lösung / Interpretation nahezu intuitiv.Denk' sie nochmal durch: Geschwindigkeit konstant, Umweg gemacht - also kann ich nicht so weit kommen wie der, der keinen Umweg macht. Du bist im Diagramm/in der Vorstellung nicht am selben Punkt. Eigentlich ist das aber das Wiedersehenereignis, wo ihr zur selben Zeit am selben Ort seid.
Glaub' mir, das hat mehr Nachteile als Vorteile.

Slash
18.09.15, 20:43
Hallo,

meines Ausführungen basieren auf den Annahmen, dass

- ich das Intertialsystem bin
- es 3 Uhren gibt: meine, die des langsamen Zwillings, die des schnellen Zwillings
- es gibt einen Start bei dem alle 3 Uhren auf 0 stehen und ein Wiedersehen, bei dem die 3 Uhren abgelesen werden
- die Vierergeschwindigkeit ist konstant.

Basierend auf diesen Annahmen versuchte ich zu veranschaulichen, warum die Uhren bei Wiedersehen unterschiedliche Zeiten anzeigen.

VG
Slash

TomS
19.09.15, 07:31
meines Ausführungen basieren auf den Annahmen, dass

- ich das Intertialsystem bin
- es 3 Uhren gibt: meine, die des langsamen Zwillings, die des schnellen Zwillings
- es gibt einen Start bei dem alle 3 Uhren auf 0 stehen und ein Wiedersehen, bei dem die 3 Uhren abgelesen werden
- die Vierergeschwindigkeit ist konstant.

Basierend auf diesen Annahmen versuchte ich zu veranschaulichen, warum die Uhren bei Wiedersehen unterschiedliche Zeiten anzeigen.
Ich denke, man benötigt gewisse geometrische Annahmen. M.E. die einfachste ist, Eigenzeit sowie Länge einer Weltlinie in der Raumzeit gleichzusetzen.

Mein Beispiel zur Veranschaulichung lautet

Ein Zwilling reist direkt von München nach Hamburg. Der andere reist von München zunächst nach Berlin und dann nach Hamburg. In Hamburg vergleichen beide den Kilometerstand ihrer Autos und stellen fest, dass sie unterschiedlich weit gefahren sind.

D.h. du benötigst m.E. nicht die Vierergeschwindigkeit an prominenter Stelle. Und du benötigst nicht unbedingt ein Koordinatensystem (später zur Rechnung ist es hilfreich, jedoch nicht zwingend).

Harti
19.09.15, 08:08
Hallo zusammen,
Ich wende bei meinen Überlegungen lediglich eine Äußerung eines Vaters der Relativitätstheorien konsequent an.
Hermann Minkowski (1908): "Von Stund an sollen Raum uns Zeit für sich völlig zu Schatten herabsinken, und nur noch eine Union der beiden soll Selbständigkeit bewahren"

In dieser Sicht der Dinge ist eine Trennung von Raum und Zeit und eine Beziehung zwischen beiden (Geschwindigkeit im herkömmlichen Sinn) nicht darstellbar. Es treten deshalb in einem Raumzeitmodell auch keine relativistischen Effekte auf. In der Raumzeit gibt es nur eine einzige Zeitachse und damit ist nur eine Uhr darstellbar. Dies wird bildlich häufig so dargestellt, dass der Raum von einem engmaschigen, zeitlichen Gitternetz durchzogen ist, an dessen Knotenpunkten gleichgehende Uhren befestigt sind.
Zwei im Verhältnis zueinander bewegte Uhren sind nicht darstellbar. Dies hätte nämlich zur Folge, dass man mathematisch zwei im Verhältnis zueinnader gedrehte oder verschobene Koordinatensysteme anwendem müsste.

[QUOTE=TomS;78447
Ganz einfaches Beispiel:
Ein Zwilling reist direkt von München nach Hamburg. Der andere reist von München zunächst nach Berlin und dann nach Hamburg. In Hamburg vergleichen beide den Kilometerstand ihrer Autos und stellen fest, dass sie unterschiedlich weit gefahren sind.[/QUOTE]

Unsere Meinungsverschiedenheit läßt sich unter Berücksichtigung der vorstehenden Äußerung auf folgende Frage reduzieren.
Bewegt der Reisezwilling, der zu seinem Ausgangspunkt zurückkehrt, sich nur in der Zeit oder auch im Raum. Diese Frage mag zunächst abwegig erscheinen, sie ist es auf der Grundlage eines Raumzeitmodells aber nicht.
Weil man in der Raumzeit Raum und Zeit nicht trennen kann, läßt sich nicht darstellen, dass der Reisezwilling sich in Raum und Zeit bewegt, der Erdzwilling aber nur in der Zeit. In einem Raumzeitmodell ist allein entscheidend, dass sich bei Ereignis1 (Abreise) und Ereigis2 (Rückkehr) beide Zwillinge am selben Ort befinden. Die Raumentfernung ist deshalb für beide Zwillinge 0. Die Berechnung sqrt (t^2- s^2) ergibt (für s = 0) = t . Die Raumzeitintervalle und damit das tatsächliche Geschehen, z.B. der Alterungsprozeß beider Zwillinge, ist gleich. Tatsächliches Geschehen ist in der Raumzeit invariant.

Ein weiteres Argument für meine Ansicht:
Uhren, die auf periodischen Bewegungen beruhen (Pendel, Quarz, Atomschwingung), werden als reine Zeitmessinstrumente angesehen.
Die Bewegung des Reisezwillings stellt im Prinzip die erste Periode einer solchen periodischen Bewegung dar.

MfG
Harti

Ich
19.09.15, 09:33
Harti,

das Forum Schulphysik darfst du gerne nutzen, um die SRT verstehen zu lernen und deine Irrtümer auszuräumen.
Wenn du derart gesicherte Physik als Gegenstand von Meinungsverschiedenheiten siehst oder dagegen argumentieren willst, dann ausschließlich im etnsprechenden Unterforum.

- Ich -

Ich
19.09.15, 09:33
Mein Tipp für dich: Lass die Raumzeit vorerst beiseite und versuche, Toms Beispiel zu verstehen.
Verstehst du, dass von München nach Hamburg unterschiedliche Distanzen zurückzulegen sind, je nachdem, wie man fährt? Und dass der gerade Weg der kürzeste ist?

TomS
19.09.15, 10:13
Ich wende bei meinen Überlegungen lediglich eine Äußerung eines Vaters der Relativitätstheorien konsequent an ...
Nein, das tust du nicht.

Einstein, Poincare, Lorentz, Minkowski u.a. haben einen mathematischen Rahmen geschaffen bzw. verwendet. Minkowski fasst hier in wenigen Worten und sehr pointiert einen Aspekt der RT zusammen.

Du kennst aber leider den mathematischen Rahmen nicht, und versuchst, dir auf diese Worte einen Reim zu machen. Das kann nur schiefgehen.

Ich hat recht: vergiss, was du vermeintlich über die RT weißt und versuche, mein einfaches Beispiel zu verstehen. Vergiss Eigenzeit, Zeitdilatation etc. und konzentriere dich auf die zurückgelegte Längen entlang der jeweiligen Reiserouten, abzulesen am Kilometerstand der Autos.

Harti
19.09.15, 10:37
Hallo,

Ich hat recht: vergiss, was du vermeintlich über die RT weißt und versuche, mein einfaches Beispiel zu verstehen. Vergiss Eigenzeit, Zeitdilatation etc. und konzentriere dich auf die zurückgelegte Längen entlang der jeweiligen Reiserouten, abzulesen am Kilometerstand der Autos.

Okay, vielleicht eine letzte Frage:

Verstehe ich die SRT richtig, dass Zeitdilatation und Längenkontraktion zwei Seiten derselben Medaille sind ?
Wie kann es dann sein, dass beim Wiedersehen der Zwillinge der Effekt der Längenkontraktion verschwindet, der Effekt der Zeitdilatation aber erhalten bleibt :confused:
MfG
Harti

Slash
19.09.15, 10:44
Wie kann es dann sein, dass beim Wiedersehen der Zwillinge der Effekt der Längenkontraktion verschwindet, der Effekt der Zeitdilatation aber erhalten bleibt :confused:
MfG
Harti

Der Unterschied ist in jedem Fall, dass mitgeführte Beschleunigungsmesser während den Reisen unterschiedliche Werte anzeigen.

Beschleunigung integriert über die Zeit ergibt Geschwindigkeit im 3D-Raum.

Ich würde nun anschaulich sagen, dass die Norm bzw. der Vierergeschwindigkeit in der Raumzeit für alle konstant bleibt. Somit reist der Zwilling, dessen Beschleunigungsmesser weniger (starke) Ausschläge anzeigt (entscheidend ist das Integral), "schneller in der Zeit" und er altert schneller, während der Zwilling der stark beschleunigt (egal ob in positiver oder negativer Raumrichtung, letzteres ist ja zum Rückkehren nötig) langsamer altert, also beim Treffen jünger ist.

VG
Slash

TomS
19.09.15, 11:15
Verstehe ich die SRT richtig, dass Zeitdilatation und Längenkontraktion zwei Seiten derselben Medaille sind ?
Wie kann es dann sein, dass beim Wiedersehen der Zwillinge der Effekt der Längenkontraktion verschwindet, der Effekt der Zeitdilatation aber erhalten bleibt
Auch so eine Darstellung, die aus dem Kontext gerissen ziemlich unsinnig erscheint. Ich halte Erklärungen der Zeitdilatation, die so vorgehen, für fragwürdig. Man kann letztere isoliert begründen.

TomS
19.09.15, 11:49
Der Unterschied ist in jedem Fall, dass mitgeführte Beschleunigungsmesser während den Reisen unterschiedliche Werte anzeigen.

Beschleunigung integriert über die Zeit ergibt Geschwindigkeit im 3D-Raum.

Ich würde nun anschaulich sagen, dass die Norm bzw. der Vierergeschwindigkeit in der Raumzeit für alle konstant bleibt. Somit reist der Zwilling, dessen Beschleunigungsmesser weniger (starke) Ausschläge anzeigt (entscheidend ist das Integral), "schneller in der Zeit" und er altert schneller, während der Zwilling der stark beschleunigt (egal ob in positiver oder negativer Raumrichtung, letzteres ist ja zum Rückkehren nötig) langsamer altert, also beim Treffen jünger ist.
Die Beschleunigung sollte bei der Erklärung des Phänomens der Zeitdilatation außen vor bleiben. Sie ist den Gleichungen nicht enthalten und sollt demnach auch nicht hineininterpretiert werden.

Slash
19.09.15, 12:04
Die Beschleunigung sollte bei der Erklärung des Phänomens der Zeitdilatation außen vor bleiben. Sie ist den Gleichungen nicht enthalten und sollt demnach auch nicht hineininterpretiert werden.

Ich denke, es geht nicht nur um die Erklärung der Zeitdilatation, sondern - wenn ich Hart richtig verstehe - gibt es zwei Ereignisse: ein Start und ein Wiedersehen und dazwischen verschieden zurückgelegte Wege beim Zwillingsparadoxon.

Deshalb sind Beschleunigungen zur Erklärung von Hartis "Problem" denke ich schon sinnvoll.

Beim Zwillingsparadoxon sind genaugenommen sehr wohl Beschleunigungen in der Problemstellung enthalten.

Ich
19.09.15, 12:40
Okay, vielleicht eine letzte Frage:

Verstehe ich die SRT richtig, dass Zeitdilatation und Längenkontraktion zwei Seiten derselben Medaille sind ?
Wie kann es dann sein, dass beim Wiedersehen der Zwillinge der Effekt der Längenkontraktion verschwindet, der Effekt der Zeitdilatation aber erhalten bleibtWas für ein Problem hast du eigentlich damit, einfach auf das zu antworten, was man dir schreibt? Interessiert dich die SRT oder nicht?

TomS
19.09.15, 13:14
Ich denke, es geht nicht nur um die Erklärung der Zeitdilatation, sondern - wenn ich Hart richtig verstehe - gibt es zwei Ereignisse: ein Start und ein Wiedersehen und dazwischen verschieden zurückgelegte Wege beim Zwillingsparadoxon.

Deshalb sind Beschleunigungen zur Erklärung von Hartis "Problem" denke ich schon sinnvoll.

Beim Zwillingsparadoxon sind genaugenommen sehr wohl Beschleunigungen in der Problemstellung enthalten.
Nein, sind sie nicht!

Zunächst mal ist die Beschleunigung zur Messung bzw. Berechnung einer Entfernung (oder Eigenzeit) entlang einer vorgegeben Reiseroute (oder Weltlinie) irrelevant. Nun ist es natürlich so, dass entlang der Reiserouten (oder Weltlinien) tatsächlich Beschleunigungen auftreten können; daraus folgt jedoch nicht, dass diese zur Erklärung der Entfernung benötigt werden. Siehe dazu mein Beispiel für die verschiedenen Reiserouten von München nach Hamburg.

Dann kann man das relativistische Zwillingsparadoxon bzgl. der Eigenzeit entlang einer Weltlinie geometrisch so formulieren, dass es fast exakt dem Beispiel der Reiseroute von München nach Hamburg und ihrer Länge entspricht. Beschleunigungen treten weder in der Rechnung noch bei der Erklärung auf. Zur mathematischen Formulierung des relativistische Zwillingsparadoxon siehe der von mir mehrfach zitierte Link bzw. Beitrag.

Mir ist bewusst, dass es „Erklärungen“ gibt, die mit Beschleunigungen argumentieren; diese sind in meisten Fällen irreführend und verfehlen den (eigtl. wesentlich einfacheren) Kern des Problems.

Slash
19.09.15, 14:05
Zunächst mal ist die Beschleunigung zur Messung bzw. Berechnung einer Entfernung (oder Eigenzeit) entlang einer vorgegeben Reiseroute (oder Weltlinie) irrelevant.

Bestreite ich auch nicht.

Eine Reiseroute besteht aber für mindestens einen der 2 oder 3 Protagonisten aus einer Geschwindigkeitsrichtungsänderung.


Mir ist bewusst, dass es „Erklärungen“ gibt, die mit Beschleunigungen argumentieren; diese sind in meisten Fällen irreführend und verfehlen den (eigtl. wesentlich einfacheren) Kern des Problems.

Hierzu wäre meine Frage, was denn außer Beschleunigungen noch Unterscheidungsmerkmale zwischen den Zwillingen relativ zueinander gesehen wären (wenn man nur 2 Zwillinge hat)?

Immerhin kann es die Relativ-Geschwindigkeit selbst nicht sein: Der eine Zwilling sieht den anderen ebenso wegfliegen und auf sich zukommen wie umgekehrt der andere (der Zwilling mit der geknickten Reiseroute könnte ja genauso argumentieren, dass doch relativ zu ihm der andere Zwilling derjenige mit der größeren Geschwindigkeit gewesen sein müsste, und dieser eben jünger sein müsste).

VG
Slash

Ich
19.09.15, 15:41
Hierzu wäre meine Frage, was denn außer Beschleunigungen noch Unterscheidungsmerkmale zwischen den Zwillingen relativ zueinander gesehen wären (wenn man nur 2 Zwillinge hat)?Du hast da grundsätzlich schon recht bei zwei Zwillingen. Es ist nur so, dass bei Erwähnung von Beschleunigung die Leute grundsätzlich der Meinung sind, diese hätten einen physikalischen Einfluss oder würden für die Erklärung benötigt. Dann kommen z.B. Schlaubies unter den Cranks und führen einen dritten "Zwilling" ein, der einwärts fliegt und am Umkehrpunkt seine Uhr nach dem auswärts fliegeneden stellt. Dann gibt es keine Beschleunigung und trotzdem einen Gangunterschied.
Daran sieht man, dass bei der Beschreibung in Inertialsystemen die Beschleunigung unnötig ist. Wichtig ist nur die geometrische Tatsache, dass die eine Weltlinie gerade ist und die andere nicht, weil sie dann unterschiedlich lang sein können.

Beschleunigung kann man auch einführen, das hat aber nur Sinn im Rahmen beschleunigter Bezugssysteme. Also im Grunde dann, wenn man ein Bezugssystem des reisenden Zwillings untersuchen will. Das ist definitiv lehhrreich und im Falle einfacher Beispiele auch gut zu rechnen. Dann ist der Zeitunterschied auf einmal der gravitativen Zeitdilatation geschuldet. Würde ich tatsächlich empfehlen, mal zu machen. Das ist aber definitiv fortgeschrittener Stoff, den man niemandem zumuten kann, der noch mit der Beschreibung in Inertialsystemen kämpft.

Slash
19.09.15, 18:29
Du hast da grundsätzlich schon recht bei zwei Zwillingen. Es ist nur so, dass bei Erwähnung von Beschleunigung die Leute grundsätzlich der Meinung sind, diese hätten einen physikalischen Einfluss oder würden für die Erklärung benötigt.

Hallo,

ja, es war nicht meine Absicht, die Beschleunigung für die Zeitdilatation verantwortlich zu machen. Ich fragte mich einfach (oder las nach), was der Unterschied zwischen den beiden Zwillingen ist, die ja relativ zueinander gesehen sich gegenseitig weg und herfliegen sehen, aber beide unterschiedlich beim Wiedersehen gealtert sind.

Von Berufswegen (Regelungstechnik, u.a.) fallen mir Geschwindigkeitssprünge sofort (negativ) auf, da sie natürlich bei realen (technischen) Systemen unendliche Leistung erfordern würden (kinetische Energie kann ja nicht sofort weg sein).

VG
Slash

Marco Polo
19.09.15, 19:53
Ich fragte mich einfach (oder las nach), was der Unterschied zwischen den beiden Zwillingen ist, die ja relativ zueinander gesehen sich gegenseitig weg und herfliegen sehen, aber beide unterschiedlich beim Wiedersehen gealtert sind.

Wenn du schon mit den Beschleunigungen argumentieren möchtest, dann dahingehend, dass nur der Reisezwilling während der Umkehr beim ZP beschleunigt ist.

Und da Beschleunigungen in der SRT nicht relativ sind, da nur der Beschleunigte Trägheitskräfte spürt (er wird in den Gurt bzw. Sitz gepresst), lässt sich absolut, also für beide Bezugssysteme verbindlich feststellen, wer von beiden beschleunigt.

Damit kann also der Reisezwilling nicht hingehen und behaupten, dass in seinem Bezugssystem der Erdzwilling es ist, der beschleunigt.

Man kann also nicht mehr von Symmetrie der beiden Bezugssysteme sprechen, da der Reisezwilling während der Umkehrphase ständig das Inertialsystem wechselt.

Aber wie bereits mehrfach erwähnt, ist der Weg über das Eigenzeitintegral der elegantere und auch den Kern der Sache treffendere.

Slash
19.09.15, 20:22
Wenn du schon mit den Beschleunigungen argumentieren möchtest, dann dahingehend, dass nur der Reisezwilling während der Umkehr beim ZP beschleunigt ist.

Ich argumentiere genau das.

Mindestens einer macht eine Geschwindigkeitsrichtungsänderung durch. Dies bedeutet eine Beschleunigung.


Und da Beschleunigungen in der SRT nicht relativ sind, da nur der Beschleunigte Trägheitskräfte spürt (er wird in den Gurt bzw. Sitz gepresst), lässt sich absolut, also für beide Bezugssysteme verbindlich feststellen, wer von beiden beschleunigt.

Damit kann also der Reisezwilling nicht hingehen und behaupten, dass in seinem Bezugssystem der Erdzwilling es ist, der beschleunigt.


Genau das will ich sagen.




Man kann also nicht mehr von Symmetrie der beiden Bezugssysteme sprechen, da der Reisezwilling während der Umkehrphase ständig das Inertialsystem wechselt.


Ja, schön, genau das wollte ich sagen.


Aber wie bereits mehrfach erwähnt, ist der Weg über das Eigenzeitintegral der elegantere und auch den Kern der Sache treffendere.

Kernpunkt beim Zwillingsparadoxon ist aus meiner Sicht jedoch, dass mind. ein Zwilling eine nicht von Null verschiedene Beschleunigung besitzt, oder eben ein Wechsel des Inertialsystems vollzieht, was als sprunghafte Änderung des Geschwindigkeitsvektors verstanden werden kann.

Aber so langsam wird´s mir auch zu blöd.

VG
Slash

Marco Polo
20.09.15, 02:57
Kernpunkt beim Zwillingsparadoxon ist aus meiner Sicht jedoch, dass mind. ein Zwilling eine nicht von Null verschiedene Beschleunigung besitzt, oder eben ein Wechsel des Inertialsystems vollzieht, was als sprunghafte Änderung des Geschwindigkeitsvektors verstanden werden kann.

Genau das ist dein Denkfehler. Beschleunigungen sind eben nicht der Kernpunkt des Zwillinngsparadoxons.

Kommen wir nochmal auf Eigenzeit und Weltlinien zurück.

Die Eigenzeit (und nur um die geht es), entspricht der Weltlinie (in diesem Fall ist das eine zeitartige Kurve in der Raumzeit) dieses Beobachters.

Die Weltlinie des Reisezwillings ist in der euklidischen Metrik länger als die des Ruhezwillings.

Intuitiv würde man jetzt fragen: Wenn die Weltlinie des Reisezwillings die längere ist, warum vergeht für ihn weniger Zeit wie beim Ruhezwilling? Müsste es nicht genau andersrum sein? Längere Weltlinie -> mehr Zeit vergangen.

Antwort: Nein, weil das eben nur eine scheinbare Länge ist. Die euklidische Geometrie der Zeichenebene stimmt nicht mit der pseudo-euklidischen Geometrie der Raumzeit überein.

So ähnlich verhält es sich bei einer Weltkarte. Wegen der dort unterschiedlichen Maßstäbe, ist die kürzere Strecke (der auch ein Flugzeug folgt) ein Bogen und nicht eine Gerade.

Die in der euklidischen Metrik kürzeste Verbindung hat in der Raumzeit-Metrik (Minkowski-Metrik) die größte Bogenlänge unter allen Weltlinien.

Das differentiell geschriebene Entfernungsmaß

ds²=c²dt²-dx²-dy² repräsentiert diese Minkowski-Metrik.

Es kann jeder Weltlinie ein Bogenmaß zugeschrieben werden: Das längs dieser Weltlinie erstreckte Integral ∫ds, das der Eigenzeit enspricht.

Es ist sinnvoll, das Zwilligsparadoxon mittels des Eigenzeitintegrals zu diskutieren. Ob da jetzt zwischendurch beschleunigt wurde oder nicht, interessiert nicht die Bohne. Maßgeblich ist nur die verstrichen Eigenzeit.

Es ist übrigens möglich, die Weltlinien der Zwillinge dergestalt zu präparieren, dass beide die gleichen Beschleunigungen erfahren, die einzelnen Abstände gleichförmiger Bewegung aber unterschiedlich lang sind.

Daran sieht man, dass Beschleunigungen für das Zustandekommen des Zwillingsparadoxons (das ohnehin keines ist) nicht maßgeblich sind.

Slash
20.09.15, 06:25
Es ist übrigens möglich, die Weltlinien der Zwillinge dergestalt zu präparieren, dass beide die gleichen Beschleunigungen erfahren, die einzelnen Abstände gleichförmiger Bewegung aber unterschiedlich lang sind.


Ja, wie?

PS: Ich habe nie behauptet, dass die Beschleunigung für die Zeitdilatation verantwortlich sind, wohl aber die unterschiedlichen Geschwindigkeiten im 3D-Raum bzgl. irgendeines Inertialssystems. Damit ein Wiedersehen statt findet, muss - will man einen Menschen oder eine Uhr zurückschicken - eine Umkehr stattfinden, das geschieht durch eine Beschleunigung dergestalt, dass ihr Integral zumindest für diese Geschwindigkeitsrichtungsänderung sorgt.

VG
Slash

TomS
20.09.15, 11:54
... wohl aber die unterschiedlichen Geschwindigkeiten im 3D-Raum bzgl. irgendeines Inertialssystems.
Auch ein Inertialsytem wird nicht benötigt!

Es vereinfacht die Rechnung, der Effekt selbst Rücktritt jedoch aus prinzipiell koordinatenfreien Darstellungen von invarianten Eigenzeiten.

Damit ein Wiedersehen statt findet,
muss ... eine Umkehr stattfinden ...
Nur dieses Wiedersehen muss man arrangieren.

das geschieht durch eine Beschleunigung dergestalt, dass ihr Integral zumindest für diese Geschwindigkeitsrichtungsänderung sorgt.
Der Witz ist aber, dass in das zu berechnende Integral lediglich der Betrag der Geschwindigkeit eingeht.

Nochmal: es verhält sich geometrisch exakt so wie bei den beiden unterschiedlichen Reiserouten von München nach Hamburg. Die Beschleunigung entlang der Fahrt ist irrelevant. Was zählt ist die Länge.

Ich verstehe übrigens nicht, warum du nicht einfach eine mathematische Herleitung akzeptierst, in der explizit keine Beschleunigung auftritt. Wenn da keine drin steckt, wie willst du sie hineininterpretieren? Wie funktioniert das für die Reisen von München nach Hamburg?

Slash
20.09.15, 12:09
Auch ein Inertialsytem wird nicht benötigt!

Ja, auch in Ordnung.

Es vereinfacht die Rechnung, der Effekt selbst Rücktritt jedoch aus prinzipiell koordinatenfreien Darstellungen von invarianten Eigenzeiten.


Nur dieses Wiedersehen muss man arrangieren.


Wie genau bitte? Geht es auch ohne Geschwindigkeitsrichtungsänderung?




Der Witz ist aber, dass in das zu berechnende Integral lediglich der Betrag der Geschwindigkeit eingeht.

Bestreite ich auch nicht.



Nochmal: es verhält sich geometrisch exakt so wie bei den beiden unterschiedlichen Reiserouten von München nach Hamburg. Die Beschleunigung entlang der Fahrt ist irrelevant. Was zählt ist die Länge.


Auch in Ordnung. Es geht hier aber (mir zumindest) um das vermeintliche Zwillingsparadoxon, bei man meinen könnte, dass sich die relativ zu einander nicht zu unterscheiden sind. Sie sind es aber, denn der, der weniger altert und somit mehr Weg zurückgelegt hat - warum, weil er mehr beschleunigt hat und Richtungsänderungen durchlaufen hat.





Ich verstehe übrigens nicht, warum du nicht einfach eine mathematische Herleitung akzeptierst, in der explizit keine Beschleunigung auftritt. Wenn da keine drin steckt, wie willst du sie hineininterpretieren? Wie funktioniert das für die Reisen von München nach Hamburg?


Beziehst du dich auf
http://www.physikerboard.de/topic,37752,-faq---zeitdilatation-und-zwillingsparadoxon.html

?

Und schreibst dort aber:

Natürlich kann v(t) des zweiten Beobachters (Zwillings) nicht vektoriell konstant sein, denn sonst könnte er nicht umkehren und zum ersten Zwilling an einem gemeinsamen Endpunkt zurückkehren.

... alles klar!

Harti
20.09.15, 15:55
Hallo Slash,

Geschwindigkeitsrichtungsänderung

Dieser von Dir verwendete Begriff macht das Problem deutlich, ob eine Bewegung als beschleunigt oder unbeschleunigt zu qualifizieren ist.
Klar ist, dass eine Bescheunigung immer dann vorliegt, wenn sich der Wert der Beziehung zwischen Weg (Raum) und Zeit ändert. Dies alleine reicht aber nicht aus für die Qualifizierung; es muss auch angegeben werden, in welchem Koordinatensystem die Bewegung betrachtet wird.

Beispiel: Freier Fall

In einem geradlinigen, kartesischen Koordinatensystem erscheint der freie Fall als beschleunigte Bewegung (Erdbeschleunigung), der Wert der Beziehung zwischen Weg und Zeit(Geschwindigkeit) ändert sich.

In einem gekrümmten Koordinatensystem, wie die Raumzeit es darstellt, erscheint der freie Fall, als unbeschleunigt, weil die Geschwindigkeitsänderung genau der Krümmung der Raumzeit an dieser Stelle entspricht. In diesem gekrümmten Koordinatensystem ändert sich die Beziehung zwischen Weg und Zeit nicht. Die Bewegung ist unbeschleunigt. Man nennt dies auch eine Bewegung auf einer Geodäte der Raumzeit.

Nach meiner Meinung ist die Richtungsänderung des Reisezwillings sowohl in einem geradlinigen Koordinatensystem wie auch in dem gekrümmten Koordinatensystem der Raumzeit als beschleunigt zu bezeichnen, weil sich der Wert der Beziehung zwischen Weg und Zeit bei der Richtungsänderung in jedem Fall ändert; der Reisezwilling bewegt sich auch in der Raumzeit nicht auf einer Geodäte.

Der Erdzwilling "bewegt" sich dagegen "unbeschleunigt", nämlich garnicht (bzw. nur in der Zeit), weil er sich räumlich nicht verändert.
Das Relativitätsprinzip ist deshalb auf die Veränderungen der Zwillinge nicht anwendbar.

MfG
Harti

TomS
20.09.15, 16:41
Geht es auch ohne Geschwindigkeitsrichtungsänderung?
Nein, weil sonst kein gemeinsamer Zielpunkt existiert. Die Richtungsänderung ist aber für den Effekt selbst irrelevant (ok, die Route von München über Berlin nach Hamburg ist länger; sie enthält mehr Kurven, also mehr Richtungsänderungen; ...; das ist aber nicht der Kern der Sache)

Es geht hier aber (mir zumindest) um das vermeintliche Zwillingsparadoxon, bei man meinen könnte, dass sich die relativ zu einander nicht zu unterscheiden sind. Sie sind es aber, denn der, der weniger altert und somit mehr Weg zurückgelegt hat ...
Genau; das ist der Kern der Sache; die Weglängen bzw. Weltlinien sind unterschiedlich

weil er mehr beschleunigt hat und Richtungsänderungen durchlaufen hat.
Das führt in die Irre. Spätestens wenn beide Zwillinge nicht gleichförmige, beschleunigte Bahnkurven durchlaufen, kannst du nicht mehr von einem "mehr beschleunigt" reden. Im Kern bleiben unterschiedlich lange Weltlinien.

Beziehst du dich auf
http://www.physikerboard.de/topic,37752,-faq---zeitdilatation-und-zwillingsparadoxon.html

?

Und schreibst dort aber:

Natürlich kann v(t) des zweiten Beobachters (Zwillings) nicht vektoriell konstant sein, denn sonst könnte er nicht umkehren und zum ersten Zwilling an einem gemeinsamen Endpunkt zurückkehren.

... alles klar!
S.o.

Die Erklärung in meinem Beitrag beginnt mit einer koordinatenfreien Darstellung, die für beliebig bewegte, auch beschleunigte Zwilling gültig ist. Daraus folgt eine Argumentation, die ohne den Begriff der Beschleunigung auskommt. Die Erklärung mit Beschleunigung bzw. Änderung der Bewegungsrichtung bzw. insbs. die Erklärung, dass die beiden Weltlinien nicht symmetrisch seien, ist erstens komplizierter und gilt zweitens nur in Spezialfällen. Ich halte sie deswegen für didaktisch ungeschickt.

Mir ist übrigens bewusst, dass 90% der Erklärungen anders erfolgen als meine. Ich halte meine trotzdem für besser :-)

Slash
20.09.15, 17:58
Das führt in die Irre. Spätestens wenn beide Zwillinge nicht gleichförmige, beschleunigte Bahnkurven durchlaufen, kannst du nicht mehr von einem "mehr beschleunigt" reden. Im Kern bleiben unterschiedlich lange Weltlinien.

Wenn ich von "mehr beschleunigt" rede, dann meine ich damit letzten Endes das Integral, also die Geschwindigkeit und dessen Integral, also die Weglänge und wir sind vermutlich (?) beisammen.

Gibt es nur ein schwarzes, dunkles Universum und nur die beiden Zwillinge, wobei jeder den anderen von sich weg und auf sich zubewegen sieht - am Ende ist der eine jünger als der andere und der der jünger ist, hat mehr auf das Gaspedal und die Bremse getreten, das ist, was ich sagen will. Und ich glaube, ich habe Recht.

VG
Slash

Slash
20.09.15, 18:05
Dieser von Dir verwendete Begriff macht das Problem deutlich, ob eine Bewegung als beschleunigt oder unbeschleunigt zu qualifizieren ist.

Hallo Harti,

in dem Zusammenhang, den ich meinte, ist die Beschleunigung "einfach" die Ableitung des Geschwindigkeitsvektors (also Betrag und Richtung der Geschwindigkeit) nach der Zeit im normalen 3D-Raum. So wie auf Wikipedia.
https://de.wikipedia.org/wiki/Beschleunigung
https://upload.wikimedia.org/math/4/9/5/49583239bed767a70e9536829fd2d3bd.png
Also ganz simpel.

Dass aus der Beschleunigung eine Geschwindigkeitsänderung (hinsichtlich Richtung und/oder Betrag) im normalen 3D-Raum folgt und damit - bei konstant angenommener Vierergeschwindigkeit eine andere "Geschwindigkeit in der Zeit" ist dann bei diesem Bild nur eine Folge davon.
Von einem gekrümmten Raum o.Ä. wollte ich bei der (für mich) anschaulichen Darstellung gar nicht ausgehen.

VG
Slash

TomS
20.09.15, 20:21
Wenn ich von "mehr beschleunigt" rede, dann meine ich damit letzten Endes das Integral, also die Geschwindigkeit und dessen Integral, also die Weglänge und wir sind vermutlich (?) beisammen.
Nein, noch nicht.

Du meinst ein Integral über eine Funktion der Geschwindigkeit; da in dieses Integral keine Beschleunigung eingeht, hat es nichts mit "mehr beschleunigt" zu tun. Geschwindigkeit und Weglänge: ja, Beschleunigung: nein.

Gibt es nur ein schwarzes, dunkles Universum und nur die beiden Zwillinge, wobei jeder den anderen von sich weg und auf sich zubewegen sieht - am Ende ist der eine jünger als der andere und der der jünger ist, hat mehr auf das Gaspedal und die Bremse getreten, das ist, was ich sagen will. Und ich glaube, ich habe Recht.
Ich glaube das nicht.

Ein Zwilling kann z.B. mit einer vergleichsweise geringen mittleren Geschwindigkeit jedoch mit starken Beschleunigungs- und Bremsmanövern unterwegs sein, während der andere mit einer hohen jedoch konstanten Geschwindigkeit unterwegs ist; letzteres gibt den Ausschlag, wieder nicht die Beschleunigung.

Berechne einfach mal

Slash
20.09.15, 23:08
Nein, noch nicht.

Du meinst ein Integral über eine Funktion der Geschwindigkeit; da in dieses Integral keine Beschleunigung eingeht, hat es nichts mit "mehr beschleunigt" zu tun. Geschwindigkeit und Weglänge: ja, Beschleunigung: nein.

Bzgl. Beschleunigungen, Geschwindigkeiten, Ableitungen und Integralen meine ich das, was Wikipedia sagt und sonst jedes Buch über Mechanik.


Ein Zwilling kann z.B. mit einer vergleichsweise geringen mittleren Geschwindigkeit jedoch mit starken Beschleunigungs- und Bremsmanövern unterwegs sein, während der andere mit einer hohen jedoch konstanten Geschwindigkeit unterwegs ist; letzteres gibt den Ausschlag, wieder nicht die Beschleunigung.

Berechne einfach mal

Beim Zwillingsparadoxon gibt es einen Start und ein Wiedersehen - diese Bedingung hinzugenommen ergibt (hoffe ich) meine Aussage:

Derjenige Zwilling der die Finger nicht vom Gas oder der Bremse (hier gemeint Verzögerung) lassen kann, ist am Ende der jüngere (behaupte ich und bin hier ziemlich sicher).

Jetzt rechne du mal

TomS
21.09.15, 04:12
Bzgl. Beschleunigungen, Geschwindigkeiten, Ableitungen und Integralen meine ich das, was Wikipedia sagt und sonst jedes Buch über Mechanik.
Wenn in ein Integral zur Berechnung der Zeitdilatation keine Beschleunigung eingeht, dann geht da keine ein.

Derjenige Zwilling der die Finger nicht vom Gas oder der Bremse (hier gemeint Verzögerung) lassen kann, ist am Ende der jüngere (behaupte ich und bin hier ziemlich sicher).

Jetzt rechne du mal
Habe ich. Du nicht. Your turn.

(ist eine ziemlich absurde Nummer, die du hier abziehst)

Slash
21.09.15, 05:53
Wenn in ein Integral zur Berechnung der Zeitdilatation keine Beschleunigung eingeht, dann geht da keine ein.


Habe ich. Du nicht. Your turn.

(ist eine ziemlich absurde Nummer, die du hier abziehst)

Danke für´s Diskreditieren. Das sagt mehr über dich aus, als über mich.

Außerdem nimmst du meine Aussagen aus dem Zusammenhang.

In die Berechnung der Zeitdilatation geht selbstverständlich nur die Geschwindigkeit ein.

Problemstellung des Zwillingsparadoxon ist aber nicht nur das der (unterschiedlichen) Zeitdilatation, sondern das des Starts und des gemeinsamen Wiedersehens. Relativ zueinander gesehen haben sie sich jeweils gegenseitig gleich voneinander weg und wieder zu bewegt, dennoch sind sie unterschiedlich gealtert.

Mir geht es auch nicht darum, dass die Beschleunigung Ursache für irgendetwas wäre, sondern das oder ein Unterscheidungsmerkmal zwischen den Zwillingen - denn sie unterscheiden sich doch offensichtlich in der Eigenzeit und dem zurückgelegten Weg - oder ? Wie kann man z.B. bzgl. unterschiedliche Wege zurücklegen, wenn man nicht beschleunigt? Geschwindigkeitssprünge / instane Richtungsändeurngen sind akademisch, aber beschreibbar durch einen Dirac-Impuls * Faktor in der Ableitung.

Ich habe auch nicht die Absicht, irgendetwas anderes zu behaupten, als bspw. in Wikipedia steht
https://de.wikipedia.org/wiki/Zwillingsparadoxon

Wenn ich mal Zeit und Lust finde, rechne ich es nach.

Ich hoffe, "ihr" seht nicht immer rote Tücher oder bekommt es in den falschen Hals, wenn jemand "Beschleunigung" im Zusammenhang mit dem Zwillingsparadoxon oder schreibt.

Immer cool bleiben, Wikipedia tut es ja auch, wie gesagt:
https://de.wikipedia.org/wiki/Zwillingsparadoxon

VG
Slash

Slash
21.09.15, 06:06
Wenn in ein Integral zur Berechnung der Zeitdilatation keine Beschleunigung eingeht, dann geht da keine ein.


Habe ich. Du nicht. Your turn.

Auf deiner Seite schreibst du:
"Natürlich kann v(t) des zweiten Beobachters (Zwillings) nicht vektoriell konstant sein, denn sonst könnte er nicht umkehren und zum ersten Zwilling an einem gemeinsamen Endpunkt zurückkehren. Man kann sich jedoch statt eines geraden Hin- und Rückfluges mit Verzögerung, Umkehren und Beschleunigen auch eine Kreisbahn mit konstantem Geschwindigkeitsbetrag vorstellen. In diesem Spezialfall folgt für das Integral "

http://www.physikerboard.de/topic,37752,-faq---zeitdilatation-und-zwillingsparadoxon.html

Also hast du eine mindestens eine (unendliche kurze) Beschleunigung und zwar in form eines Dirac-Impulses - dessen Integral 1 ergibt - * mal eines Vektors, der Betrag und die Änderung der Richtung angibt.

Kannst du das Zwillingsparadoxon auch formulieren, ohne Geschwindigkeits-Richtungsänderung?

VG
Slash

Marco Polo
21.09.15, 08:20
Hallo Slash,

natürlich muss man beschleunigen, um die Richtung zu ändern. Und natürlich muss man auch beschleunigen um zum anderen Zwilling zurückzukehren.

Aber dennoch ist die Beschleunigung nicht der Kern des Zwillingsparadoxons, wie weiter oben von dir behauptet.

Da zählen eben nur die Eigenzeiten. Und da, wie Tom bereits mehrfach betont hat, Beschleunigungen bei der Berechnung des Eigenzeitintegrals nicht auftauchen, werden diese wohl kaum den Kern des Zwillingsparadoxons ausmachen.

Hawkwind
21.09.15, 11:18
Hallo Slash,

natürlich muss man beschleunigen, um die Richtung zu ändern. Und natürlich muss man auch beschleunigen um zum anderen Zwilling zurückzukehren.

Aber dennoch ist die Beschleunigung nicht der Kern des Zwillingsparadoxons, wie weiter oben von dir behauptet.

Da zählen eben nur die Eigenzeiten. Und da, wie Tom bereits mehrfach betont hat, Beschleunigungen bei der Berechnung des Eigenzeitintegrals nicht auftauchen, werden diese wohl kaum den Kern des Zwillingsparadoxons ausmachen.

Wenn ich mich recht entsinne, wird dabei über

sqrt{1-[v(t)/c]^2}

integriert. Damit für 2 Beobachter, die gemeinsam starten, unterschiedliche v(t) zustande kommen, müssen die beiden - wohl oder übel - asymmetrisch beschleunigen.

Insofern wird Beschleunigung schon gebraucht. Dennoch ergibt sich der Zeitunterschied im wesentlichen durch die mit großem v durchlaufenen Wegstücke. Man "gewinnt" also mehr bei einem weitetstgehend gleichförmigen Dilatationsflug mit hochrelativistischem v als durch wilde Hin- und Herbeschleunigerei. :)

----
Prof. Lesch betont die Rolle von Beschleunigungen beim Zeitparadoxon übrigens auch sehr:


Tatsächlich ist es so, dass beim Zwillingsparadoxon der große Unterschied darin besteht, dass sich der, der wegfliegt, in einem beschleunigten Bezugssystem befindet. ...

aus seinem Buch "Der Außerirdische ist auch nur ein Mensch". :)
Aber an sich beantwortet das simple Integral eh alle Fragen.

Ich
21.09.15, 12:58
Hi Slash,

man könnte so eine Größe wie a²t definieren, die analog zu dem dir bekannten I²t die aufgetretenen Beschleunigungen summiert. Und dann ist es (in flachser Raumzeit) sicher auch so, dass der Zwilling mit a²t>0 jünger ist, wenn der andere a²t=0 hat. Und auch andersherum gilt: wenn wir von Weltlinien zweier Zwillinge sprechen, dann hat der jüngere a²t>0. Da gibt es also einen qualitativen Unterschied in den Zwillingen, und das wir auch gerne in der Populärwissenschaft herausgestrichen, um die oft fälschlich gemachte Annahme zu widerlegen, die beiden Bezugssysteme seie gleichwertig (was zu einem Paradox führen würde).
Also alles schön und gut.
Aber:
Wenn für beide a²t>0 gilt, kannst du anhand dieser Kenntnis nicht sagen, für wen weniger Zeit vergeht. Man kann natürlich über Integration aus der Beschleunigung die Geschwindigkeit ermitteln und dann \sqrt{1-v^2} wiederum integrieren, um an die Zeitdilatation zu kommen. Aber in dieser Rechnung ist a faktisch eliminiert und nur noch v vorhanden, so dass es vielleicht etwas zweifelhaft ist, in diesem Zusammenhang von "mehr beschleunigt" zu reden.

TomS
21.09.15, 13:09
In die Berechnung der Zeitdilatation geht selbstverständlich nur die Geschwindigkeit ein.
Gut.

Problemstellung des Zwillingsparadoxon ist aber nicht nur das der (unterschiedlichen) Zeitdilatation, sondern das des Starts und des gemeinsamen Wiedersehens. Relativ zueinander gesehen haben sie sich jeweils gegenseitig gleich voneinander weg und wieder zu bewegt, dennoch sind sie unterschiedlich gealtert.
Nein, haben sie nicht.

Wenn beide Zwillinge sich (unterschiedlich) beschleunigt bewegen, dann definieren beide keine Inertialsysteme im Sinne der SRT. Du müsstest jetzt die Bewegung eines Zwillings im Bezugssystem des anderen Zwillings darstellen; letzteres ist aber kein Inertialsystem. Natürlich sind dann die Weltlinie von 1 aus Sicht von 2 sowie die Weltlinie von 2 aus Sicht von 1 nicht mehr symmetrisch (wären sie es, müssten die Eigenzeitdifferenzen natürlich Null sein). Das Problem bei dieser Vorgehensweise ist zunächst, dass es extrem künstlich und aufwändig ist; bereits ein geradlinig und gleichförmig beschleunigtes Bezugssystem (Rindler-Koordinaten) ist mathematisch sehr aufwändig. Insbs. sind beide Koordinatensysteme nicht geometrisch vollständig, d.h. sie enthalten Horizonte, hinter denen dser jeweils andere Zwilling "verschwindet"; damit ist eine wechselweise Beschreibung wie "Weltlinie von 1 aus Sicht von 2 " evtl. gar nicht durchführbar.

Mir geht es auch nicht darum, dass die Beschleunigung Ursache für irgendetwas wäre, sondern das oder ein Unterscheidungsmerkmal zwischen den Zwillingen - denn sie unterscheiden sich doch offensichtlich in der Eigenzeit und dem zurückgelegten Weg - oder ?
Ja.

Wie kann man z.B. bzgl. unterschiedliche Wege zurücklegen, wenn man nicht beschleunigt? Geschwindigkeitssprünge / instane Richtungsändeurngen sind akademisch, aber beschreibbar durch einen Dirac-Impuls * Faktor in der Ableitung.
Da nur der Betrag der Geschwindigkeit in die Berechnung eingeht, ist eine Kreisbahn mit konstanter Bahngeschwindigkeit denkbar.

Der Punkt ist, dass du von einem Integral und "größerer Beschleunigung" gesprochen hast. Und das ist schlichtweg falsch. Du kannst explizit Weltlinien konstruieren, entlang derer eine kleine (mittlere) Geschwindigkeit mit einer prinzipiell unbeschränkt großen Beschleunigung vorliegt, und diese mit einer Weltlinie mit großer (mittlerer) Geschwindigkeit jedoch kleiner Beschleunigung vergleichen. Hier gibt die große (mittlere) Geschwindigkeit den Ausschlag, nicht die große Beschleunigung. Deswegen ist der Hinweis auf "größere Beschleunigung" oder "den Zwilling, der mehr beschleunigt oder verzögert", falsch.


Ich habe auch nicht die Absicht, irgendetwas anderes zu behaupten, als bspw. in Wikipedia steht
Nicht alles, was in der Wikipedia steht ist gut und richtig. Der genannte Artikel gefällt mir überhaupt nicht, da ständig von Beschleunigung und Inertialsystemen die Rede ist; beides ist unnötig und verschweigt den eigtl. geometrischen Kern der Sache.

Wenn ich mal Zeit und Lust finde, rechne ich es nach.
Ich werde eine Beitrag im Physikerboard mit einem Beispiel ergänzen.

EDIT: erledigt - siehe letzter Beitrag http://www.physikerboard.de/topic,37752,-faq---zeitdilatation-und-zwillingsparadoxon.html

Ich hoffe, "ihr" seht nicht immer rote Tücher oder bekommt es in den falschen Hals, wenn jemand "Beschleunigung" im Zusammenhang mit dem Zwillingsparadoxon oder schreibt.
Nicht unbedigt rote Tücher - es gibt schlimmeres. Es ist nur eben so, dass der Erklärungszugang über die Beschleunigung irreführend ist.

TomS
21.09.15, 13:14
Also hast du eine mindestens eine (unendliche kurze) Beschleunigung und zwar in form eines Dirac-Impulses ...
Nein, habe ich nicht.

Ein Zwilling bleibt in Ruhe auf der Erde. Der zweite Zwilling fliegt auf einer Kreisbahn mit konstanter Bahngeschwindigkeit; die Kreisbahn tangiert die Erde; er passiert die Erde periodisch nach jeweils einem Rundflug; bei jedem Vorbeiflug liest er seine Uhr ab stellt sie wieder auf Null zurück. Der Zwilling auf der Erde geht genauso vor. Bei jedem Vorbeiflug (Abstand Null) vergleichen die beiden Zwillinge ihre Uhrenm bevor sie sie auf Null zurücksetzen.

TomS
21.09.15, 13:15
Prof. Lesch betont die Rolle von Beschleunigungen beim Zeitparadoxon übrigens auch sehr

...

Aber an sich beantwortet das simple Integral eh alle Fragen.
Ja, leider tut er das. Warum, bleibt sein Geheimnis.

Hawkwind
21.09.15, 13:52
Nein, habe ich nicht.

Ein Zwilling bleibt in Ruhe auf der Erde. Der zweite Zwilling fliegt auf einer Kreisbahn mit konstanter Bahngeschwindigkeit; die Kreisbahn tangiert die Erde; er passiert die Erde periodisch nach jeweils einem Rundflug; bei jedem Vorbeiflug liest er seine Uhr ab stellt sie wieder auf Null zurück. Der Zwilling auf der Erde geht genauso vor. Bei jedem Vorbeiflug (Abstand Null) vergleichen die beiden Zwillinge ihre Uhrenm bevor sie sie auf Null zurücksetzen.

Das ist immer noch eine Situation, in der asymmetrisch beschleunigt wird: die Uhr dessen Zwilling, der die (beschleunigte) Kreisbewegung ausführt, zeigt weniger an.

Es macht m.E. keinen Sinn, die erforderliche asymmetrische Beschleunigung aus der Diskussion komplett herauslassen zu wollen. Sonst landet man bei einem echten Paradoxon, wo jeder meint, die Uhr des anderen zeige weniger an.

Bruhn kommt in
http://www.mathematik.tu-darmstadt.de/~bruhn/Eigenzeit-Integral.html
zu folgendem Fazit:

Die Ursache der Zeitdifferenz ΔT ist in der Unsymmetrie der Aufenthaltsbedingungen beider Zwillinge zu sehen. Während der erste Zwilling in einem Inertialsystem ruht, bewegt sich der reiselustige Zwilling in einem Nicht-Inertialsystem, d.h. er wechselt bei seiner Reise die Inertialsysteme: Das geschieht im Fall der Oszillatorbahn kontinuierlich, im Fall der vernachlässigten Beschleunigungsphasen dagegen sprungartig am Reiseanfang und -Ende sowie am Umkehrpunkt. Gelegentlich wird argumentiert, es sei die Verweildauer in bewegten Inertialsystemen der Grund für das Auftreten der Zeitdifferenz. Doch wäre ein Verweilen in bewegten Inertialsystemen ohne (u.U. abrupt erfolgende) Beschleunigung/Verzögerung überhaupt nicht möglich. Überdies sieht man an den obigen beiden Beispielen, dass Art und Verlauf der Beschleunigung den Wert der Zeitdifferenz wesentlich bestimmen. (s. dazu auch [2]).

Und m.E. trifft er den Nagel auf den Kopf.

TomS
21.09.15, 15:10
Es macht m.E. keinen Sinn, die erforderliche asymmetrische Beschleunigung aus der Diskussion komplett herauslassen zu wollen.
Das will auch niemand.

Bruhn kommt in zu folgendem Fazit ... Und m.E. trifft er den Nagel auf den Kopf.
M.E. nicht.

Der Kern ist die rein geometrische Eigenschaft der unterschiedlichen Länge von Weltlinien.

Argumentationen mit Beschleunigung, Inertialsystem vs. nicht-Inertialsystem usw. sind m.E. verfehlt (verbreitet, aber verfehlt!)

Ich zitiere hier gerne Ich - nicht mich:

Es ist nur so, dass bei Erwähnung von Beschleunigung die Leute grundsätzlich der Meinung sind, diese hätten einen physikalischen Einfluss oder würden für die Erklärung benötigt ... Daran sieht man, dass bei der Beschreibung in Inertialsystemen die Beschleunigung unnötig ist. Wichtig ist nur die geometrische Tatsache, dass die Weltlinien ... unterschiedlich lang sein können.

Hawkwind
21.09.15, 15:26
Der Kern ist die rein geometrische Eigenschaft der unterschiedlichen Länge von Weltlinien.


Aber die Länge von Weltlinien hat ja nun einmal mit Beschleunigung zu tun: wenn du 2 Ereignisse durch Weltlinien miteinander verbindest, dann sind die Weltlinien unbeschleunigter Objekte eben am längsten, d.h. auf ihnen vergeht am meisten Eigenzeit.

Slash
21.09.15, 17:43
Hi Slash,

man könnte so eine Größe wie a²t definieren, die analog zu dem dir bekannten I²t die aufgetretenen Beschleunigungen summiert. Und dann ist es (in flachser Raumzeit) sicher auch so, dass der Zwilling mit a²t>0 jünger ist, wenn der andere a²t=0 hat. Und auch andersherum gilt: wenn wir von Weltlinien zweier Zwillinge sprechen, dann hat der jüngere a²t>0. Da gibt es also einen qualitativen Unterschied in den Zwillingen, und das wir auch gerne in der Populärwissenschaft herausgestrichen, um die oft fälschlich gemachte Annahme zu widerlegen, die beiden Bezugssysteme seie gleichwertig (was zu einem Paradox führen würde).
Also alles schön und gut.
Aber:
Wenn für beide a²t>0 gilt, kannst du anhand dieser Kenntnis nicht sagen, für wen weniger Zeit vergeht. Man kann natürlich über Integration aus der Beschleunigung die Geschwindigkeit ermitteln und dann \sqrt{1-v^2} wiederum integrieren, um an die Zeitdilatation zu kommen. Aber in dieser Rechnung ist a faktisch eliminiert und nur noch v vorhanden, so dass es vielleicht etwas zweifelhaft ist, in diesem Zusammenhang von "mehr beschleunigt" zu reden.

Hallo Ich,

ich würde folgende Größe vorschlagen:

J = sqrt ( int ( aT(t) * a(t) ) dt ) und t = t_start bis t_ende

a(t) = (ax(t), ay(t), az(t))T , also der Beschleunigungsvektor (transponiert, damit oben das Skalarprodukt ein Skalar ergibt)


Wenn für beide a²t>0 gilt, kannst du anhand dieser Kenntnis nicht sagen, für wen weniger Zeit vergeht.

Ich bin mir auch nicht sicher, ob J dann ein Maß dafür wäre, festzustellen, welcher Zwilling der jüngere ist, ich glaube aber ja - müsste man nachrechnen.

VG
Slash

Slash
21.09.15, 18:06
Aber die Länge von Weltlinien hat ja nun einmal mit Beschleunigung zu tun: wenn du 2 Ereignisse durch Weltlinien miteinander verbindest, dann sind die Weltlinien unbeschleunigter Objekte eben am längsten, d.h. auf ihnen vergeht am meisten Eigenzeit.

Gibt es es - wenn es nur den leeren Raum(zeit) gibt und die beiden Zwillinge - ein für sie messbares Unterscheidungsmerkmal, zu entscheiden, wer von ihnen der jüngere ist (außer der Eigenzeit)?

VG
Slash

Slash
21.09.15, 19:06
Aber dennoch ist die Beschleunigung nicht der Kern des Zwillingsparadoxons, wie weiter oben von dir behauptet.

Hallo Marco,

ich denke Kern ist die Asymmetrie.

VG
Slash

Slash
21.09.15, 19:11
Nein, habe ich nicht.

Ein Zwilling bleibt in Ruhe auf der Erde. Der zweite Zwilling fliegt auf einer Kreisbahn mit konstanter Bahngeschwindigkeit; die Kreisbahn tangiert die Erde; er passiert die Erde periodisch nach jeweils einem Rundflug; bei jedem Vorbeiflug liest er seine Uhr ab stellt sie wieder auf Null zurück. Der Zwilling auf der Erde geht genauso vor. Bei jedem Vorbeiflug (Abstand Null) vergleichen die beiden Zwillinge ihre Uhrenm bevor sie sie auf Null zurücksetzen.


Aber dann hast du die Beschleunigung:

a = v² / r

https://de.wikipedia.org/wiki/Zentrifugalkraft#Formeln


es gilt außerdem Umfang u = 2*pi*r
und T = Zeit pro Rundflug
also v = u / T

also a = v² / r = 4*pi² * r/ T)

also a ~ r -> Beschleunigung proportional zum Radius , eben der Weglänge.

Das gilt auch bei zwei Zwillingen.

Vielleicht sehe ich es auch falsch, bitte gerne korrigieren.

VG
Slash


VG
Slash

Slash
21.09.15, 19:29
Ich werde eine Beitrag im Physikerboard mit einem Beispiel ergänzen.

EDIT: erledigt - siehe letzter Beitrag http://www.physikerboard.de/topic,37752,-faq---zeitdilatation-und-zwillingsparadoxon.html


Du hast aber hier in deinem Beispiel mit der kreisförmigen Bewegung auch eine Beschleunigung.

(Wie oben in einem anderen Beitrag bemerkt).

Ich
21.09.15, 20:57
Ich bin mir auch nicht sicher, ob J dann ein Maß dafür wäre, festzustellen, welcher Zwilling der jüngere ist, ich glaube aber ja - müsste man nachrechnen.
Nein, sicher nicht. Wenn du von der Regelungstechnik kommst, kannst du dir bei gegebenem J beliebige Bewegungsprofile ausdenken, die alle vollkommen unterschiedliche Durchschnittsgeschwindigkeiten (bzw. Zeitdilatationen) haben.
Ein fast triviales Beispiel ist, wenn einer am Anfang wegbeschleunigt und spät wieder so zurück beschleunigt, dass er wieder am Ausgangspunkt ankommt. Der hat eine höhere mittlere Geschwindigkeit als einer, der erst lange wartet und kurz voe Schluss weg- und hinbeschleunigt.
ich denke Kern ist die Asymmetrie.
Ja, Kern des vermeintlichen Paradoxons ist die nicht vorhandene, aber unterstellte Symmetrie. Das kann man auch über die Erwähnung von Beschleunigung auflösen. Damit kommst du aber nur soweit, dass man keine gleichen Ergebnisse für die Zwillinge erwarten muss, aber nicht dahin, was man stattdessen erwarten sollte.

Was alle anderen hier immer meinen ist nicht das Paradoxon an sich, denke ich, sondern die Erklärung für die Tatsache, dass unterschiedliche Zeiten vergehen. Die liegt in der Geometrie. Das Schöne an dieser Erklärung ist, dass sie gleichzeitig das Paradoxon auf höchst intuitive Weise auflöst, viel nachvollziehbarer als jede Rechnerei mit Beschleunigungen. Es ist einfach für jeden (außer Harti, fürchte ich) vollkommen klar, dass unterschiedliche Wege unterschiedliche Längen haben können.

TomS
21.09.15, 21:28
Die Argumentation mit fehlender Symmetrie läuft dann ins Leere, wenn beide Zwillinge jeweils unterschiedlich beschleunigen und somit keiner von beiden ein Inertialsystem definiert. Dann kann man z.B. durch simplen Vergleich der beiden Beschleunigungen a(t) sofort feststellen, dass keine Symmetrie vorliegt. Daraus alleine folgt jedoch nichts!

Betrachtet man dagegen die Geschwindigkeiten v(t) und berechnet die Integrale, dann folgt daraus sofort die Zeitdilatation.

@Slash, Hawkwind: ja, auch in meinen Beispielen liegen Beschleunigungen vor; aber daraus folgt - außer in Spezialfällen - nichts.

Hawkwind
21.09.15, 21:44
Betrachtet man dagegen die Geschwindigkeiten v(t) und berechnet die Integrale, dann folgt daraus sofort die Zeitdilatation.


Ja, sowieso.:)

Hawkwind
21.09.15, 21:55
Gibt es es - wenn es nur den leeren Raum(zeit) gibt und die beiden Zwillinge - ein für sie messbares Unterscheidungsmerkmal, zu entscheiden, wer von ihnen der jüngere ist (außer der Eigenzeit)?

VG
Slash

Der Unterschied der Eigenzeiten ist doch per definitionem genau das, was die Zeitdilatation ausmacht. Eigenzeit ist die Zeit die auf der mitgenommenen Armbanduhr des Reisenden verstreicht.

Einfach einsetzen in Toms 2. Formel von oben in seiner Postings im anderen Forum:

http://www.matheboard.de/latex2png/latex2png.php?\tau%20=%20\int_C%20d\tau%20=%20\int _{t_a}^{t_b}dt\sqrt{\dot{x_\mu}\dot{x^\mu}}=%20\in t_{t_a}^{t_b}dt\sqrt{1-\vec{v}^2(t)}

Das mit den Weltlinien und den Eigenzeiten ist auch bei Embacher ganz gut erklärt, finde ich:
http://homepage.univie.ac.at/franz.embacher/SRT/Zwillingsparadoxon.html

Slash
21.09.15, 22:00
Die Argumentation mit fehlender Symmetrie läuft dann ins Leere, wenn beide Zwillinge jeweils unterschiedlich beschleunigen und somit keiner von beiden ein Inertialsystem definiert. Dann kann man z.B. durch simplen Vergleich der beiden Beschleunigungen a(t) sofort feststellen, dass keine Symmetrie vorliegt. Daraus alleine folgt jedoch nichts!

Man könnte doch ein 3. Inertialsystem definieren, auf welchem Start und Wiedersehereignis stattfinden.


Betrachtet man dagegen die Geschwindigkeiten v(t) und berechnet die Integrale, dann folgt daraus sofort die Zeitdilatation.

Betrachtet man dagegen Beschleunigung und Anfangsbedingung a(t) + v0 berechnet die Integrale, also die Geschwindigkeiten v(t) und berechnet die Integrale, dann folgt daraus fast auch sofort die Zeitdilatation. :p:D ;)


@Slash, Hawkwind: ja, auch in meinen Beispielen liegen Beschleunigungen vor; aber daraus folgt - außer in Spezialfällen - nichts.

Da bin ich zu wenig Physiker, um das beurteilen zu können.

VG
Slash

Slash
21.09.15, 22:16
Nein, sicher nicht. Wenn du von der Regelungstechnik kommst, kannst du dir bei gegebenem J beliebige Bewegungsprofile ausdenken, die alle vollkommen unterschiedliche Durchschnittsgeschwindigkeiten (bzw. Zeitdilatationen) haben.

Da wäre ich mir nicht so sicher. Nach dem Satz von Feldbaum (gibt es leider keinen eigenen Wikipedia-Eintrag) kommst du am schnellsten von A nach B, wenn du bis zur Mitte maximal beschleunigst ist und danach maximal verzögerst. Natürlich wäre es in diesem Fall die Umkehrung. So etwas wie int (I² (t)) dt oder int( a² (t) ) dt stehen da schon im Zusammenhang mit dem Weg, da ja die Kurvenlänge (Weglänge) ganz ähnlich definiert wird - nämlich über das Integral der quadrierten Geschwindigkeit und Wurzel, etc. Und die Durchschnittsgeschwindigkeit ist ja Weglänge / Zeit.

Außerdem sind ja die Randbedingungen gegeben, dass sich die Zwillinge beim Start und am Ende treffen.

VG
Slash

Struktron
21.09.15, 22:32
Hallo TomS,

...
@Slash, Hawkwind: ja, auch in meinen Beispielen liegen Beschleunigungen vor; aber daraus folgt - außer in Spezialfällen - nichts.

In #22 (http://www.quanten.de/forum/showpost.php5?p=77987&postcount=22) schriebst Du noch:
Nein, das bedeutet, dass man zur Lösung auch beschleunigte Bewegungen betrachten muss, was im Rahmen der SRT möglich ist.

Jetzt kommt die Diskussion langsam in die Richtung, dass durch Slash's Beitrag in #125 (http://www.quanten.de/forum/showpost.php5?p=78613&postcount=125) bei mir die Idee geweckt wird, dass viele kleine Beschleunigungen im Sinne der Newton'schen Idee für die Entwicklung der Infinitesimalrechnung als stückweise gerade Geschwindigkeitsänderungen interpretiert werden könnten. Die Knickfunktionen können dann mMn auf Heaviside'sche Sprungfunktionen und diese auf Dirac'sche Deltafunktionen führen. Mit diesen können wir dann die (scheinbar?) kontinuierlichen Beschleunigungen erklären. Hinter jedem kleinen Intervall steckt dann ein Ereignis, welches auch den Weg vergrößert.
Müssen wir vielleicht doch die Beschleunigung als etwas Grundlegendes mit in den Erklärungsversuch einbeziehen?

@alle anderen:
Wenn wir gespürte Beschleunigungen nicht berücksichtigen, ist es dann beim "Zwillingsparadoxon" nicht so wie bei "Schrödingers Katze"? Bevor man nicht nachsieht (landet) kann man nicht wissen, wessen Standpunkt richtig ist?

MfG
Lothar W.

Slash
22.09.15, 05:30
Der Unterschied der Eigenzeiten ist doch per definitionem genau das, was die Zeitdilatation ausmacht. Eigenzeit ist die Zeit die auf der mitgenommenen Armbanduhr des Reisenden verstreicht.

Einfach einsetzen in Toms 2. Formel von oben in seiner Postings im anderen Forum:

http://www.matheboard.de/latex2png/latex2png.php?\tau%20=%20\int_C%20d\tau%20=%20\int _{t_a}^{t_b}dt\sqrt{\dot{x_\mu}\dot{x^\mu}}=%20\in t_{t_a}^{t_b}dt\sqrt{1-\vec{v}^2(t)}

Das mit den Weltlinien und den Eigenzeiten ist auch bei Embacher ganz gut erklärt, finde ich:
http://homepage.univie.ac.at/franz.embacher/SRT/Zwillingsparadoxon.html


Der Gedanke, den ich meinte war folgender, kannst ja korrigieren:

Variante A:
Alice und Berta sind zusammen und stellen ihre Uhren auf Null.
Berta fliegt von Alice weg und kommt nach 5 Jahren wieder zu Alice.
Am Ende stellen sie fest, dass Berta 6 Jahre gealtert ist.


Variante B:
Alice und Berta sind zusammen und stellen ihre Uhren auf Null.
Berta fliegt von Alice weg und kommt nach 5 Jahren wieder zu Alice.
Am Ende stellen sie fest, dass Berta 4 Jahre gealtert ist.


Ohne Hinzunahme von Beschleunigung - oder sonst einem Unterscheidungsmerkmal - können doch sowohl A und B möglich sein. Wie ist denn sonst unterscheidbar, wer von wem wegfliegt und am Ende den längeren Weg zurücklegt, wenn nicht dadurch beispielsweise, dass einer der Zwillinge eben beschleunigt?

VG
Slash



PS: In deinem Link steht selbst:


Gegeben seien zwei Ereignisse A und B, die durch Weltlinien verbunden werden können. (Diese Bedingung soll ausschliessen, dass Überlichtgeschwindigkeit nötig ist, um von A nach B zu gelangen). Unter allen Weltlinien, die in A beginnen und in B enden, entspricht jene der kräftefreien Bewegung, für die die längste Eigenzeit vergangen ist. Eine solche Weltlinie wird Geodäte (der Raumzeit) genannt. Eine Geodäte der Raumzeit ist, wenn man es so ausdrücken will, die "längste Verbindung zweier Punkte" − wobei "längste" im Sinne der Eigenzeit gilt, unter "Verbindung" eine Weltlinie gemeint ist und "Punkte" für "Ereignisse" steht.

Ich
22.09.15, 06:37
Da wäre ich mir nicht so sicher.Dann probier's aus. Gegenbeispiele lassen sich leicht finden, ich hab' schon eins genannt.

Timm
22.09.15, 09:51
Das mit den Weltlinien und den Eigenzeiten ist auch bei Embacher ganz gut erklärt, finde ich:
http://homepage.univie.ac.at/franz.embacher/SRT/Zwillingsparadoxon.html
Finde ich auch, liest sich fast wie ein Konzentrat dieses Threads, abzüglich dessen Verirrungen. Das letzte Diagramm mit der verallgemeinerten Darstellung der Ereignisse A und B enthält meines Erachtens den sog. "Kern" und darüber hinaus nichts unnötiges.

Marco Polo
22.09.15, 11:08
Finde ich auch, liest sich fast wie ein Konzentrat dieses Threads, abzüglich dessen Verirrungen. Das letzte Diagramm mit der verallgemeinerten Darstellung der Ereignisse A und B enthält meines Erachtens den sog. "Kern" und darüber hinaus nichts unnötiges.

Seh ich auch so.

Vielleicht kann man sagen, dass die Behandlung des ZP mittels der koordinatenfreien Eigenzeit, vorbereitend auf den Übergang zu der, der SRT übergeordneten Theorie der ART, die geeignete Maßnahme ist, weil allgemeingültiger.

Mit anderen Worten: Es entspricht schon eher der ART-Denkweise, über das Eigenzeitintegral zu gehen, als über die SRT-typischen Lorentztrafos.

soon
22.09.15, 12:22
Wie stellt sich die Situation in einem Versuch ohne Richtungsänderung dar?

Person_A und Person_B bewegen sich von Ort_1 zu Ort_2. Sie starten gleichzeitig. Person_A bewegt sich langsam. Person_B bewegt sich sehr schnell.

Person_B wartet an Ort_2 auf Person_A. Wenn Person_A ebenfalls Ort_2 erreicht hat, stellen beide fest, dass Person_A stärker gealtert ist, - in jedem Fall von Geschwindigkeits- und Wartezeitkonstellationen.

Ist das zutreffend?

Ich
22.09.15, 12:33
Wie stellt sich die Situation in einem Versuch ohne Richtungsänderung dar?

Person_A und Person_B bewegen sich von Ort_1 zu Ort_2. Sie starten gleichzeitig. Person_A bewegt sich langsam. Person_B bewegt sich sehr schnell.

Person_B wartet an Ort_2 auf Person_A. Wenn Person_A ebenfalls Ort_2 erreicht hat, stellen beide fest, dass Person_A stärker gealtert ist, - in jedem Fall von Geschwindigkeits- und Wartezeitkonstellationen.

Ist das zutreffend?Ja, ist es.
Das mit der "Richtungsänderung" gilt natürlich nur im System des "ruhenden" (hier: langsam bewegten) Zwillings. Eigentlich handelt es sich um eine Geschwindigkeitsänderung, sprich: Knick in der Weltlinie.

Hawkwind
22.09.15, 13:33
Der Gedanke, den ich meinte war folgender, kannst ja korrigieren:

Variante A:
Alice und Berta sind zusammen und stellen ihre Uhren auf Null.
Berta fliegt von Alice weg und kommt nach 5 Jahren wieder zu Alice.
Am Ende stellen sie fest, dass Berta 6 Jahre gealtert ist.


Variante B:
Alice und Berta sind zusammen und stellen ihre Uhren auf Null.
Berta fliegt von Alice weg und kommt nach 5 Jahren wieder zu Alice.
Am Ende stellen sie fest, dass Berta 4 Jahre gealtert ist.


Ohne Hinzunahme von Beschleunigung - oder sonst einem Unterscheidungsmerkmal - können doch sowohl A und B möglich sein. Wie ist denn sonst unterscheidbar, wer von wem wegfliegt und am Ende den längeren Weg zurücklegt, wenn nicht dadurch beispielsweise, dass einer der Zwillinge eben beschleunigt?


Da oben steht ja das Eigenzeitintegral: wenn du dich nun für ein Inertialsystem entscheidest und in diesem dann das entsprechende v(t) von Alice einsetzt und es berechnest, dann bekommst du für Alice z.B. 4h - und das ist sogar UNABHÄNGIG VON DEM INERTIALSYSTEM, für das du dich entschieden hast - immer 4 h. Und für Berta z.B. immer 5 h. Diese Eigenzeit ist ja eine Invariante, d.h. unabhängig vom IS.
Alles andere wäre auch absurd: die Armbanduhr von Alice kann nach Abschluss ihrer Reise ja nicht für den einen Beobachter was anderes anzeigen als für einen anderen.

soon
22.09.15, 13:36
Abschweifende, vielleicht weiterführende Frage:

In der Phase der konstanten Geschwindigkeit wird keine Energie verbraucht.

In den Phasen der Beschleunigung verbraucht die langsame Person_A weniger Energie als Person_B. Beim Wiederzusammentreffen ist Person_B zwar weniger stark gealtert, dafür ist aber der Tank des Raumschiffs leer.

Bei der Alterung/Weiterentwicklung der Personen finden eine unterschiedliche Anzahl an Wechselwirkungen statt.
Bei den unterschiedlichen Beschleunigungen wird unterschiedlich viel Energie benötigt und es findet damit ebenfalls eine unterschiedliche Anzahl an Wechselwirkungen statt.

Ist es denkbar, dass die Anzahl der benötigten Wechselwirkungen beim Wiederzusammentreffen der Personen unterm Strich wieder gleich ist?


Das mit der "Richtungsänderung" gilt natürlich nur im System des "ruhenden" (hier: langsam bewegten) Zwillings. Eigentlich handelt es sich um eine Geschwindigkeitsänderung, sprich: Knick in der Weltlinie.

Gibt es einen greifbaren Zusammenhang zwischen den Begriffen 'Ereignis' in der Raumzeit und 'Wechselwirkung'?

Marco Polo
22.09.15, 14:37
Ist es denkbar, dass die Anzahl der benötigten Wechselwirkungen beim Wiederzusammentreffen der Personen unterm Strich wieder gleich ist?

Die Frage ergibt für mich keinen Sinn.

Gibt es einen greifbaren Zusammenhang zwischen den Begriffen 'Ereignis' in der Raumzeit und 'Wechselwirkung'?

Ein Ereignis kann alles Mögliches sein. Auch nur ein Punkt in einem Raumzeitdiagramm.

Timm
22.09.15, 15:23
Vielleicht kann man sagen, dass die Behandlung des ZP mittels der koordinatenfreien Eigenzeit, vorbereitend auf den Übergang zu der, der SRT übergeordneten Theorie der ART, die geeignete Maßnahme ist, weil allgemeingültiger.

Mit anderen Worten: Es entspricht schon eher der ART-Denkweise, über das Eigenzeitintegral zu gehen, als über die SRT-typischen Lorentztrafos.
Könnte sein, die invariante Eigenzeit spielt jedenfalls bei Diskussionen zum Fall ins Schwarze Loch eine große Rolle. Andererseits gibt es auch die ART-typischen und bevorzugten mitbewegten Koordinaten.

Ich glaube den Übergang zur ART findet man eher beim Vergleich benachbarter Geodäten.

Ich
22.09.15, 16:11
Bei der Alterung/Weiterentwicklung der Personen finden eine unterschiedliche Anzahl an Wechselwirkungen statt.
Bei den unterschiedlichen Beschleunigungen wird unterschiedlich viel Energie benötigt und es findet damit ebenfalls eine unterschiedliche Anzahl an Wechselwirkungen statt.

Ist es denkbar, dass die Anzahl der benötigten Wechselwirkungen beim Wiederzusammentreffen der Personen unterm Strich wieder gleich ist?

Was dieser Beschreibung wohl am ehesten entspricht ist das Prinzip der kleinsten Wirkung, das dir den eingeschlagenen Weg unter Einbeziehung äußerer Kräfte gibt. Da ist so was wie die "Summe aus Eigenzeit und Wechselwirkung" konstant - ganz grob gesprochen. Vielleicht hilft dir eine Suche nach principle least action special relativity oder so weiter. Oder TomS.

Slash
22.09.15, 18:36
Dann probier's aus. Gegenbeispiele lassen sich leicht finden, ich hab' schon eins genannt.

Ehrlich gesagt habe ich es nicht ganz verstanden. Könntest du grob Zahlenwerte nennen, wann die Personen warten und wann sie beschleunigen?

Mir kommt es so vor, als ob dein Gegenbeispiel nur auf den ersten Blick plausibel erscheint, es muss aber bedacht werden, dass die beiden sich am Ende wieder treffen. Derjenige, die die größte Weglänge in dieser Zeit geflogen ist, sollte auch betragsmäßig am meisten aufintegriert beschleunigt haben und das sollte im 1-dimensionalen Fall wie im 3-dim. vektoriellen Fall gelten.

Kann aber sein, dass ich mich irre, nur dein Beispiel hatte ich nicht verstanden.

VG
Slash

Marco Polo
22.09.15, 19:06
Andererseits gibt es auch die ART-typischen und bevorzugten mitbewegten Koordinaten.

Genau diese entsprechen doch der Eigenzeit, oder nicht?

TomS
22.09.15, 20:33
Das mit den Weltlinien und den Eigenzeiten ist auch bei Embacher ganz gut erklärt, finde ich:
http://homepage.univie.ac.at/franz.embacher/SRT/Zwillingsparadoxon.html
Finde ich nur teilweise.

M.E. sollte die Argumentation nicht auf Inertialsysteme fokussieren (diese sind nur Spezialfälle) und insbs. nicht mittels Lorentztransformationen argumentieren.

TomS
22.09.15, 20:38
Derjenige, die die größte Weglänge in dieser Zeit geflogen ist, sollte auch betragsmäßig am meisten aufintegriert beschleunigt haben ...
Ich habe bzgl. Beschleunigung wie versprochen das Gegenargument explizit durchgerechnet; siehe Link, letzter = neuester Beitrag; "betragsmäßig am meisten aufintegriert beschleunigt" ist demnach irrelevant.

Marco Polo
22.09.15, 20:40
M.E. sollte die Argumentation nicht auf Inertialsysteme fokussieren (diese sind nur Spezialfälle) und insbs. nicht mittels Lorentztransformationen argumentieren.

Embacher hat das in seinem Bild mit den Weltlinien ja relativiert.

Slash
22.09.15, 21:29
Ich habe bzgl. Beschleunigung wie versprochen das Gegenargument explizit durchgerechnet; siehe Link, letzter = neuester Beitrag; "betragsmäßig am meisten aufintegriert beschleunigt" ist demnach irrelevant.

Danke.

Ich verstehe deine Rechnung nicht ganz.

Es handelt sich um eine kreisförmige Bewegung.

Bezeicnet delta * v(t) eine tangentiale Beschleunigung bzw. den Anteil der zur tangentialen Geschwindigkeit aufgrund einer (tangentialen) Beschleunigung hinzuaddiert wird?

Wie behandelst du die Beschleunigung zum Mittelpunkt der Flugbahn für die Zwillinge?

Das für in die Formel für die Zeitdilatation nur die Geschwindigkeit eingeht habe ich (hoffe ich) nicht bestritten, aber das letzten Endes für die Weglänge und die Bedingung des gemeinsamen Starts und Wiedersehens eine Beschleunigung diese beeinflusst, sowie die Geschwindigkeit ist doch offensichtlich. Insofern weiß ich nicht, um was es dir genau geht.

Ich bin mittlerweile hier gelandet:

http://www.einstein-online.info/vertiefung/Zwillinge?set_language=de
http://www.einstein-online.info/vertiefung/ZeitdilatationWanderer

VG
Slash

Ich
22.09.15, 21:34
Kann aber sein, dass ich mich irre, nur dein Beispiel hatte ich nicht verstanden.
Du gibst z.B. jedem die gleichen Beschleunigungsphasen, wobei die Umkehrbeschleunigung doppelt so stark ist wie die Auswärtsbeschleunigung. Dann dauert das Zurückfliegen genauso lange wie das Rausfliegen.
Der eine startet ganz am Anfang und kehrt zur Hälfte der Zeit um.
Der andere startet nach 98% der Zeit und kehrt nach 99% um.
Dann vergeht für den einen bei weitem weniger Zeit als für den anderen. Und daran ändert sich auch nichts, wenn du ihn betragsmäßig aufintegriert etwas mehr oder weniger beschleunigen lässt als den anderen.

Mal ein kurzer Ausflug in höhere Gefilde (ich habs aber schon mal angesprochen): Wenn du schon mit Beschleunigung argumentieren willst, dann in beschleunigten Bezugssystemen. Die dort wirkenden Scheinkräfte bewirken "gravitative" Zeitdilatation bzw. -kontraktion. Die ist proportional zum Produkt aus Abstand und Beschleunigung - im Erdschwerefeld z.B. g*h genannt. Und das musst du integrieren, um den gravitativen Anteil an der Zeitdilatation zu bekommen.
Dazu musst du die geschwindigkeitsabhängige Zeitdilatation zählen. Im beschleunigten System ist ja der Raketenzwilling in Ruhe und der Erdzwilling unterwegs. Dieser Anteil würde für den Erdzwilling weniger verstrichene Zeit bedeuten.
Die gravitative Zeitdilatation aber wirkt entgegengesetzt und doppelt so stark. In Summe vergeht doch für den Erdzwilling mehr Zeit.

Das alles kann man noch rechnen, es hat da aber einige Fallstricke. Ich denke, das würde hier zu weit führen.

Timm
22.09.15, 21:43
Genau diese entsprechen doch der Eigenzeit, oder nicht?
Die Zeit, die für einen mitbewegten Beobachter vergeht, ist seine Eigenzeit. Was aber eigentlich trivial ist. Vielleicht meinst Du es anders.

Marco Polo
22.09.15, 21:50
Die Zeit, die für einen mitbewegten Beobachter vergeht, ist seine Eigenzeit. Was aber eigentlich trivial ist. Vielleicht meinst Du es anders.

Ich meinte es nicht anders. Des mitbewegten Beobachters Eigenzeit entspricht des beobachteten Objektes Eigenzeit.

Slash
22.09.15, 22:08
Du gibst z.B. jedem die gleichen Beschleunigungsphasen, wobei die Umkehrbeschleunigung doppelt so stark ist wie die Auswärtsbeschleunigung. Dann dauert das Zurückfliegen genauso lange wie das Rausfliegen.
Der eine startet ganz am Anfang und kehrt zur Hälfte der Zeit um.
Der andere startet nach 98% der Zeit und kehrt nach 99% um.
Dann vergeht für den einen bei weitem weniger Zeit als für den anderen. Und daran ändert sich auch nichts, wenn du ihn betragsmäßig aufintegriert etwas mehr oder weniger beschleunigen lässt als den anderen.

Hallo,

ja, ich hatte mir nun auch Gedanken gemacht.

Es gibt ja schon Verfahren und teilweise auch belegte Sätze in der Regelungstechnik, z.B. wie man von A nach B in kürzester Zeit oder mit geringster Stellleistung (Antrieb einer Roboterachse, etc.) kommt.

Um aber die Problemstellung anhand dieser Verfahren "einfach" lösen zu können, wären zumindest noch weitere Randbedingungen erforderlich, z.B. ob die Zwillinge beim Start v= 0 zueinander haben und ob sie auch beim Wiedersehen v=0 wieder haben sollen, sonst wird es denke ich wirklich unübersichtlich.

Außerdem müsste auch eine Beschleunigungsgrenze (z.B. maximale Schubkraft eines Triebwerks) definiert werden, um das noch übersichtlich lösen zu können.

Wenn all dies der Fall ist, dann gilt - denke ich - in der Tat der Satz von Feldbaum, der verkürzt lautet: Gib Vollgas bis zur Mitte, bremse dann bis zum Schluss, so kommst du am weitesten in vorgebener Zeit, dann für den Rückweg natürlich das Gleiche, wobei man am Ende stillsteht.



Mal ein kurzer Ausflug in höhere Gefilde (ich habs aber schon mal angesprochen): Wenn du schon mit Beschleunigung argumentieren willst, dann in beschleunigten Bezugssystemen. Die dort wirkenden Scheinkräfte bewirken "gravitative" Zeitdilatation bzw. -kontraktion. Die ist proportional zum Produkt aus Abstand und Beschleunigung - im Erdschwerefeld z.B. g*h genannt. Und das musst du integrieren, um den gravitativen Anteil an der Zeitdilatation zu bekommen.

Nein, danke, mir reicht die Betrachtung im 3D-Raum !! :p

Aber stimmt, das könnte man bestimmt auch gut simulieren.

VG
Slash

Marco Polo
22.09.15, 22:18
Um aber die Problemstellung anhand dieser Verfahren "einfach" lösen zu können, wären zumindest noch weitere Randbedingungen erforderlich, z.B. ob die Zwillinge beim Start v= 0 zueinander haben und ob sie auch beim Wiedersehen v=0 wieder haben sollen, sonst wird es denke ich wirklich unübersichtlich.

Nein, es wird nicht unübersichtlich. Es spielt qualitativ keine Rolle für den Zeitversatz, welche Relativgeschwindigkeit beim Start und Wiedersehen beider Zwillinge berücksichtigt wird.

TomS
22.09.15, 22:25
Es handelt sich um eine kreisförmige Bewegung.

Bezeichnet delta * v(t) eine tangentiale Beschleunigung bzw. den Anteil der zur tangentialen Geschwindigkeit aufgrund einer (tangentialen) Beschleunigung hinzuaddiert wird?
Zunächst ja. Man könnte das Argument natürlich auch auf nicht-kreisförmige Bewegungen verallgemeinern.

Wie behandelst du die Beschleunigung zum Mittelpunkt der Flugbahn für die Zwillinge?
Gar nicht; sie ist irrelevant (siehe Herleitung im ersten Beitrag)

;)

Dass für in die Formel für die Zeitdilatation nur die Geschwindigkeit eingeht habe ich (hoffe ich) nicht bestritten ...
Stimmt, hast du nicht.

... aber dass letzten Endes für die Weglänge und die Bedingung des gemeinsamen Starts und Wiedersehens eine Beschleunigung diese beeinflusst, sowie die Geschwindigkeit ist doch offensichtlich.
Stimmt, das ist auch offensichtlich. Eine andere Beschleunigung ergibt eine andere Bahnkurve - aber nicht unbedingt eine andere Eigenzeit.

Insofern weiß ich nicht, um was es dir genau geht.
Es geht genau darum, dass die Beschleunigung nicht eingeht. Bereits dass die Geschwindigkeit eingeht, ist eine rechentechnische praktische, jedoch prinzipiell unnötige Vorgehensweise. Im ersten Beitrag habe ich das auch zu Beginn so nicht eingeführt.

1)
Von München nach Hamburg sind's (laut Google Maps) 775 km.
Von München nach Berlin und dann nach Hamburg sind's 584 km + 288 km.
Die zweite Route ist länger als die erste.
Ende der Information.

2)
Alternativ gebe ich dir zwei (vektorwertige) Funktionen a(t) und fordere dich auf, die entsprechenden Reiserouten zu vergleichen. Du stellst nach längerer Rechnung fest, dass sie unterschiedlich lang sind, dass sie jedoch - bei identischem Startpunkt - auch identischen Zielpunkt haben.
Wiederum ist die zweite Route länger als die erste.

Warum ist es von München über Berlin nach Hamburg weiter als direkt von München nach Hamburg? Weil das aus einer komplizierten Berechnung mittels der Beschleunigungen folgt? Inkl. Abbremsen, Beschleunigen, Tankstopps, Anhalten zum Kaffeetrinken, Stau, ...? Oder einfach weil die Strecke länger ist? Wie würdest du argumentieren, wenn jemand dich fragt, welche Strecke länger ist? Im Atlas nachmessen? Oder die beiden Funktionen a(t) integrieren?

Ich würde nachmessen und konstatieren, dass die Strecke über Berlin weiter ist, unabhängig davon, wie schnell ich fahre oder wie ich beschleunige und abbremse. Insbs. weiß ich, dass eine derartige Reise für alle gleich lang ist, unabhängig davon, wie schnell sie fahren oder wie sie beschleunigen. Analog ist das mit den Eigenzeiten bzw. Längen der Weltlinien (außer dass der Entfernungsbegriff in der Minkowsi-Geometrie anders definiert ist).

Ich
22.09.15, 22:31
Um aber die Problemstellung anhand dieser Verfahren "einfach" lösen zu können, wären zumindest noch weitere Randbedingungen erforderlich, z.B. ob die Zwillinge beim Start v= 0 zueinander haben und ob sie auch beim Wiedersehen v=0 wieder haben sollen, sonst wird es denke ich wirklich unübersichtlich.Ich habe OBdA mal angenommen, dass sie am Anfang in Ruhe zueinander sind und sich nach Ablauf einer gesetzten Zeit mir beliebiger Geschwindigkeit wieder treffen.
Außerdem müsste auch eine Beschleunigungsgrenze (z.B. maximale Schubkraft eines Triebwerks) definiert werden, um das noch übersichtlich lösen zu können.Derlei Grenzen musst du nicht der Übersichtlichkeit halber einführen, sondern um überhaupt eine Lösung, wie speziell auch immer, zu erhalten. Weil der von dir postulierte Zusammenhang mit der Beschleunigung nicht existiert.
Wenn all dies der Fall ist, dann gilt - denke ich - in der Tat der Satz von Feldbaum, der verkürzt lautet: Gib Vollgas bis zur Mitte, bremse dann bis zum Schluss, so kommst du am weitesten in vorgebener Zeit, dann für den Rückweg natürlich das Gleiche, wobei man am Ende stillsteht.Ja schön, hilft aber nicht wirklich für die Problematik. Zwei solche Profile zusammengesetzt würden wohl tatsächlich eine extremale Zeitdilatation unter den gegebenen Randbedingungen liefern, aber danach hat niemand gesucht. Die Frage war, ob das Beschleunigungsintegral auf die Zeitdilatation schließen lässt, und das ist nicht der Fall.
Nein, danke, mir reicht die Betrachtung im 3D-Raum !Das ist auch 3D-Raum. Nur eben ein beschleunigtes Bezugssystem. Ich fasse das Ergebnis nochmal zusammen, dann muss man sich die Details nicht antun:
Wenn man die Bezugssysteme der beiden Zwillinge betrachtet, dann ist das eine ein Inertialsystem, das andere ist beschleunigt.
In inertialen Systemen gibt es nur die kinematische Zeitdilatation (von Geschwindigkeit herrührend, das, wovon wir hier immer reden). In beschleunigten Systemen gibt es zusätzlich gravitative Zeitdilatation.
Die kinematische Dilatation ist im wesentlichen gleich für beide Zwillinge. Nach ihr geht die Uhr des jeweils anderen langsamer. Das ist hübsch symmetrisch, und wenn das alles wäre, gäbe es ein Paradox.
Die gravitative Zeitdilatation im beschleunigten System aber bewirkt, dass am Schluss sich doch beide einig sind, wessen Uhr nachgeht: Die des Reisenden nämlich.

Marco Polo
22.09.15, 22:34
Ich würde nachmessen und konstatieren, dass die Strecke über Berlin weiter ist, unabhängig davon, wie schnell ich fahre oder wie ich beschleunige und abbremse. Insbs. weiß ich, dass eine derartige Reise für alle gleich lang ist, unabhängig davon, wie schnell sie fahren oder wie sie beschleunigen. Analog ist das mit den Eigenzeiten bzw. Längen der Weltlinien (außer dass der Entfernungsbegriff in der Minkowsi-Geometrie anders definiert ist).

Genau das ist der Kern der Problematik. Keine Ahnung, wie man das anders sehen kann.

Slash
22.09.15, 23:02
Genau das ist der Kern der Problematik. Keine Ahnung, wie man das anders sehen kann.

Ich weiß ja nicht, ob du mich damit meinst, aber ich habe das auch nie bestritten.

Gegen was reagiert "man" denn hier so allergisch, nur weil einem das Wort Beschleunigung im Zusammenhang mit dem Zwillingsparadoxon nicht passt?

Und dass mindestens einmal ein Zwilling eine Geschwindigkeitsrichtungsänderung durchmacht ist in der Aufgabenstellung nunmal angelegt.

Es will doch niemand die Formel der Zeitdilatation in Frage stellen in diesem Kontext. Ich jedenfalls nicht, das kann ich gar nicht.

VG
Slash

Marco Polo
22.09.15, 23:12
Ich weiß ja nicht, ob du mich damit meinst...

Indirekt vielleicht schon.

Gegen was reagiert "man" denn hier so allergisch, nur weil einem das Wort Beschleunigung im Zusammenhang mit dem Zwillingsparadoxon nicht passt?

Ich habe nicht den Eindruck, dass auf die Beschleunigungsgeschichte hier allergisch reagiert wird. Eher sachlich.

Und dass mindestens einmal ein Zwilling eine Geschwindigkeitsrichtungsänderung durchmacht ist in der Aufgabenstellung nunmal angelegt.

Das bestreitet ja auch niemand.

Es will doch niemand die Formel der Zeitdilatation in Frage stellen in diesem Kontext. Ich jedenfalls nicht, das kann ich gar nicht.


Natürlich nicht.

Slash
22.09.15, 23:16
Die Frage war, ob das Beschleunigungsintegral auf die Zeitdilatation schließen lässt, und das ist nicht der Fall.

Mein Gedankengang war lediglich, dass ich einen Zusammenhang zwischen Beschleunigungsintegral und Weglänge herstellen will / wollte und das ganz einfach mit den üblichen Gleichungen der Technischen Mechanik, fertig.

Alles andere habe ich nicht behauptet und würde da um ein bisschen Fairness bitten.

Aber ich habe ein Wenig den Eindruck, dass man ein bisschen Spaß darain findet, andere misszuverstehen. Der kann auch falsch sein.

Bzgl. der anderen Aussagen wäre ich mir nicht so sicher. Die Herleitung des Satzes von Feldbaum ist meines Wissens nach sehr kompliziert und das Ergebnis sehr einfach. Mit offenen Randbedingungen geht es sicherlich auch.

VG
Slash

Marco Polo
22.09.15, 23:20
Mein Gedankengang war lediglich, dass ich einen Zusammenhang zwischen Beschleunigungsintegral und Weglänge herstellen will / wollte und das ganz einfach mit den üblichen Gleichungen der Technischen Mechanik, fertig.

Die "technische Mechanik" ist nicht geeignet, dass Zwillingsparadoxon zu diskutieren.

Slash
22.09.15, 23:26
Nein, es wird nicht unübersichtlich. Es spielt qualitativ keine Rolle für den Zeitversatz, welche Relativgeschwindigkeit beim Start und Wiedersehen beider Zwillinge berücksichtigt wird.

Auf diese Idee wäre ich auch nicht gekommen, woraus schließt du denn, dass ich das geschrieben oder gemeint hätte?

???

Slash
22.09.15, 23:29
Die "technische Mechanik" ist nicht geeignet, dass Zwillingsparadoxon zu diskutieren.

Es ging wie ich schrieb nicht um die Zeitdilatation, sondern eine ganz andere Aufgabenstellung, die im Rahmen der Diskussion aufkam.

Abgesehen davon mal wieder ein Satz aus dem Zusammenhang gerissen.

Schade!

Ich
22.09.15, 23:51
Mein Gedankengang war lediglich, dass ich einen Zusammenhang zwischen Beschleunigungsintegral und Weglänge herstellen will / wollte und das ganz einfach mit den üblichen Gleichungen der Technischen Mechanik, fertig.

Alles andere habe ich nicht behauptet und würde da um ein bisschen Fairness bitten.

Aber ich habe ein Wenig den Eindruck, dass man ein bisschen Spaß darain findet, andere misszuverstehen. Der kann auch falsch sein.

Die Fairness ist da, und insbesondere von mir weiß ich, dass ich mir alle Mühe gebe, dich zu verstehen und auf dich einzugehen. Ich habe sogar deine "Wechselwirkungen" in Wirkungsintegrale übersetzt, und ich wette, dass das kaum jemand nachvollziehen kann.
Und jetzt erwarte ich von dir, dass du anhand meines Beispiels nachvollziehst, dass es auch keinen Zusammenhang zwischen Beschleunigungsintegral und Weglänge gibt. Damit du deinen Vorwurf selbst entkräftest.

TomS
22.09.15, 23:58
Gegen was reagiert "man" denn hier so allergisch, nur weil einem das Wort Beschleunigung im Zusammenhang mit dem Zwillingsparadoxon nicht passt?
Es geht nicht um allergische sondern um sachliche Argumentation. Siehe hier - 19.09. - ungefähr der Beitrag, an dem sich diese Dikussion entzündet hat:

... somit reist der Zwilling, dessen Beschleunigungsmesser weniger (starke) Ausschläge anzeigt (entscheidend ist das Integral), "schneller in der Zeit" und er altert schneller, während der Zwilling der stark beschleunigt ... langsamer altert, also beim Treffen jünger ist.
Ich habe zunächst argumentiert, dass in keiner Berechnung der Zeitdilatation überhaupt eine Beschleunigung verwendet wird, natürlich auch kein Integral, das deiner Aussage zufolge angeblich entscheidend ist.

Dann habe ich anhand einer konkreten Berechnung gezeigt, dass starke (und sogar beliebig große) Beschleunigungen irrelevant sind, und dass weiterhin die Geschwindigkeiten dominieren, entgegen deiner Aussage.

Ich habe außerdem darauf hingewiesen, dass die Längen der Weltlinien wesentlich sind, das Auftreten der Geschwindigkeiten ein "technisches Detail", und die Beschleunigungen sogar überflüssig und unnötig kompliziert (da eh' nur dazu verwendet, sie zu integrieren unm Geschwindigkeiten zu berechnen).

Ich habe argumentiert, dass unterschiedliche Beschleunigungen notwendig sind, um einen gemeinsamen Start- und Zielpunkt zu haben, jedoch nicht hinreichend für eine einfache Argumentation im Falle zweier beschleunigter Reisender.

Es geht doch darum, eine vernünftige, allgemeingültige und möglichst einfache Begründung für die Zeitdilatation zu finden. Genau das leisten Beschleunigungen nicht: sie verkomplizieren die Argumentation unnötig (weil man sie erst integrieren muss), oder sie suggerieren eine Argumentation mittels Asymmetrie (ein Reisender beschleunigt, der andere nicht) - eine Argumentation, die sofort zusammenbricht, wenn beide beschleunigen.

Ich finde nun nicht, dass wir dich da missverstanden haben, sondern dass du den Kern der Zeitdilatation missverstanden hast.

Slash
23.09.15, 05:23
Ich habe argumentiert, dass unterschiedliche Beschleunigungen notwendig sind, um einen gemeinsamen Start- und Zielpunkt zu haben, jedoch nicht hinreichend für eine einfache Argumentation im Falle zweier beschleunigter Reisender.


Das war auch meine Argumentation ich versuchte lediglich, sie etwas zu quantifizieren - vielleicht falsch, vielleicht richtig.



Es geht doch darum, eine vernünftige, allgemeingültige und möglichst einfache Begründung für die Zeitdilatation zu finden.

Vielleicht ist das das Missverständnis, mir ging nicht darum eine Begründung für die Zeitdilatation zu finden, sondern um Hartis Frage zu beantworten.



Genau das leisten Beschleunigungen nicht: sie verkomplizieren die Argumentation unnötig (weil man sie erst integrieren muss), oder sie suggerieren eine Argumentation mittels Asymmetrie (ein Reisender beschleunigt, der andere nicht) - eine Argumentation, die sofort zusammenbricht, wenn beide beschleunigen.

Ich finde nun nicht, dass wir dich da missverstanden haben, sondern dass du den Kern der Zeitdilatation missverstanden hast.

Für mich jedenfalls kann ich nur sagen, dass es hier doch um eine Aufgabenstellung, nämlich das Zwillingsparadoxon geht.

Das zu beschreiben, zu lösen, zu diskutieren, darum ging es mir, aufgrund der anschaulichen Annahme der konstanten Vierergeschwindigkeit.

Das war mein Ansatz um es einem anderen Laien im Rahmen des Forums "Schulphysik" zu erklären.

Es wird dieses Beispiel "Zwillingsparadox" diskutiert, als handle es sich selbst um ein Grundgesetz der Natur.

Slash
23.09.15, 05:38
Die Fairness ist da, und insbesondere von mir weiß ich, dass ich mir alle Mühe gebe, dich zu verstehen und auf dich einzugehen. Ich habe sogar deine "Wechselwirkungen" in Wirkungsintegrale übersetzt, und ich wette, dass das kaum jemand nachvollziehen kann.
Und jetzt erwarte ich von dir, dass du anhand meines Beispiels nachvollziehst, dass es auch keinen Zusammenhang zwischen Beschleunigungsintegral und Weglänge gibt. Damit du deinen Vorwurf selbst entkräftest.

Kann ich bestätigen.

Ich war nur etwas "schockiert", als du das Wort katastrophal verwendest, nachdem ich überhaupt Beschleunignung in den Mund genommen hatte. Ich vermute, das liegt daran, dass es zu dem Thema - mir gar nicht bekannt - schon früher schon Diskussionen gab.

Ich kann ohne die Randbedingungen: Gemeinsamer Startpunkt, Gemeinsame Geschwindigkeit beim Start und beim Wiedersehen = 0 sowie eine betragsmäßig maximal mögliche Beschleunigung (wie in der Technik oft vorhanden) in der Tat keine allgemeine gültige Aussage treffen, die meine "Beschleunigungsintegral" - Argumentation untermauert. Wenn diese Bedingungen vorhanden sind, dann denke ich, dass der Satz von Feldbaum anwendbar ist (dann sind es Rechtecke, also die maximale Fläche unter dem Integral und alles andere ist vom Betrag her flächenmäßig weniger).

Das einzige was ich kann ist folgendes feststellen:

Es gibt anschauliche Formulierungen / Darstellungen des Zwillingsparadoxon, die argumentieren, dass mindestens ein Zwilling (kurzzeitig) beschleunigt sein muss, während es der andere nicht muss, weil eben ein Zwilling eine Richtungsänderung durchführt. Wenn man nun sagt, dass eine Richtungsänderung eine Beschleunigung ist, dann finde ich es nicht so abwegig, dies auch so zu nennen. Andererseits ist der Begriff Beschleunigung vielleicht im Rahmen der SRT / ART zu sehr vorbelastet. Aus dieser Richtung komme ich jedoch nicht.

VG
Slash

Slash
23.09.15, 05:45
Mein Eindruck ist, dass du das Wort "anschaulich", dass ich vor allem in den ersten Beiträgen meiner Antwort an Harti verwendet habe übersehen hast.

VG
Slash

TomS
23.09.15, 06:00
Das war auch meine Argumentation ich versuchte lediglich, sie etwas zu quantifizieren - vielleicht falsch, vielleicht richtig.
So wie ich das sehe, sicher falsch.


... mir ging nicht darum eine Begründung für die Zeitdilatation zu finden, sondern um Hartis Frage zu beantworten.
Ich sehe nicht, welche Frage deine Argumentation sinnvoll beantworten könnte.

Für mich jedenfalls kann ich nur sagen, dass es hier doch um eine Aufgabenstellung, nämlich das Zwillingsparadoxon geht.

Das zu beschreiben, zu lösen, zu diskutieren, darum ging es mir, aufgrund der anschaulichen Annahme der konstanten Vierergeschwindigkeit.
Man landet auch dann bei Längen von Weltlinien, nicht bei Beschleunigungen.

Man sollte m.E. zur Erklärung eines physikalischen Sachverhalts die Begriffe verwenden, die minimal notwendig und mathematisch relevant sind.

TomS
23.09.15, 08:12
Es gibt anschauliche Darstellungen des Zwillingsparadoxon, die argumentieren, dass mindestens ein Zwilling (kurzzeitig) beschleunigt sein muss, während es der andere nicht muss ... Wenn man nun sagt, dass eine Richtungsänderung eine Beschleunigung ist, dann finde ich es nicht so abwegig, dies auch so zu nennen.
Das darf man so nennen.

Ich bin jedoch der Meinung - und das habe ich auch mehrfach geschrieben - dass diese Darstellung zwar möglicherweise anschaulich und sicher verbreitet, dadurch jedoch nicht automatisch gut ist. Ich halte meine Argumentation für im Kern präziser. Und wenn man sich Zeit nimmt, darüber nachzudenken, ist sie auch nicht komplizierter. Sie wird nur dadurch verkompliziert, dass oft reflexartig Beschleunigungen, Inertialsysteme u.ä. genannt werden, was ich für didaktisch fragwürdig halte.

Andererseits ist der Begriff Beschleunigung vielleicht im Rahmen der SRT / ART zu sehr vorbelastet. Aus dieser Richtung komme ich jedoch nicht.
Das war bisher auch nicht mein Hauptargument

Es ist jedoch tatsächlich so, dass meine Argumentation sofort auf die ART verallgemeinert werden kann; man muss praktisch nichts Neues verstehen und keine einzige wirklich neue Gleichung hinschreiben; siehe auch mein verlinkter Beitrag. Wenn man dagegen mit Beschleunigung argumentiert, dann muss man vor der Diskussion der Zeitdilatation im Rahmen der ART einen vollständigen Mind-Reset durchführen und von vorne beginnen; warum tut man das den Lernenden an, obwohl eine im Kern präzisere Darstellung existiert, die auch nicht komplizierter ist?)

Hawkwind
23.09.15, 09:36
Finde ich nur teilweise.

M.E. sollte die Argumentation nicht auf Inertialsysteme fokussieren (diese sind nur Spezialfälle) und insbs. nicht mittels Lorentztransformationen argumentieren.

Ich finde es meistens didaktisch sinnvoll, mit dem speziellen und überschaubareren Fall zu beginnen und danach erst die Allgemeinheit zu erschließen. Aus der Diskussion des Zwillingsparadoxons würde ich so auch die ART heraushalten - sie wird ja nicht benötigt. Im Kontext der SRT macht es viel Sinn, einen Beobachter in einem Inertialsystem zu platzieren - warum auch nicht?

Ich
23.09.15, 09:56
Ich war nur etwas "schockiert", als du das Wort katastrophal verwendest, nachdem ich überhaupt Beschleunignung in den Mund genommen hatte. Ich vermute, das liegt daran, dass es zu dem Thema - mir gar nicht bekannt - schon früher schon Diskussionen gab.
Das hast du falsch in Erinnerrung. Als "didaktisch katastrophal" habe ich die "Interpretation der Vierergeschwindigkeit" aus Wikipedia bezeichnet, die du verwendet hast. Da geht's nicht um Beschleunigung, sondern um "Geschwindigkeit durch den Raum" und "Geschwindigkeit durch die Zeit" und solche Dinge. Und ja, dazu gab's frühere Diskussionen, siehe hier (http://www.astronews.com/forum/showthread.php?7028-Was-ist-Zeit&p=98005#post98005)und die Links darin.
Hat übrigens nichts mit dir zu tun, ich habe diese Veranschaulichung früher auch verwendet. Empfinde sie aber mittlerweile eben als "didaktisch katastrophal".
Ich kann ohne die Randbedingungen: Gemeinsamer Startpunkt, Gemeinsame Geschwindigkeit beim Start und beim Wiedersehen = 0 sowie eine betragsmäßig maximal mögliche Beschleunigung (wie in der Technik oft vorhanden) in der Tat keine allgemeine gültige Aussage treffen, die meine "Beschleunigungsintegral" - Argumentation untermauert. Wenn diese Bedingungen vorhanden sind, dann denke ich, dass der Satz von Feldbaum anwendbar ist (dann sind es Rechtecke, also die maximale Fläche unter dem Integral und alles andere ist vom Betrag her flächenmäßig weniger).Die Randbedingungen darfst du gerne verwenden, bis auf die begrenzte Beschleunigung, die wirkt m.E. künstlich und hat nichts mit dem Paradoxon zu tun.
Aber selbst wenn du sie verwendest, kriegst du nur raus, dass die Kurve mit der größten Zeitdilatation auch maximalen integrierten Beschleunigungsbetrag hat. Schon der Umkehrschluss ist aber falsch, und für alle Kurven außer dieser einen gibt es gar keinen Zusammenhang mehr. Du kannst wirklich einfache Gegenbeispiele finden.
Es gibt anschauliche Formulierungen / Darstellungen des Zwillingsparadoxon, die argumentieren, dass mindestens ein Zwilling (kurzzeitig) beschleunigt sein muss, während es der andere nicht muss, weil eben ein Zwilling eine Richtungsänderung durchführt. Wenn man nun sagt, dass eine Richtungsänderung eine Beschleunigung ist, dann finde ich es nicht so abwegig, dies auch so zu nennen. Andererseits ist der Begriff Beschleunigung vielleicht im Rahmen der SRT / ART zu sehr vorbelastet.Nein, das ist alles schön und gut. Damit kann man die Asymmetrie begründen, deren Nichtberücksichtigung zu einem Paradoxon führen würde. Aber mehr nicht. Das habe ich dir schon mal (http://www.quanten.de/forum/showpost.php5?p=78616&postcount=126) geschrieben.

TomS
23.09.15, 11:01
Ich finde es meistens didaktisch sinnvoll, mit dem speziellen und überschaubareren Fall zu beginnen und danach erst die Allgemeinheit zu erschließen.
Das tue ich auch, allerdings ohne verfehlten Fokus auf Inertialsysteme und ganz ohne Lorentztransformationen.

Aus der Diskussion des Zwillingsparadoxons würde ich so auch die ART heraushalten - sie wird ja nicht benötigt.
Das tue ich auch (bis auf einen kleinen Hinweis).

Im Kontext der SRT macht es viel Sinn, einen Beobachter in einem Inertialsystem zu platzieren - warum auch nicht?
Weil es auch ohne diese Einschränkung genauso verständlich erklärbar ist und weil man dadurch Missverständnisse vermeidet. Die Einschränkung auf Inertialsysteme muss man insbs. immer dann vornehmen, wenn man zuvor mittels dieser blödsinnigen Lorentz-Transformation argumentiert hat.

Meine Strategie ist ganz simpel:
1) Eigenzeiten entsprechen Längen von Weltlinien;
2) das "Zwillingsparadoxon" resultiert aus zwei unterschiedlich langen Weltlinien;
3) zum anschaulichen Diskutieren und Vergleichen von (Längen von) Weltlinien können diese irgendwie geartet sein; ich benötige keine Geraden;
4) für eine einfache Beispielrechnung führe ich dann evtl. einen Beobachter in einem Inertialsystem ein; ich zeige aber auch, wie das für zwei nicht-inertiale (z.B. kreisförmig bewegte) Beobachter funktioniert

Es geht nur darum, klarzumachen, dass (4) ein Spezialfall ist; die Schritte (1 - 3) funktionieren aber genausogut ohne diese Einschränkung. Wer nur bis (3) liest, kann völlig beruhig abbrechen und ist trotzdem nicht auf der falschen Spur.

Ist das falsch?

Der einzige Nachteil dieser Vorgehensweise wäre - wenn alle Darstellungen sich daran halten würden - dass es keine Threads wie diesen gäbe ...

Hawkwind
23.09.15, 12:16
4) für eine einfache Beispielrechnung führe ich dann evtl. einen Beobachter in einem Inertialsystem ein; ich zeige aber auch, wie das für zwei nicht-inertiale (z.B. kreisförmig bewegte) Beobachter funktioniert

Es geht nur darum, klarzumachen, dass (4) ein Spezialfall ist; die Schritte (1 - 3) funktionieren aber genausogut ohne diese Einschränkung. Wer nur bis (3) liest, kann völlig beruhig abbrechen und ist trotzdem nicht auf der falschen Spur.

I

Da haben wir uns missverstanden: (4) ist kein Spezialfall. ich führe einen inertialen Beobachter ein, in dessen Ruhesystem ich die Eigenzeitintegrale der beiden nichtinertialen Reisenden berechne. Das ist ja jetzt kein Verlust an Allgemeinheit, oder?

Die Argumentation mit den Längen der Weltlinien finde ich persönlich ja auch sehr schön und elegant. Sie ist aber irgendwie schon etwas "abgehoben" für einen Anfänger, finde ich. Z.B. muss man sich immer vor Augen halten, dass dies Linien im 4-dim. Raum mit Minkowski-Metrik sind, wo z.B. erstaunlicherweise die Geraden zwischen 2 Ereignissen (die inertial Reisende ja zurücklegen) am längsten sind; deshalb altern sie ja mehr.

Unter einer Beschleunigung andererseits kann sich jeder etwas vorstellen; Weltlinien in 4-dim. nicht-euklidischen Räumen sind schon ein anderes Kaliber.
Deshalb finde ich die Argumentation über asymmetrische Beschleunigung der beiden Reisenden (damit eben der Effekt relativistischer Geschwindigkeiten sich "manifestiert") auch nicht so verkehrt.

TomS
23.09.15, 12:33
Da haben wir uns missverstanden: (4) ist kein Spezialfall. ich führe einen inertialen Beobachter ein, in dessen Ruhesystem ich die Eigenzeitintegrale der beiden nichtinertialen Reisenden berechne. Das ist ja jetzt kein Verlust an Allgemeinheit, oder?
OK, da haben wir uns in der Tat missverstanden.

Ja, zur Berechnung führt man irgendein Koordinatensystem ein; und praktischerweise ein Inertialsystem. Das ist kein Verlust an Allgemeinheit.

Ich bin nur vorsichtig in meiner Argumentation und sage, dass ich in Worten - nicht in Formeln - alles erklären kann, ohne ein Inertialsystem einführen zu müssen.

Die Argumentation mit den Längen der Weltlinien finde ich persönlich ja auch sehr schön und elegant. Sie ist aber irgendwie schon etwas "abgehoben" für einen Anfänger, finde ich. Z.B. muss man sich immer vor Augen halten, dass dies Linien im 4-dim. Raum mit Minkowski-Metrik sind, wo z.B. erstaunlicherweise die Geraden zwischen 2 Ereignissen (die inertial Reisende ja zurücklegen) am längsten sind; deshalb altern sie ja mehr.
Das kann sein, obwohl ich das persönlich nicht glaube.

Das Problem ist, dass ich meinen Ansatz nie wirklich durchexerzieren kann, weil jeder schon irgendwie was weiß, oder meint, was zu wissen. Man muss immer erst mal mit irgendwelchen falschen Vorstellungen aufräumen.

TheoC
23.09.15, 13:03
Ich hab das Gedankenexperiment mit dem kreisförmig fliegenden Zwilling erweitert um ein mächtiges schwarzes Loch in der Mitte.

Ein Zwilling, ausserhalb des SR, aber schon im Einfluss der Graviation des SL "steht" an einem fixen Punkt (unser SL rotiert nicht), muss also, um an diesen "Fixpunkt" zu bleiben, ständig beschleunigen um nicht in das SL zu fallen.
Der andere Zwilling fliegt mit hoher Geschwindigkeit um das SL herum, wobei die Fliehkraft exakt gleich ist wie die Anziehungskraft.

Der "stehende" Zwilling ist somit ständig beschleunigt (kann er messen durch die Kraft mit der es ihm an die Wand drückt), der rotierende nicht (ist schwerelos), zumindest nicht nach der ersten Runde, nachdem er einmalig beschleunigt worden ist.

Der rotierende Zwilling trifft zyklisch den stehenden Zwilling, und durch Uhrenvergleich (wenn die Zwillinge nach einer Umdrehung des rotierenden Zwillings wieder am selben Punkt sind) kann jetzt leicht festgestellt werden, welche Uhr langsamer geht.

==> der schnell fliegende aber unbeschleunigte Zwilling alter weniger schnell.

Ist das korrekt?

lg
Theo

Plankton
23.09.15, 15:02
Spannend jetzt. :D

TomS
23.09.15, 15:38
Wie hast du den Zwilling auf einem Orbit angesetzt? a) auf einer Geodäte (kein Kreis oder Ellipse), oder b) auf einem exakten Kreis?

Für (b) must du die Periodendauer explizit berechnen; für (a) fällt sie bei der Lösung der Geodätengleichung mit ab.

Am einfachsten verwendest du zunächst einen Spezialfall, nämlich den stabilen, geodätischen und zugleich kreisförmigen Orbit beim 3-fachen des Schwarzschildradius r = 3RS, d.h. speziell (a = b).

In dem von mir verlinkten Beitrag steht eine Formel zu (b)

Außerdem siehe hier:

https://en.wikipedia.org/wiki/Schwarzschild_geodesics

TheoC
23.09.15, 18:05
Hi TomS

ich denke es ist eine Geodäte, auf der unser Zwilling fliegt, gleich einem um die Erde kreisenden Satelliten.

Eigentlich interessiert mich nur, ob mein Gedankenmodell stimmt, und wir hier einen Fall haben eines nicht beschleunigten, schnellen Zwillings, und eines permanent beschleunigten langsamen Zwilling, bei welchem der langsame, aber permanent beschleunigte Zwilling schneller altert.

lg + Danke
Theo

TomS
23.09.15, 19:05
Hi TomS

ich denke es ist eine Geodäte, auf der unser Zwilling fliegt, gleich einem um die Erde kreisenden Satelliten.

Eigentlich interessiert mich nur, ob mein Gedankenmodell stimmt, und wir hier einen Fall haben eines nicht beschleunigten, schnellen Zwillings, und eines permanent beschleunigten langsamen Zwilling, bei welchem der langsame, aber permanent beschleunigte Zwilling schneller altert.
OK

Ich hatte nur gerade die einfache Rechnung zu einer Kreisbahn vor Augen, die nicht unbedingt einer Geodäten entspricht. Es gilt

T^2(r,v) = [f(r) - v^2] t^2

T: Eigenzeit
t: Koordinatenzeit

Für großen Radius r geht f(r) gegen 1, d.h. wir erhalten das Ergebnis der SRT für flache Raumzeit.

Offensichtlich nimmt T mit wachsendem v ab. Außer dass wir hier ein f(r) statt einer 1 haben, sieht die Formel praktisch so aus wie in der ART. Die Schlussfolgerungen sind ähnlich. Die für beide identische Raumzeit-Geometrie, kodiert in f(r), beeinflusst beide gleich; der Unterschied in den Eigenzeiten resultiert ausschließlich aus v. Der Langsame altert also schneller, weil er langsamer ist. Dabei ist es auch gleichgültig, ob eine Geodäte vorliegt oder nicht.

Ich würde nicht mit "Beschleunigung", "Fliehkraft" usw. argumentieren.

Das Ergebnis ist ein Spezialfall und gilt so nicht für beliebige Raumzeiten. Insbs. kann man i.A. die Effekte der Raumzeit, hier f(r) sowie der Geschwindigkeit v nicht sauber trennen.

Dedi
25.11.17, 14:23
Wenn zwei Beobachter die sich gegenseitig beobachten solange sie nur innerhalb einer gleichbleibenden Relativgeschwindigkeit ohne Beschleunigung, Kurven oder Gravitation zueinander stehen das gleiche sehen, dann möchte ich dieses mit folgenden Versuchsaufbau durchgehen.

Zwei Punkte im Weltraum die ohne Bewegung zueinander stehen einer als Startpunkt der andere als Ziel, alles ohne Gravitation.
Die Geschwindigkeit zwischen Start und Ziel ist jeweils konstant, Beschleunigungen erfolgen außerhalb der Beobachtungsstrecke.
Es gibt ein Zwillingspaar, jeder erhält eine Uhr, einen Sender für die Zeit und einen Empfänger um den jeweils anderen zu beobachten.
Einer verbleibt am Start der andere bewegt sich mittels einer Rakete Richtung Ziel.
Die Beobachtung beider Zwillinge beginnt bei Start und sie wird am Ziel unterbrochen, während der Beobachtungszeit darf jeder Zwilling behaupten jeweils das Gleiche zu beobachten.
Die Beobachtung für den Rückweg beginnt nun für beide wenn die Rakete das Ziel passiert und endet am Start.
Es wird etwas dauern bis das erste Signal empfangen wird, die Beobachtung beider Zwillinge ist wiederum jeweils die gleiche wie die des anderen Zwillings.
Zwischen Hin und Rückreise werden nicht die gleichen Beobachtungen gemacht, jeder Zwilling wird aber das beobachten was auch der andere Zwilling beobachtet.
Wenn sie sich am Start wieder treffen, können beide behaupten das die Alterung des jeweils anderen, innerhalb Start zum Ziel und zurück zum Start, die Gleiche ist, damit sind beide innerhalb der Beobachtungsstrecke die die Rakete zurückgelegt hat gleich gealtert.

Marco Polo
26.11.17, 01:33
Wenn sie sich am Start wieder treffen, können beide behaupten das die Alterung des jeweils anderen, innerhalb Start zum Ziel und zurück zum Start, die Gleiche ist, damit sind beide innerhalb der Beobachtungsstrecke die die Rakete zurückgelegt hat gleich gealtert.

Nein.

Hier dein Denkfehler:

Die Geschwindigkeit zwischen Start und Ziel ist jeweils konstant, Beschleunigungen erfolgen außerhalb der Beobachtungsstrecke.

Du kannst die Beschleunigungen aber nicht ausserhalb der Beobachtungsstrecke legen, weil diese zum Gedankenbeispiel dazu gehören und die Umkehr des Reisezwillings erst ermöglichen.

Die als Folge des Inertialsystemwechels später gemessene Eigenzeitdifferenz, kann man hier (http://homepage.univie.ac.at/franz.embacher/SRT/Zwillingsparadoxon.html) sehr schön und anschaulich in einem Raumzeitdiagramm ablesen.

Und da Beschleunigungen in der SRT nun mal nicht relativ sind, ist klar, wer derjenige ist, der beschleunigt ist. Nämlich der, der die dabei auftretenden Trägheitskräfte zu spüren bekommt, indem er in den Sitz oder Gurt gepresst wird.

TomS
26.11.17, 07:21
Dedi, ich denke, es geht dir darum, eine absolut symmetrische Situation herbeizuführen, dabei aus Sicht eines Zwillings dem jeweils anderen eine andere Zeit zuzuschreiben, und daraus einen Widerspruch zu konstruieren.

Denk dir zwei Zwillinge, die sich beide auf einer kreisförmigen Bahn mit jewels konstanter Geschwindigkeit bewegen. Die beiden Kreise ergeben zusammen eine Acht, gemeinsamer Start- und Zielpunkt der Reise ist im Berührpunkt der Acht.

Man kann diesen Fall exakt berechnen. Da absolute Symmetrie vorliegt, haben beide Zwillinge am Ziel der Reise das selbe Alter; ihre mitgeführten Uhren zeigen die selbe Zeit.

Marco Polo
26.11.17, 08:18
Da absolute Symmetrie vorliegt, haben beide Zwillinge am Ziel der Reise das selbe Alter; ihre mitgeführten Uhren zeigen die selbe Zeit.

Im Grunde ist noch nicht mal diese "absolute" Symmetrie gefordert.

Sind beide Weltlinien gleich lang, ergibt sich keine Eigenzeitdifferenz, egal wie sich deren Verlauf im Einzelnen darstellt.

Dedi
26.11.17, 11:58
Ich bekomme das Problem das die Zwillinge jeweils behaupten dürfen das gleiche zu beobachten nicht weg.
Wenn ich mein Experiment nur für den Rückflug betrachte, einen dritten Beobachter dazunehme der in Ruhe zum Experiment steht wird er eindeutig sehen welcher Zwilling sich bewegt.
Während der Beschleunigung vor und nach der Beobachtungstrecke werden die Zwillinge nicht das gleiche sehen, deswegen hab ich dies aus der Beobachtung herausgenommen.
Das Experiment kann mit unterschiedlichen Entfernungen durchgeführt werden, wobei die Beschleunigungen die gleich bleiben, der Fehler durch die Beschleunigungen bleibt dann jeweils der selbe.
Es erfolgt eine Zeitdilation die innerhalb der Strecke erfolgt die beide Zwillinge beobachten, der dritte Beobachter kann hier eindeutig sagen welcher Zwilling sich bewegt.
Was aber sehen die Zwillinge untereinander wenn beide sich als in Ruhe betrachten dürfen?

Marco Polo
26.11.17, 15:53
Wenn ich mein Experiment nur für den Rückflug betrachte, einen dritten Beobachter dazunehme der in Ruhe zum Experiment steht wird er eindeutig sehen welcher Zwilling sich bewegt.

Das kannst du zwar machen, nur ist das dann leider nicht mehr das Zwillingsparadoxon.

Da ändern auch noch so viele Beobachter nichts daran.

Raumzeitdiagramme aufzeichnen und dann wird das unmittelbar klar.

Hier (https://www.youtube.com/watch?v=wvvngeHEq2M)ein aktuelles Video dazu.

Ich
27.11.17, 10:42
Das Experiment kann mit unterschiedlichen Entfernungen durchgeführt werden, wobei die Beschleunigungen die gleich bleiben, der Fehler durch die Beschleunigungen bleibt dann jeweils der selbe.Der "Effekt" gleicher Beschleunigungen ist proportional zur Distanz, die zwischen den Beschleunigungen zurückgelegt wurde. Du denkst zu mechanistisch, es ist nicht so, dass die Beschleunigung jemanden altern lässt oder so. Zeitdilatation lässt sich nur geometrisch verstehen, als unterschiedliche Streckenlängen in der Raumzeit. Deswegen werden dir auch immer Raumzeitdiagramme ans Herz gelegt.

Dedi
28.11.17, 09:53
Und auch mit
http://homepage.univie.ac.at/franz.embacher/SRT/Zwillingsparadoxon.html
bleiben meine Probleme.
Durch den Wechsel der Inertialsysteme entsteht zwar ein schwarzer Fleck, andereseits ist zwischen dem Richtungswechsel und mag es auch noch so kurz sein ein bewegungsloser Zustand und während dieser Zeit trifft es mitten ins schwarze.
Es handelt sich dann um drei verschiedene Inertialsysteme in denen jeweils der Bruder und die Schwester behaupten das Gleiche zu sehen.
Auch sollte gelten das die Schwester sich als ruhend betrachten darf, dazu kann
das Dreieck einmal nach links gespiegelt werden, sie kann dann von der ruhenden Mitte aus ihren Bruder zur Linken betrachten, wie der Bruder aus der Ruhe die Schwester zur Rechten betrachten kann.

Ich
28.11.17, 10:50
Auch sollte gelten das die Schwester sich als ruhend betrachten darf, dazu kann
das Dreieck einmal nach links gespiegelt werden, sie kann dann von der ruhenden Mitte aus ihren Bruder zur Linken betrachten, wie der Bruder aus der Ruhe die Schwester zur Rechten betrachten kann.Nein, Spiegeln - so wie du es verstehst - ist keine erlaubte Transformation. Wenn du in ein anderes Bezugssystem wechseln willst, dann enspricht das am ehesten einer Scherung oder Rotation, so wie in den letzten beiden Bildern deines Links. Wichtig ist: Egal, in welches Bezugssystem man wechselt, der Knick bleibt immer in derselben Weltlinie. Und die Weltlinie mit dem Knick ist die mit der geringeren verstrichenen Eigenzeit.

Dedi
28.11.17, 12:03
Mit aufbiegen der Winkel auf +X hab ich keine brauchbre Lösung gefunden, mit
–X hab ich dann diese Spiegelung, der Wegstrecke bleibt gleich nur die Richtung ist gespiegelt, die beiden sehen sich dann von der Ruheposition in gegenüberliegende Richtung entfernen.
Es geht auch mehr um das was beide wahrnehmen, und das unterscheidet sich innerhalb eines Inertialsystem nicht.
Mit hin, stop und wieder zurück sind es drei Inertialsysteme innerhalb derer gleiche Beobachtungen gemacht werden und der schwarze Bereich welcher die Erlärung für den Zeitunterschied geliefert hat ist mit warten auch entfernt.
Mein Hauptproblem ist das das bewegte System als bewegt erkannt werden darf.
Die Reise kann in drei Teile unterteilt werden, für jeden Abschnitt einzeln betrachtet darf der Ruhende gewechselt werden, nur als ganzes geht es nicht mehr.
Dann könnte man aber immer argumentieren das ein bewegtes System sich anders verhält als das Ruhende auch das könnte gerade beschleunigt sein und hat deshalb eine vom Ruhendem abweichende Weltlinie, woran soll ich das jetzt unterscheiden?

Ich
28.11.17, 12:27
Die Reise kann in drei Teile unterteilt werden, für jeden Abschnitt einzeln betrachtet darf der Ruhende gewechselt werden, nur als ganzes geht es nicht mehr.
Dann könnte man aber immer argumentieren das ein bewegtes System sich anders verhält als das Ruhende auch das könnte gerade beschleunigt sein und hat deshalb eine vom Ruhendem abweichende Weltlinie, woran soll ich das jetzt unterscheiden?Ich kapiere dein Problem nicht. Du malst ein Dreieck auf ein Blatt Papier. Wenn eine Linie nach oben zeigt, dann sei der entsprechendende Beobachter "in Ruhe". Ich kann das Papier beliebig drehen, um jede beliebige Line nach oben zeigen zu lassen. Das Dreieck ändert sich darurch aber kein bisschen.
Und ja, natürlich unterscheidet sich die Linie mit Knick drin von der ohne Knick. Es hat auch nie jemand etwas anderes behauptet. "Gerade gleichförmige Bewegung" ist ununterscheidbar von "in Ruhe", nicht jede beliebige Bewegung.
Eine Linie mit Knick drin ist keine gerade Linie und repräsentiert auch keine "gerade gleichförmige Bewegung". Du kannst sie also nicht als "in Ruhe" ansehen und zu einer Linie ohne Knick machen, egal wie viel du das Papier drehst bzw. das Bezugssystem wechselst.

Dedi
28.11.17, 12:46
Das Problem ergiebt sich wenn ich weiß wer sich bewegt, dem bewegten eine Lichtuhr mitgebe nicht mehr weiß wie ich unter Bewegung die Lichtrahlen in der senkrechten halten soll.
Ich kann der Weltline im Zwillingsparadoxon folgen, auch das ein Inertialsystem jeweils in Ruhe betrachtet werden darf, nur zusammen betrachtet ergiebt sich ein Problem.

Ich
28.11.17, 13:04
Das Problem ergiebt sich wenn ich weiß wer sich bewegt, dem bewegten eine Lichtuhr mitgebe nicht mehr weiß wie ich unter Bewegung die Lichtrahlen in der senkrechten halten soll.Da es keine absolute Bewegung gibt, gibt es auch keine absolute Senkrechte auf die Bewegungsrichtung. Ergo ist es auch absolut egal, wie du die Lichtuhr hältst.
Du kannst ein Bezugssystem finden, in dem die Uhr senkrecht zur Bewegungsrichtung steht, und ein anderes, wo sie parallel steht. Das darf also keinen Unterschied machen.
Ich kann der Weltline im Zwillingsparadoxon folgen, auch das ein Inertialsystem jeweils in Ruhe betrachtet werden darf, nur zusammen betrachtet ergiebt sich ein Problem.Welches denn?

Dedi
28.11.17, 13:18
Welches denn?

Bei bewegt, die mitbewegte Lichtuhr und den dazu ruhenden Lichtstrahl, das bekomme ich nicht auseinander.

Ich
28.11.17, 16:32
Was? Dass der Lichtstrahl im Zickzack geht und nicht einfach auf und ab?

Dedi
29.11.17, 11:55
Was? Dass der Lichtstrahl im Zickzack geht und nicht einfach auf und ab?

War ein Verständnißproblem, der Lichtstrahl einer Lichtuhr geht aus der Sicht der Lichtuhr oder eines dazu ruhenden Beobachters immer senkrecht zwischen den Spiegeln, solange sie ohne Beschleunigung gleichmäßig bewegt ist.

Ich
29.11.17, 12:21
Und was ist da sonst noch ein Problem?

Dedi
29.11.17, 12:43
Und was ist da sonst noch ein Problem?

Momentan hab ich das Problem ein bewegtes System auch als ruhend zu betrachten.
Betrachte ich nur die Hinreise zum Ziel, die Reisedauer nehm ich die vier Jahre aus dem Video, also 5 Jahre im Ruhesystem.
Werden Lichtsignale im Abstand von einem Jahr vom daheimgebliebenen Zwilling und auch vom reisenden Zwilling ausgesandt kommen diese beim jeweils anderen nach zwei Jahren an, sie sehen hier beide das Gleiche.
Betrachte ich das mit dem Ruhesystem des Ziels, es werden auch hier jeweils nach einem Jahr Lichtsignale ausgesendet, bekommt der Reisende fünf Lichtsignale wobei er selber nur 4 ausgesandt hat, das sieht nicht gleich aus.

Ich
29.11.17, 13:07
Momentan hab ich das Problem ein bewegtes System auch als ruhend zu betrachten.
Betrachte ich nur die Hinreise zum Ziel, die Reisedauer nehm ich die vier Jahre aus dem Video, also 5 Jahre im Ruhesystem.
Werden Lichtsignale im Abstand von einem Jahr vom daheimgebliebenen Zwilling und auch vom reisenden Zwilling ausgesandt kommen diese beim jeweils anderen nach zwei Jahren an, sie sehen hier beide das Gleiche.Ich hab das Video nicht angeschaut, kann mir das Gedankenexperiment aber vorstellen. Scheint so weit zu stimmen.
Betrachte ich das mit dem Ruhesystem des Ziels, es werden auch hier jeweils nach einem Jahr Lichtsignale ausgesendet, bekommt der Reisende fünf Lichtsignale wobei er selber nur 4 ausgesandt hat, das sieht nicht gleich aus.Verstehe ich nicht. Das Ruhesystem des Ziels ist dasselbe, in dem auch der Startpunkt in Ruhe ist. Insgesamt bekommt der Reisende 5 Signale, der Verbleibende 4, egal welches Bezugssystem man wählt. Das ist natürlich nicht gleich, wieso sollte es das sein?

Dedi
29.11.17, 14:24
Wenn der Reisende durch aufbiegen der Winkel zum Ruhenden wird und das Ziel zum Reisenden kann der jetzt bewegte nur noch vier Signale senden dafür der jetzt ruhende fünf.
Die Anzahl der Lichtsignale die man aussenden kann sollte sich aber nicht ändern.

Ich
29.11.17, 14:34
Wenn der Reisende durch aufbiegen der Winkel zum Ruhenden wird und das Ziel zum Reisenden kann der jetzt bewegte nur noch vier Signale senden dafür der jetzt ruhende fünf.
Die Anzahl der Lichtsignale die man aussenden kann sollte sich aber nicht ändern.Und was für eine Operation soll das "Aufbiegen der Winkel" bitte sein? Der Wechsel in ein anderes Inertialsystem?

Dedi
29.11.17, 15:19
Und was für eine Operation soll das "Aufbiegen der Winkel" bitte sein? Der Wechsel in ein anderes Inertialsystem?

https://m.youtube.com/watch?v=wvvngeHEq2M
nach ca. 4 min im Video wird das aufbiegen von Winkeln für den Wechsel zwischen Ruhe und zum bewegten System erklärt.
Man betrachtet eine Situation einmal als sich ruhend und einmal als sich bewegend, das was man selber sieht sollte sich dadurch nicht ändern wenn man sich immer als ruhend betrachten darf.

Ich
29.11.17, 15:31
Auweh, ich hasse diese 2-in-1-Diagramme. Sorgen immer für Verwirrung.
Dir ist aber schon klar, dass er nur neue x'- und t'-Achsen reinmalt und das Dreieck davon vollkommen unberührt ist? Es gibt nur ein Dreieck, und an dem ändert sich überhaupt nichts. Er kann da noch zwanzig neue Achen reinmalen, das Dreieck ist und bleibt dasselbe, und die Linie mit Knick wird nicht geradegebügelt und die Linie ohne Knick wird nicht geknickt. Und die ganzen Lichtlinien bleiben auch gleich, alles bleibt gleich.

Dedi
29.11.17, 15:48
Auweh, ich hasse diese 2-in-1-Diagramme. Sorgen immer für Verwirrung.
Dir ist aber schon klar, dass er nur neue x'- und t'-Achsen reinmalt und das Dreieck davon vollkommen unberührt ist? Es gibt nur ein Dreieck, und an dem ändert sich überhaupt nichts. Er kann da noch zwanzig neue Achen reinmalen, das Dreieck ist und bleibt dasselbe, und die Linie mit Knick wird nicht geradegebügelt und die Linie ohne Knick wird nicht geknickt. Und die ganzen Lichtlinien bleiben auch gleich, alles bleibt gleich.

Ich bin nur bei einer Hinreise, da ist kein Knick, aber schon der Widerspruch mit vier oder fünf Jahren.

JoAx
29.11.17, 21:49
Ich bin nur bei einer Hinreise, da ist kein Knick, aber schon der Widerspruch mit vier oder fünf Jahren.

Der Knick mag noch nicht da zu sein, aber das Ereignis, wann dieser Knick passieren soll, ist da und bleibt auch da. Dieses Ereignis hat im IS K (die ganze Zeit ruhender Zwilling) die Koordinaten:

x = 0,6*5 = 3 Lichtjahre
t = 5 Jahre

Im IS K', in dem der reisende Zwilling bis zur Umkehr ruht, hat dieses Ereignis die Koordinaten:

x' = 0
t' = 4 Jahre

Daran ändert sich nichts. Egal, aus welchem IS du die Situation betrachtest.

Das Video ist nicht schlecht, aber weiter kommt man erst, wenn man anfängt Mathematik und die Diagramme zu nutzen; sprich - sie mit eigenen Händen zeichnen, aufschreiben. Probiere das aus. Fange an, wenigstens, die Diagramme zu zeichnen.

Dedi
30.11.17, 11:47
Der Knick mag noch nicht da zu sein, aber das Ereignis, wann dieser Knick passieren soll, ist da und bleibt auch da. Dieses Ereignis hat im IS K (die ganze Zeit ruhender Zwilling) die Koordinaten:

x = 0,6*5 = 3 Lichtjahre
t = 5 Jahre

Im IS K', in dem der reisende Zwilling bis zur Umkehr ruht, hat dieses Ereignis die Koordinaten:

x' = 0
t' = 4 Jahre

Daran ändert sich nichts. Egal, aus welchem IS du die Situation betrachtest.

Das Video ist nicht schlecht, aber weiter kommt man erst, wenn man anfängt Mathematik und die Diagramme zu nutzen; sprich - sie mit eigenen Händen zeichnen, aufschreiben. Probiere das aus. Fange an, wenigstens, die Diagramme zu zeichnen.
Ich hab angefangen zu zeichnen.
Es geht mir nicht darum das in dem bewegten System weniger Zeit vergeht, sondern darum das schon auf einen beliebigen Hinweg erkannt werden kann wer sich bewegt, unabhängig davon ob am Ziel die Reise zurückgeht, mit gleicher Geschwindigkeit fortgesetzt wird oder die Rakete explodiert.
Wie ist das mit dem Relativitätsprinzip vereinbar.

Timm
30.11.17, 15:04
Es geht mir nicht darum das in dem bewegten System weniger Zeit vergeht, sondern darum das schon auf einen beliebigen Hinweg erkannt werden kann wer sich bewegt, unabhängig davon ob am Ziel die Reise zurückgeht, mit gleicher Geschwindigkeit fortgesetzt wird oder die Rakete explodiert.
Wie ist das mit dem Relativitätsprinzip vereinbar.
Ich denke, der Knoten läßt sich leicht lösen, denn du hast das Relativitätsprinzip noch nicht verinnerlicht. Es besagt, daß auf dem Hinweg eben gerade nicht erkannt werden kann, wer sich bewegt. Bis zu diesem Punkt sind beide Zwillinge gleichberechtigt und jeder sieht den anderen relativ zu sich bewegt und langsamer altern.
Das sog. Paradoxon entsteht erst nach dem Rückweg mit dem Vergleich der Uhren.

In dem Video siehst du, daß aus der Sicht des Erdzwillings der Reisende langsamer altert. Nimmst du das System des Reisenden (im Video mit "Dach" bezeichnet) als ruhendes System, dann altert aus dessen Sicht der Erdzwilling langsamer. Es wäre eine gute Übung, das analog zu dem wie im Viedeo gezeigt, einzuzeichnen. Dann altert aus der Sicht des Reisenden der Erdzwilling langsamer. Und dann praktizierst du das Relativitätsprinzip auf einem Stück Papier, der beste Weg zum Verständnis.

Ich
30.11.17, 15:08
Ich hab angefangen zu zeichnen.
Es geht mir nicht darum das in dem bewegten System weniger Zeit vergeht, sondern darum das schon auf einen beliebigen Hinweg erkannt werden kann wer sich bewegt, unabhängig davon ob am Ziel die Reise zurückgeht, mit gleicher Geschwindigkeit fortgesetzt wird oder die Rakete explodiert.
Wie ist das mit dem Relativitätsprinzip vereinbar.Das stimmt doch gar nicht. Nach 5 Jahren im System A sind im System B 4 Jahre vergangen. Das heißt exakt: Das Ereignis t=5,x=0 ist im System A gleichzeitig zu dem Ereignis t'=4,x'=0. *
Symmetrie bedeutet: Das Ereignis t'=5,x=0 ist im System B gleichzeitig zu dem Ereignis t=4,x=0. Im Diagramm ist die Gleichzeitigkeitslinie von System B geneigt, nicht horizontal, deswegen funktioniert das auch. Versuch das mal zu zeichnen.

*die gestrichelten Koordinaten gelten im System B, die ungestrichelten im Ruhesystem A. System B hat die gekippten Achsen, System A die rechtwinkligen.
Tipp: Das Ereignis t'=5,x=0 hat im System A die Koordinaten t=6.25,x=3.75.

Dedi
30.11.17, 15:56
Ich vermisse die Symmetrie wenn der Reisende mit dem Ziel Lichtsignale austauscht, nur der Ruhende kann mehr Signale aussenden als der Reisende, der Reisende erhält mehr Lichtsignale als der Ruhende.

Ich
30.11.17, 16:30
Du wolltest nur den Hinweg anschauen. Dann mach das auch. Du darfst dann natürlich nicht bei A nach 5 Jahren aufhören und bei B schon nach 4 Jahren. Lass beide 5 Jahre lang reisen, oder beide 4 Jahre lang (jeweils Eigenzeit!). Siehe dazu meinen letzten Beitrag.

Dedi
30.11.17, 18:00
Dann mit dem Diagramm. Beide dürfen annehmen in Ruhe zu sein. Es würden beide die Möglichkeit erhalten 5 Signale zu senden wo der andere nur 4 senden kann. Soweit ist die Symmetrie gegeben.
Am Ziel angekommen könnten beide behaupten die 5 Signale gesendet und nur 4 empfangen zu haben, hier entsteht ein Widerspruch, nur einer kann recht haben.

Ich
30.11.17, 18:20
Am Ziel angekommen könnten beide behaupten die 5 Signale gesendet und nur 4 empfangen zu haben, hier entsteht ein Widerspruch, nur einer kann recht haben.Natürlich haben beide 5 Signale gesendet. Jeder meint aber, dass der andere das letzte Signal gesendet hat, nachdem er selbst sein letztes gesendet hatte.
Mal's dir halt auf.

Dedi
30.11.17, 18:53
Wenn ich es zeichne habe ich die Möglichkeit beide als Ruhend zu betrachten, das bestreite ich nicht.
Aber einer von den beiden, der der wirklich gereist ist, ist nur vier Jahre gealtert und kann auch nur vier Signale gesendet haben. Ruhender und Reisender unterliegen nicht der gleichen Alterung, wie auch im Zwillingsparadoxon einer Jünger als der andere ist.
Das Zwillingsparadoxon entsteht nicht erst durch die Umkehr, es ist schon in einer gewöhnlichen Hinreise auch ohne umzukehren erkennbar.

JoAx
30.11.17, 20:14
Aber einer von den beiden, der der wirklich gereist ist, ist nur vier Jahre gealtert und kann auch nur vier Signale gesendet haben. Ruhender und Reisender unterliegen nicht der gleichen Alterung, wie auch im Zwillingsparadoxon einer Jünger als der andere ist.
Das Zwillingsparadoxon entsteht nicht erst durch die Umkehr, es ist schon in einer gewöhnlichen Hinreise auch ohne umzukehren erkennbar.

Nein. Solange keine Umkehr stattfindet, können beide sich als ruhend betrachten und haben auch beide Recht.

Dass der "Reisende" weniger altert als der "Ruhende" liegt an der Relativität der Gleichzeitigkeit. Er altert nicht absolut langsamer, sondern nur aus der Sicht des "Ruhenden".

Und wie Ich es schon richtig gesagt hat, wenn du nur den Hinweg betrachtest, dann darfst du auch nicht weiter als t = t' = 4 Jahre (der Eigenzeit(en)) gehen.
Kleiner Tipp: beide Ereignisse liegen auf der Parabel des Zukunftskegels mit Radius = 4 LJ.

JoAx
30.11.17, 20:15
Dann mit dem Diagramm.

Zeige uns doch mal deine Diagramme. Mit Kommentaren und Formeln dazu.

Ich
30.11.17, 20:46
Ja, zeig ein Diagramm. Hinweis: wenn nicht von beiden gleich viele Signale gesendet wurden, ist es falsch.

Dedi
01.12.17, 12:16
Die Zeichnung sieht so aus wie im Video, hab diese ja nur übernommen.

https://picload.org/view/drigrlgi/0d117de8-cdda-4d57-bfce-d66eda.jpg.html

Links sind die Lichtstrahlen beim Entfernen von Start zum Ziel, beide senden einen Lichtstrahl pro Jahr aus und erhalten alle zwei Jahre einen. Beide Beobachter sehen genau das Gleiche.

Auf der rechten Seite die obere Zeichnung ist der Rückweg in etwa wie im Video, die Rückreise ist hier nur ohne Strahlen aus dem Hinweg dargestellt und kann als Eigenständige Reise auch ohne Hinweg betrachtet werden.Hier sehen beide Beobachter nicht das Gleiche, das wird auch erwartet bei einer unterschiedlichen Alterung.

Recht unten ist der Hinweg,auch dieser kann eigenständig betrachtet werden, es macht keinen Unterschied was anschließend passiert. Er sieht vom Zeitablauf so aus wir der Rückweg, beide Beobachter erleben nicht das Gleiche. Der Reisende verkürzt seine Entfernung zum Ziel und damit die Zeit die das Licht zu ihm braucht, das ist beim ruhenden Beobachter nicht der Fall.

Ich
01.12.17, 15:04
Auf der rechten Seite die obere Zeichnung ist der Rückweg in etwa wie im Video, die Rückreise ist hier nur ohne Strahlen aus dem Hinweg dargestellt und kann als Eigenständige Reise auch ohne Hinweg betrachtet werden.Hier sehen beide Beobachter nicht das Gleiche, das wird auch erwartet bei einer unterschiedlichen Alterung.

Recht unten ist der Hinweg,auch dieser kann eigenständig betrachtet werden, es macht keinen Unterschied was anschließend passiert. Er sieht vom Zeitablauf so aus wir der Rückweg, beide Beobachter erleben nicht das Gleiche. Der Reisende verkürzt seine Entfernung zum Ziel und damit die Zeit die das Licht zu ihm braucht, das ist beim ruhenden Beobachter nicht der Fall.Erst mal Danke für das Diagramm. Ich habe immer noch keine Ahnung, was das Problem ist, und warum du jetzt effektiv doch den Rückweg betrachtest statt dem Hinweg.
Wenn dein Problem ist, dass im Abschnitt rechts unten der eine vier Strahlen schickt und der andere 5: Das ist doch logisch, wenn der eine vier Jahre vorher anfängt und der andere 5 Jahre vorher. Die Situation wird dann symmetrisch, wenn du dem Reisenden noch das Sendeereignis t=-1.25, x=-0.75 dazugibst. Dann sind beide 5 Jahre unterwegs und schicken je 5 Signale.

Dedi
01.12.17, 15:37
Jetzt hab ich verstanden was mit hinzufügen gemeint ist, es sieht jetzt gleich aus.
Der Rückflug ist die gleiche Situation wie der Hinflug, hier ist kein Unterschied.
Jetzt hab ich beim Hinflug fünf Signale aber ich kann das auch für den Rückflug anwenden und hab dann auch dort fünf Signale. Die beiden Reisen untereinander sehen in der Grafik bis auf die Richtung gleich aus.
Mein Problem bleibt, entweder gleiche Alterung und Relativitätsprinzip oder unterschiedliches altern und kein Relativitätsprinzip.

Dedi
01.12.17, 15:57
https://picload.org/view/drigogoa/image.jpg.html

Noch Die Grafik dazu
Und ich habe zu danken für eure Geduld

JoAx
01.12.17, 16:54
Hier Vorschaubilder als solche eingebettet:

https://picload.org/thumbnail/drigrlgi/0d117de8-cdda-4d57-bfce-d66eda.jpg (https://picload.org/view/drigrlgi/0d117de8-cdda-4d57-bfce-d66eda.jpg.html)

https://picload.org/thumbnail/drigogoa/image.jpg (https://picload.org/view/drigogoa/image.jpg.html)

JoAx
01.12.17, 18:07
Mein Problem bleibt, entweder gleiche Alterung und Relativitätsprinzip oder unterschiedliches altern und kein Relativitätsprinzip.

Sowohl als auch.

Schau, auf dem Bild unten betrachte ich ganz normalen Euklidischen Raum:

https://www2.pic-upload.de/thumb/34398631/EuklRaum.png (https://www.pic-upload.de/view-34398631/EuklRaum.png.html)

Da habe ich zwei Bezugssystem K (x, y) und K' (x', y') eingezeichnet.
Ein Kreis: r^2 = dx^2+dy^2 = dx'^2+dy'^2
Keines dieser Bezugssysteme ist besser als das andere. Sie sind völlig gleichberechtigt. Stimmst du dem zu?

Ich hoffe, du hast mit "ja" geantwortet und gehe nun weiter. Die Strecken OA und OA' sind gleich lang.
Stimmt's?
Ihre Projektionen auf das jeweils andere Bezugssystem (OB bzw. OB') haben andere Längen. OA ≠ OB und OA' ≠ OB'. Aber unter einander sind die "B-Strecken" auch gleich. OB = OB'
Stimt das auch?

Wenn du bis hierhin alles bejat hast, dann musst du nur noch begreifen, dass das Relativitätsprinzip genau das Gleich behauptet, nur nicht für den euklidischen Raum, sondern für die pseudo-euklidische Raumzeit.

Das, was für den euklidischen Raum der Kreis ist, ist für die Raumzeit die
Hyperbel: ds^2 = dt^2 - dx^2 = dt'^2 - dx'^2

Ja, das ist die "Metrik".
Die 5 Jahre des "Ruhenden" sind eine Projektion der 4 Jahre des "Reisenden". Klingt komisch? Is' aber so.

Besorge dir am Besten das Buch:
Physik der Raumzeit: Eine Einführung in die spezielle Relativitätstheorie Gebundene Ausgabe – 1994
von Edwin F Taylor,‎ A John Wheeler

Da ist es wunderbar erklärt.

Dedi
01.12.17, 18:30
Begreifen wird wieder etwas dauern.
Die Gleichberechtigung unter den Bezugssystemen glaub ich ja, das ist wie mit dem Bahnsteig und dem Fahrenden Zug.
Die beiden Grafiken von mir kann man als einzelne aber auch als komplette Reise des Zwillingsparadoxon betrachten, sowohl mit 2 mal 4 Jahren als auch mit 2 mal 5.
Ich möchte die Gleichbehandlung auch im Zwillingsparadoxon, das Umkehren am Ziel ändert nichts an diesen Zeichnungen, beides ist möglich.

JoAx
01.12.17, 22:30
Ich möchte die Gleichbehandlung auch im Zwillingsparadoxon, das Umkehren am Ziel ändert nichts an diesen Zeichnungen, beides ist möglich.

Das geht so nicht. Wenn man deine Forderung/Erwartung auf den euklidischen Raum überträgt, dann würde sie lauten:

"Ob man von Punkt A zu Punkt B direkt geht, oder einen Umweg macht, die Länge des Weges bleibt immer gleich."

Dass diese Aussage falsch ist, das weißt du. Die Richtige lautet:

"Die kürzeste Verbindung zwischen zwei Punkten ist eine Gerade."

Und in der Raumzeit lautet die entsprechende Aussage:

"Die längste Verbindung zwischen zwei Ereignissen ist eine Gerade."

Um nichts anderes geht es beim Zwillingsparadoxon. Beide Zwillinge fangen ihre Reise im Ereignis A an und beenden diese im Ereignis B. Der ruhende geht auf direktem Wege, während der reisende einen Umweg macht.

Das kann nicht über die ganze Reisedauer symmetrisch sein.

Bernhard
02.12.17, 15:52
Ich möchte die Gleichbehandlung auch im Zwillingsparadoxon, das Umkehren am Ziel ändert nichts an diesen Zeichnungen, beides ist möglich.
Das geht prinzipiell nicht. Der Zwilling, der beschleunigt und abbremst, spürt die zugehörigen Trägheitskräfte. Das System des jüngeren Zwillings ist also nachweisbar ein anderes, als das System des älteren Zwillings. Trägheitskräfte kann man nicht wegdiskutieren.

Oder diskutiert ihr hier das Paradoxon mit drei Inertialsystemen?

Dedi
02.12.17, 20:33
Das geht prinzipiell nicht. Der Zwilling, der beschleunigt und abbremst, spürt die zugehörigen Trägheitskräfte. Das System des jüngeren Zwillings ist also nachweisbar ein anderes, als das System des älteren Zwillings. Trägheitskräfte kann man nicht wegdiskutieren.

Oder diskutiert ihr hier das Paradoxon mit drei Inertialsystemen?

Es ist anders, aber wer legt in einem System mit Relativgeschwindigkeiten fest wer sich wann wie bewegt hat, woher weiß ich das man durch Beschleunigung aus der Ruhe in Bewegung übergeht oder aus Bewegung in Ruhe.

Bernhard
02.12.17, 22:12
wer legt in einem System mit Relativgeschwindigkeiten fest wer sich wann wie bewegt hat
Der Beobachter. Eine Besonderheit der Relativitätstheorie ist die Notwendigkeit der Definition eines Beobachters.

woher weiß ich das man durch Beschleunigung aus der Ruhe in Bewegung übergeht oder aus Bewegung in Ruhe.
Beschleunigungen kann man wegen der Trägheit von Testkörpern messen. Nimm dazu eine Rakete ohne Fenster. Wenn eine frei bewegliche Masse lokal auf die Rakete eine Kraft ausübt, gibt es entweder eine Beschleunigung oder Gravitation, wobei ich Gravitation bei diesem Thema außen vor lassen würde.

Anders formuliert: Wenn es den oder die Raumfahrer in der Rakete in die Sitze presst, so beschleunigt die Rakete. Ohne diese Kraft ruht die Rakete oder sie bewegt sich mit konstanter Geschwindigkeit. Den Absolutwert der Geschwindigkeit bestimmt wieder ein Beobachter. Wenn der oder die Raumfahrer in der Rakete frei schweben, können diese sich selbst als ruhend betrachten.

Marco Polo
02.12.17, 22:24
Ich möchte die Gleichbehandlung auch im Zwillingsparadoxon, das Umkehren am Ziel ändert nichts an diesen Zeichnungen, beides ist möglich.

Das ist nicht korrekt.

Recht anschaulich finde ich die Betrachtung mittels Austausch von Lichtsignalen:

https://de.wikipedia.org/wiki/Zwillingsparadoxon#Austausch_von_Lichtsignalen

Bisher wurde dargestellt, was die Beobachter unter Berücksichtigung der ihnen bekannten Ausbreitungsgeschwindigkeit des Lichtes für das reale Geschehen halten. Im Folgenden sei beschrieben, was beide Zwillinge unmittelbar sehen, wenn sie einmal pro Jahr ein Lichtsignal zu ihrem Bruder senden. Die Wege von Lichtsignalen im obigen Weg-Zeit-Diagramm sind Geraden mit einem Anstieg von 45° in Senderichtung. Bezogen auf dieses Beispiel ergeben sich die nebenstehenden Diagramme. Zunächst bewegen sich die Zwillinge voneinander weg, sodass die Lichtstrahlen durch den Dopplereffekt (https://de.wikipedia.org/wiki/Dopplereffekt) rotverschoben sind. Diese Lichtstrahlen sind im Diagramm rot dargestellt. Nach der Hälfte der Reise bewegen sich die Zwillinge aufeinander zu, sodass die Lichtstrahlen blauverschoben werden, daher sind diese Lichtstrahlen im Bild blau dargestellt. Aufgrund des Relativitätsprinzips messen beide Beobachter die gleichen Zeitintervalle von jeweils 2 Jahren zwischen den roten Signalen, und jeweils einem halben Jahr zwischen den blauen Signalen, was im Bild sofort klar wird.
Damit führt die Annahme, beide Zwillinge wären nach der Rückkehr gleich alt, sodass beide Zwillinge gleich viele Signale vom anderen empfangen hätten, nun aber zu einem Widerspruch. Denn während der reisende Zwilling am Umkehrpunkt und damit nach der halben Reisezeit sofort die zeitlich komprimierten Signale erhält, erreichen den irdischen Zwilling die gedehnten Signale noch länger. Aufgrund des Relativitätsprinzips bekommt also der Beobachter, der für einen längeren Zeitraum blauverschobene Signale erhält, insgesamt mehr Signale als der andere. Der reisende Zwilling bekommt also mehr Signale als der Zwilling auf der Erde, sodass beide übereinstimmend feststellen, dass der reisende Zwilling langsamer gealtert ist.
Im Zahlenbeispiel des nebenstehenden Bildes sieht der reisende Zwilling, bedingt durch eine Kombination von relativistischen Effekten und Laufzeiteffekten, den irdischen zunächst in 4 Jahren um 2 Jahre altern und in weiteren 4 Jahren um 8 Jahre, insgesamt also um 10 Jahre. Der irdische Zwilling sieht entsprechend den Reisenden zunächst in 8 Jahren um 4 Jahre altern und anschließend in 2 Jahren um 4 Jahre, insgesamt also um 8 Jahre.

Dedi
02.12.17, 23:32
Der Beobachter. Eine Besonderheit der Relativitätstheorie ist die Notwendigkeit der Definition eines Beobachters.


Beschleunigungen kann man wegen der Trägheit von Testkörpern messen. Nimm dazu eine Rakete ohne Fenster. Wenn eine frei bewegliche Masse lokal auf die Rakete eine Kraft ausübt, gibt es entweder eine Beschleunigung oder Gravitation, wobei ich Gravitation bei diesem Thema außen vor lassen würde.

Anders formuliert: Wenn es den oder die Raumfahrer in der Rakete in die Sitze presst, so beschleunigt die Rakete. Ohne diese Kraft ruht die Rakete oder sie bewegt sich mit konstanter Geschwindigkeit. Den Absolutwert der Geschwindigkeit bestimmt wieder ein Beobachter. Wenn der oder die Raumfahrer in der Rakete frei schweben, können diese sich selbst als ruhend betrachten.

Eine Beschleunigung heißt nicht das ich eine Geschwindigkeit erhöhe.
Wenn von einer Bewegung die nach links geht nach recht auf null abgebremst wird ist es das Gleiche wie wenn ich aus der Ruhe nach rechts Beschleunige. Auch ein Beobachter kann nicht behaupten still zu stehen, alle Bewegungen sind relativ.
Beim Zwillingsparadoxon gibt es eine Beschleunigung am Start eine am Ziel und wieder eine am Start, damit weiß ich das der Reisende irgendwann in Bewegung sein muß, aber nicht wann und auch nicht ob und wann der eventuell Ruhenden in Bewegung ist. Alle Geschwindigkeiten sind nur relativ zueinander nie Absulut.

Bernhard
03.12.17, 07:46
Eine Beschleunigung heißt nicht das ich eine Geschwindigkeit erhöhe.
Es geht darum, dass eine Beschleunigung als Unterscheidungskriterium von Bezugssystemen verwendet werden kann. Wenn ich ein System mit und ein System ohne Beschleunigung habe, so sind diese experimentell voneinander unterscheidbar.

TomS
03.12.17, 09:59
Ich habe gerade jetzt wenig Zeit - evtl. heute Abend - aber ich möchte dennoch eine alternative Herangehensweise vorschlagen, die zunächst vollständig ohne jegliches Bezugsystem auskommt; dieses kommt für Berechnungen als mathematisches Werkzeug mit ins Spiel, hat jedoch keine fundamentale Bedeutung!

Möglicherweise hilft das beim Verständnis weiter.

Ich
03.12.17, 10:17
Du musst Beiträge #222 und #224 von JoAx lesen.

Du darfst es dir nicht so vorstellen, dass die eine Uhr langsamer geht als die andere. Das funktioniert nicht. Dann müsste nämlich eine die langsamste sein, es könnte keine dass Symmetrie geben, dass die jeweils andere langsamer geht, und man müsste wissen, welche Uhr sich wirklich bewegt.

Stell dir vor, dass die Zeit je nach Bewegungszustand in verschiedene (aber gleichwertige!) Richtungen vergeht. Das ist symmetrisch. Erst, wenn man den Punkt festlegt, an dem man sich wieder trifft, wird festgelegt, welche Richtung die richtige ist. Das bricht die Symmetrie. Je nachdem, ob die Uhren von anfang an in der festgelegten Richtung unterwegs waren oder nicht, vergeht auf ihnen in Summe mehr oder weniger Zeit, bis sie das Zielereignis erreichen. Wie gesagt, lies dir dazu JoAx' Analogie durch, das ist eigentlich einfache Geometrie - wenn man mal den richtigen Abstraktionsschritt gemacht hat.

Dedi
03.12.17, 13:02
Das ist nicht korrekt.

Recht anschaulich finde ich die Betrachtung mittels Austausch von Lichtsignalen:

https://de.wikipedia.org/wiki/Zwillingsparadoxon#Austausch_von_Lichtsignalen

Hier hatte mich gestört das die Reise nur als ganzes dargestellt wird, deswegen hab ich es auf zwei einzelne Reisen aufgeteilt. Beitrag 221 erste Zeichnung rechte Seite.
Das Ergebniß für das Zwillingsparadoxon ist das Gleiche, nur kann man hier sehen das schon die Hinreise Unterschiede in der Alterung aufweist.
Anhand von unterschiedlich eintreffenden Lichtstrahlen ist auch hier die Symmetrie gebrochen.
Diese unterschiedlich eintreffenden Lichtstrahlen können auch ohne das eine Rückreise stattfindet, ermittelt werden.
Die Grafiken sollten immer gleichwertig behandelt werden, man kann nicht einfach behaupten eine geschlossene Hin und Rückreise sei etwas anderes als eine getrennt ablaufende Reise bzw. nur eine Hinreise.
Die Möglichkeiten bei einer einfachen Reise sowohl der Ruhende als auch der Reisende oder auch alles mögliche dazwischen zu sein, würde durch die Messung der Lichtstrahlen ermittelt werden können, damit währe auch hier festgelegt wer sich wie bewegt.
Die Gültigkeit des Relativitätsprinzip währe dann solange gegeben bis eine Messung feststellt wer sich wie bewegt.

Timm
04.12.17, 09:16
Hier hatte mich gestört das die Reise nur als ganzes dargestellt wird, deswegen hab ich es auf zwei einzelne Reisen aufgeteilt. Beitrag 221 erste Zeichnung rechte Seite.
Das Ergebniß für das Zwillingsparadoxon ist das Gleiche, nur kann man hier sehen das schon die Hinreise Unterschiede in der Alterung aufweist.
Anhand von unterschiedlich eintreffenden Lichtstrahlen ist auch hier die Symmetrie gebrochen.

Nein, genau das ist nicht der Fall. Darauf hatte ich dich schon mal hingewiesen.

Wenn man ein Verständnisproblem hat, muß man Schritt für Schritt vorgehen. Die vielen Versuche der letzten Tage dir Hin- und Rückreise mit Rechnungen und Diagrammen nahe zu bringen, laufen ins Leere, solange du darauf besteht, daß schon bei der Hinreise die Symmetrie gebrochen ist. Der erste Schritt ist zu erkennen, daß das falsch ist.

Du siehst anhand der Lichtsignale die Symmetrie auf der Hinreise scheinbar gebrochen. Hier ruht der Zwilling auf der Erde. Symmetrie bedeutet aber, daß du nicht unterscheiden kannst, ob dieser oder oder der Zwilling auf der Hinreise ruht. Oder anders, jeder der beiden sieht den anderen langsamer altern. Hier ist einfach wichtig, das nicht als reale sondern als scheinbare Alterung zu erkennen.

Die Symmetrie ist erst mit Hin- und Rückreise gebrochen. Denn dann ändert sich das System des reisenden Zwillings am Umkehrpunkt, aber das des Zwillings auf der Erde nicht. Und erst dann wird aus der scheinbaren eine reale Alterung.

Dedi
04.12.17, 13:30
Nein, genau das ist nicht der Fall. Darauf hatte ich dich schon mal hingewiesen.

Wenn man ein Verständnisproblem hat, muß man Schritt für Schritt vorgehen. Die vielen Versuche der letzten Tage dir Hin- und Rückreise mit Rechnungen und Diagrammen nahe zu bringen, laufen ins Leere, solange du darauf besteht, daß schon bei der Hinreise die Symmetrie gebrochen ist. Der erste Schritt ist zu erkennen, daß das falsch ist.

Du siehst anhand der Lichtsignale die Symmetrie auf der Hinreise scheinbar gebrochen. Hier ruht der Zwilling auf der Erde. Symmetrie bedeutet aber, daß du nicht unterscheiden kannst, ob dieser oder oder der Zwilling auf der Hinreise ruht. Oder anders, jeder der beiden sieht den anderen langsamer altern. Hier ist einfach wichtig, das nicht als reale sondern als scheinbare Alterung zu erkennen.

Die Symmetrie ist erst mit Hin- und Rückreise gebrochen. Denn dann ändert sich das System des reisenden Zwillings am Umkehrpunkt, aber das des Zwillings auf der Erde nicht. Und erst dann wird aus der scheinbaren eine reale Alterung.

Eine Messung schon auf der Hinreise durchzuführen würde nur Bedeuten Ergebnisse zu bekommen die zwischen der eine Reist und der andere Ruht und der eine Ruht und der andere Reist liegen. Die Messung mag ermitteln wer sich wie bewegt das Ergebnis besteht weiterhin aus allen Möglichkeiten.

Timm
04.12.17, 13:48
Eine Messung schon auf der Hinreise durchzuführen würde nur Bedeuten Ergebnisse zu bekommen die zwischen der eine Reist und der andere Ruht und der eine Ruht und der andere Reist liegen.
Es würde für die Hinreise bedeuten, daß "der eine Reist und der andere Ruht und der eine Ruht und der andere Reist" nicht unterscheidbar ist. Und das bedeutet, daß die Symmetrie nicht gebrochen ist. Du hast aber geschrieben: "Anhand von unterschiedlich eintreffenden Lichtstrahlen ist auch hier die Symmetrie gebrochen."

Dedi
04.12.17, 14:30
Ist die Symmetrie bei Myonen die sich mit 99,92 der Lichtgeschwindigkeit zur Erde bewegen nicht gebrochen?
http://greiterweb.de/spw/Myonen.htm

Timm
04.12.17, 15:23
Die Relativbewegung ist symmetrisch. Aus der Sicht der Erde fliegt das Myon auf sie zu und altert langsamer. Umgekehrt fliegt aus der Sicht des Myons die Erde auf es zu und altert die Erde langsamer.

Dedi
04.12.17, 16:06
Die Relativbewegung ist symmetrisch. Aus der Sicht der Erde fliegt das Myon auf sie zu und altert langsamer. Umgekehrt fliegt aus der Sicht des Myons die Erde auf es zu und altert die Erde langsamer.

Nehmen wir wieder Lichtsignale mit hinzu.
Ausgesendet an der Stelle wo das Myon seine Reise beginnt Richtung Erde und von der Stelle aus an dem die Reise des Myons endet Richtung Myon.
Das Myon reist dem Lichtsignal von der Erde entgegen, die Erde reist dem Signal des Myons nicht entgegen.

Timm
04.12.17, 16:48
Das Myon reist dem Lichtsignal von der Erde entgegen, die Erde reist dem Signal des Myons nicht entgegen.
Doch, aus der Sicht des Myons reist die Erde ihm entgegen. "Aus der Sicht des ..." bedeutet im Ruhesytem des ... Im Ruhesytem des Myons reist die Erde ihm entgegen. Wahrscheinlich erweckt der riesige Unterschied von Erde und Myon falsche Vorstellungen. Du machst es dir komplizierter, bleib besser bei A und B.
Oder stell dir vor, du bist in einem Raumschiff unterwegs und ein anderes kommt dir entgegen. Dann gibt es nichts mehr als eine Relativgeschwindigkeit. Jeder kann behaupten er bewegt sich auf den anderen zu oder der andere auf ihn. Beides ist richtig, denn die Bewegung ist relativ, beide sind gleichberechtigt.

JoAx
04.12.17, 16:51
Eine Messung schon auf der Hinreise durchzuführen würde nur Bedeuten Ergebnisse zu bekommen die zwischen der eine Reist und der andere Ruht und der eine Ruht und der andere Reist liegen. Die Messung mag ermitteln wer sich wie bewegt das Ergebnis besteht weiterhin aus allen Möglichkeiten.

Nur so zur Info - "Spezielle Relativitätstheorie" hat nichts, aber wirklich gar nichts mit "Quantenmechanik" zu tun. ;)

Marco Polo
04.12.17, 17:46
Ist die Symmetrie bei Myonen die sich mit 99,92 der Lichtgeschwindigkeit zur Erde bewegen nicht gebrochen?
http://greiterweb.de/spw/Myonen.htm

Natürlich nicht. Warum sollte hier die Symmetrie gebrochen sein?

Das Myonenbeispiel ist vergleichbar mit der separaten Hinreise oder der separaten Rückreise des Reisezwillings, also ohne den ausschlaggebenden Inertialsystemwechsel bei der Umkehr.

Ist doch völlig wurscht ob es zwei Raumfahrer, oder die Erde und ein Myon oder zwei Käsebrote sind. Das Prinzip ist das Gleiche.

Dedi
04.12.17, 17:49
Nur so zur Info - "Spezielle Relativitätstheorie" hat nichts, aber wirklich gar nichts mit "Quantenmechanik" zu tun. ;)

Sieht nur ähnlich aus, hier ermittelt die Messung was auch ohne Messung schon vorhanden ist.

JoAx
04.12.17, 17:52
Hier hatte mich gestört das die Reise nur als ganzes dargestellt wird,

Das ist eine der Aufgabe adequate Darstellung.


nur kann man hier sehen das schon die Hinreise Unterschiede in der Alterung aufweist.


Vergiss es. Inertiale Bewegung ist relativ. Das bedeutet, dass man sie von Ruhe nicht unterscheiden kann. Das bedeutet, dass es absolute Bewegung nicht gibt. Das hat seiner Zeit Galilei herausgefunden und ist bereits bei Newton so. Wenn du etwas anderes bekommst, dann hast du einen Fehler gemacht. Und du bist derjenige, der diesen Fehler bei sich finden muss.

JoAx
04.12.17, 17:53
Sieht nur ähnlich aus, hier ermittelt die Messung was auch ohne Messung schon vorhanden ist.

Nicht ein mal annährend. Hat ganz und gar nichts mit "Messung" zu tun.

Dedi
04.12.17, 19:31
Nicht ein mal annährend. Hat ganz und gar nichts mit "Messung" zu tun.

Im Zwillingsparadoxon wird auf der Rückreise ein Unterschied in der Ankunft von ausgesandten Lichtsignalen hingewiesen.
Das ist eine Messung, genauso wie der Vergleich der Uhren am Ende der Reise.
Es werden Unterschiede aufgedeckt.
Diese Messung mit Lichtsignalen kann mann auch für die Myonen in einem Gedankenexperiment durchführen.

Es geht mir um eine Gleiche Vorgehensweise wie die Experimente durchgeführt werden.
Wenn Lichtsgnale ein Beweis sind, dann sollte dieser immer gelten.

Wenn alles immer relativ zueinander steht dann würde das Umkehren im Zwillingsparadoxon nur Aufzeigen das etwas anders ist aber alles ohne einen absoluten Betrag, alle Möglichkeiten zwischen der eine bewegt sich und der andere ruht bzw. der eine ruht und der andere bewegt sich würden erhalten bleiben.

Das Experiment ohne relativistische Geschwindigkeit.
Statt Start und Ziel nehme ich einen Zug links und einen rechts, diese beiden sind immer in Ruhe zueinander.
Ein weiterer Zug ruht zu Beginn beim linken Zug, dieser beschleunigt auf 60 km/h Richtung rechtem Zug dort angekommen kehrt er um und nähert sich dem linken Zug wiederum mit 60 km/h. alles Relativgeschwindigkeiten.
In keinem Punkt kann aus relativer Sicht ermittelt werden wann wer welche absolute Geschwindigkeit hatte, nicht mal was überhaupt eine absolute Geschwindigkeit ist , gleiches gilt auch bei der relativistischen Betrachtung.
Ohne festes Bezugssystem gibt es keine absoluten Geschwindigkeiten, auch nicht im relativistischen System.

Slash
04.12.17, 21:35
Hallo,

nimmt man einen dritten Beobachter hinzu (als Schiedsrichter), der sich permanent in einem Inertialsystem befindet, dann kann dieser doch aber feststellen, welcher der beiden Zwillinge während den 10 Jahre mehr "im Raum" bewegt hat und folglich weniger gealtert ist.

Aber ich vermute, die Fragestellung bezog sich auf das Problem, ohne einen 3. Beobachter hinzuzunehmen, richtig?

Slash
09.01.23, 16:09
Special Relativity
https://youtu.be/ZdrZf4lQTSg?t=627

Sabine Hossenfelder erklärt es ggf. besser als ich.

....


In die Berechnung der Zeitdilatation geht selbstverständlich nur die Geschwindigkeit ein.

Problemstellung des Zwillingsparadoxon ist aber nicht nur das der (unterschiedlichen) Zeitdilatation, sondern das des Starts und des gemeinsamen Wiedersehens. Relativ zueinander gesehen haben sie sich jeweils gegenseitig gleich voneinander weg und wieder zu bewegt, dennoch sind sie unterschiedlich gealtert.

Mir geht es auch nicht darum, dass die Beschleunigung Ursache für irgendetwas wäre, sondern das oder ein Unterscheidungsmerkmal zwischen den Zwillingen - denn sie unterscheiden sich doch offensichtlich in der Eigenzeit und dem zurückgelegten Weg - oder ? Wie kann man z.B. bzgl. unterschiedliche Wege zurücklegen, wenn man nicht beschleunigt? Geschwindigkeitssprünge / instane Richtungsändeurngen sind akademisch, aber beschreibbar durch einen Dirac-Impuls * Faktor in der Ableitung.

Ich habe auch nicht die Absicht, irgendetwas anderes zu behaupten, als bspw. in Wikipedia steht
https://de.wikipedia.org/wiki/Zwillingsparadoxon

Wenn ich mal Zeit und Lust finde, rechne ich es nach.

Ich hoffe, "ihr" seht nicht immer rote Tücher oder bekommt es in den falschen Hals, wenn jemand "Beschleunigung" im Zusammenhang mit dem Zwillingsparadoxon oder schreibt.

Immer cool bleiben, Wikipedia tut es ja auch, wie gesagt:
https://de.wikipedia.org/wiki/Zwillingsparadoxon

VG
Slash

HaPe
25.03.23, 16:40
Hallo,

es ist wohl nicht mehr zumutbar, alle Beiträge dieses Threads zu lesen. Sollte ich also etwas wiederholen, dann: Sorry.

Alle mir bekannten Deutungen des Zwillingsparadoxons sind zwar intellektuell erfassbar und soweit auch plausibel, aber so wirklich glücklich wird man IMO damit nicht. Deshalb fand ich die Visualisierung des Sachverhalts durch ein Minkowski-Diagramm enorm hilfreich. Wer das noch nicht kennt (Ihr kennt das natürlich alle) => extrem nützlich zur Deutung des Zwillingsparadoxon.

Und wer tatsächlich ein Bauchgefühl dafür bekommen will: Bei Wikipedia nach „Relativität der Gleichzeitigkeit“ ( https://de.wikipedia.org/wiki/Relativität_der_Gleichzeitigkeit ) suchen. Ganz unten ist eine animierte Grafik „Der beschleunigte Beobachter“, die ich extrem inspirierend fand. Fortgeschrittene Kenntnisse in der Interpretation von Minkowski-Diagrammen sind aber obligatorisch.